You are on page 1of 167

1

18.01 EXERCISES

Unit 1. Dierentiation
1A. Graphing

1A-1 By completing the square, use translation and change of scale to sketch
b) y = 3x2 + 6x + 2
a) y = x2 2x 1
1A-2 Sketch, using translation and change of scale
2
b) y =
a) y = 1 + |x + 2|
(x 1)2
1A-3 Identify each of the following as even, odd , or neither
x3 + 3x
a)
b) sin2 x

1 x4

tan x
c)
d) (1 + x)4
1 + x2

e) J0 (x2 ), where J0 (x) is a function you never heard of

1A-4 a) Show that every polynomial is the sum of an even and an odd function.
b) Generalize part (a) to an arbitrary function f (x) by writing
f (x) + f (x) f (x) + f (x)
+
2
2
Verify this equation, and then show that the two functions on the right are respec
tively even and odd.
f (x) =

c) How would you write

1
as the sum of an even and an odd function?
x+a

1A-5. Find the inverse to each of the following, and sketch both f (x) and the
inverse function g(x). Restrict the domain if necessary. (Write y = f (x) and solve
for y; then interchange x and y.)
x1
a)
b) x2 + 2x
2x + 3
1A-6 Expressin the form A sin (x + c)
a) sin x + 3 cos x b) sin x cos x
1A-7 Find the period , amplitude , and phase angle, and use these to sketch
b) 4 cos (x + /2)
a) 3 sin (2x )
COPYRIGHT DAVID JERISON AND MIT 1996, 2003
1

E. 18.01 Exercises

1. Dierentiation

1A-8 Suppose f (x) is odd and periodic. Show that the graph of f (x) crosses the
x-axis innitely often.
1A-9 a) Graph the function f that consist of straight line segments joining the
points (1, 1), (1, 2), (3, 1), and (5, 2). Such a function is called piecewise linear.
b) Extend the graph of f periodically. What is its period?
c) Graph the function g(x) = 3f ((x/2) 1) 3.

1B. Velocity and rates of change


1B-1 A test tube is knocked o a tower at the top of the Green building. (For
the purposes of this experiment the tower is 400 feet above the ground, and all the
air in the vicinity of the Green building was evacuated, so as to eliminate wind
resistance.) The test tube drops 16t2 feet in t seconds. Calculate
a) the average speed in the rst two seconds of the fall
b) the average speed in the last two seconds of the fall
c) the instantaneous speed at landing
1B-2 A tennis ball bounces so that its initial speed straight upwards is b feet per
second. Its height s in feet at time t seconds is given by s = bt 16t2
a) Find the velocity v = ds/dt at time t.
b) Find the time at which the height of the ball is at its maximum height.
c) Find the maximum height.
d) Make a graph of v and directly below it a graph of s as a function of time.
Be sure to mark the maximum of s and the beginning and end of the bounce.
e) Suppose that when the ball bounces a second time it rises to half the
height of the rst bounce. Make a graph of s and of v of both bounces, labelling
the important points. (You will have to decide how long the second bounce lasts
and the initial velocity at the start of the bounce.)
f) If the ball continues to bounce, how long does it take before it stops?
2

1. Dierentiation

E. 18.01 Exercises

1C. Slope and derivative

1C-1 a) Use the dierence quotient denition of derivative to calculate the rate of
change of the area of a disk with respect to its radius. (Your answer should be the
circumference of the disk.)
b) Use the dierence quotient denition of derivative to calculate the rate of
change of the volume of a ball with respect to the radius. (Your answer should be
the surface area of the ball.)
1C-2 Let f (x) = (x a)g(x). Use the denition of the derivative to calculate that
f (a) = g(a), assuming that g is continuous.

1C-3 Calculate the derivative of each of these functions directly from the denition.
2
a) f (x) = 1/(2x + 1)
b) f (x) = 2x
+ 5x + 4
2
c) f (x) = 1/(x + 1)
d) f (x) = 1/ x
e) For part (a) and (b) nd points where the slope is +1, 1, 0.
1C-4 Write an equation for the tangent line for the following functions
2
b) f (x) = 2x
+ 5x + 4 at x = a
a) f (x) = 1/(2x + 1) at x = 1
2
d) f (x) = 1/ x at x = a
c) f (x) = 1/(x + 1) at x = 0

1C-5 Find all tangent lines through the origin to the graph of y = 1 + (x 1)2 .

1C-6 Graph the derivative of the following functions directly below the graph of
the function. It is very helpful to know that the derivative of an odd function is
even and the derivative of an even function is odd (see 1F-6).
a) semicircle

b) parabola

c) odd function

d) even function

e) periodic; period = ?
-2

4
2
-2

1D. Limits and continuity


3

E. 18.01 Exercises

1. Dierentiation

1D-1 Calculate the following limits if they exist. If they do not exist, then indicate
whether they are +, or undened.

4
x 0 x 1
4x2
d) lim
x2+ 2 x
4x2
4x
g) lim
x x 2

4x
x 2 x + 1
4x2
e) lim
x2 2 x
x2 + 2x + 3
i) lim
x 3x2 2x + 4

a) lim

b) lim

4x2
x2 x + 2
4x2

f) lim

x x 2
x2
j) lim 2
x2 x 4
c) lim

1D-2 For which of the following should one use the one-sided limit? Evaluate it.
a)

lim

x0

b)

1
x1 x 1
lim

c)

1
x1 (x 1)4
lim

d)

lim | sin x|

x0

e)

|x|
x0 x
lim

1D-3 Identify and give the type of the points of discontinuity of each of the
following:
a)

x2
x2 4

b)

c)

x4
x3

x + a, x > 0
d) f (x) =
a x, x < 0

e) f (x), for the f (x) in d)

f)

1
sin x

(f (x))2 ,

where f (x) =

d
|x|
dx

1D-4 Graph the following functions.


4x2
1
b) 2
a)
(See 1D-1efg.)
x2
x + 2x + 2

1D-5 Dene f (x) =

ax + b, x 1;
x2 , x < 1.

a) Find all values of a, b such that f (x) is continuous.

b) Find all values of a, b such that f (x) is continuous. (Be careful!)

1D-6 For each of the following functions, nd all values of the constants a and b
for which the function is dierentiable.

a) f (x) =

x2 + 4x + 1, x 0;
ax + b, x < 0.

b) f (x) =
4

x2 + 4x + 1, x 1;
ax + b, x < 1.

1. Dierentiation

E. 18.01 Exercises

1D-7 Find the values of the constants a, b and c for which the following function
is dierentiable. (Give a and b in terms of c.)

cx2 + 4x + 1, x 1;
f (x) =
ax + b, x < 1.
1D-8 For each of the following functions, nd the values of the constants a and b
for which the function is continuous, but not dierentiable.

a) f (x) =

ax + b, x > 0;
sin 2x, x 0.

b) f (x) =

ax + b, x > 0;
cos 2x, x 0.

1D-9 Find the values of the constants a and b for which the following function is
dierentiable, but not continuous.

ax + b, x > 0;
f (x) =
cos 2x, x 0.
1D-10* Show that
f (a + h) f (a)
g(h) =
h

has a removable discontinuity at h = 0

f (a) exists.

1E. Dierentiation formulas: polynomials, products, quotients

1E-1 Find the derivative of the following polynomials.


b) e2 + 1 ( e = base of natural logs)
a) x10 + 3x5 + 2x3 + 4
3
c) x/2 +
d) (x3 + x)(x5 + x2 )
1E-2 Find the antiderivative of the following polynomials.
b) x
6 + 5x5 + 4x3
a) ax + b (a and b are constants.)
3
2

c) (x + 1)

1E-3 Find the points (x, y) of the graph y = x3 + x2 x + 2 at which the slope of
the tangent line is horizontal.
1E-4 For each of the following, nd all values of a and b for which f (x) is dier
entiable.

a) f (x) =

ax2 + bx + 4, x 0;
5x5 + 3x4 + 7x2 + 8x + 4, x > 0.

b) f (x) =

1E-5 Find the derivatives of the following rational functions.

a)

x
1+x

b)

x+a
(a is constant)
x2 + 1

ax2 + bx + 4, x 1;
5x5 + 3x4 + 7x2 + 8x + 4, x > 1.

E. 18.01 Exercises
c)

x+2
x2 1

1. Dierentiation
d)

x4 + 1
x

1F. Chain rule, implicit dierentiation

1F-1 Find the derivative of the following functions:


a) (x2 + 2)2

(two methods)

b) (x2 + 2)100 . Which of the two methods from part (a) do you prefer?
1F-2 Find the derivative of x10 (x2 + 1)10 .
1F-3 Find dy/dx for y = x1/n by implicit dierentiation.
1F-4 Calculate dy/dx for x1/3 + y 1/3 = 1 by implicit dierentiation. Then solve
for y and calculate y using the chain rule. Conrm that your two answers are the
same.

1F-5 Find all points of the curve(s) sin x + sin y = 1/2 with horizontal tangent
lines. (This is a collection of curves with a periodic, repeated pattern because the
equation is unchanged under the transformations y y + 2 and x x + 2.)

1F-6 Show that the derivative of an even function is odd and that the derivative
of an odd function is even.
(Write the equation that says f is even, and dierentiate both sides, using the
chain rule.)

1F-7 Evaluate the derivatives. Assume all letters represent constants, except for
the independent and dependent variables occurring in the derivative.

a) D =

(x a)2 + y0 2 ,

mg
,
c) F =
(1 + r2 )3/2

dD
=?
dx

dF
=?
dr

b) m =

m0
v 2 /c2

1
at
d) Q =
,
(1 + bt2 )3

dm
=?
dv
dQ
=?
dt

1F-8 Evaluate the derivative by implicit dierentiation. (Same assumptions about


the letters as in the preceding exercise.)
6

1. Dierentiation

E. 18.01 Exercises

1 2
dr
r h,
=?
3
dh
c) c2 = a2 + b2 2ab cos ,

a) V =

b) P V c = nRT,
da
=?
db

dP
=?
dV

1G. Higher derivatives


1G-1 Calculate y for the following functions.

a) 3x2 + 2x + 4 x
c)

5
x + 5

x
x + 5

x2 + 5x

d)
x+5

b)

1G-2 Find all functions f (x) whose third derivative f (x) is identically zero.
(Identically is math jargon for always or for every value of x.)
1G-3 Calculate y using implicit dierentiation and simplify as much as possible.
x2 a2 + y 2 b2 = 1
1G-4 Find the formula for the nth derivative y (n) of y = 1/(x + 1).
1G-5 Let y = u(x)v(x).
a) Find y , y , and y .
b) The general formula for y (n) , the n-th derivative, is called Leibniz formula:
it uses the same coecients as the binomial theorem , and looks like


n (n1) (1)
n (n2) (2)
u
v +
u
v + ... + uv (n)
y (n) = u(n) v +
1
2
Use this to check your answers in part (a), and use it to calculate y (p+q) , if y =
xp (1 + x)q .

1H. Exponentials and Logarithms: Algebra

1H-1 The half-life of a radioactive substance decaying according to the law


y = y0 ekt is dened to be the time it takes the amount to decrease to 1/2 of the
initial amount y0 .
a) Express the half-life in terms of k. (Do this from scratch dont just plug
into formulas given here or elsewhere.)
7

E. 18.01 Exercises

1. Dierentiation

b) Show using your expression for that if at time t1 the amount is y1 , then at
time t1 + it will be y1 /2, no matter what t1 is.
1H-2 If a solution containing a heavy concentration of hydrogen ions (i.e., a strong
acid) is diluted with an equal volume of water, by approximately how much is its
pH changed? (Express (pH)diluted in terms of (pH)original .)
1H-3 Solve the following for y:
b) log(y + 1) = x2 + log(y 1)

a) ln(y + 1) + ln(y 1) = 2x + ln x
c) 2 ln y = ln(y + 1) + x

1H-4 Solve

ln a
= c for a in terms of b and c; then repeat, replacing ln by log.
ln b

1H-5 Solve for x (hint: put u = ex , solve rst for u):

a)

ex + ex
= y
ex ex

b) y = ex + ex

1H-6 Evaluate from scratch the number A = log e ln 10. Then generalize the
problem, and repeat the evaluation.
1H-7 The decibel scale of loudness is
L = 10 log10 (I/I0 )
where I, measured in watts per square meter, is the intensity of the sound and
I0 = 1012 watt/m2 is the softest audible sound at 1000 hertz. Classical music
typically ranges from 30 to 100 decibels. The human ears pain threshold is about
120 decibels.
a) Suppose that a jet engine at 50 meters has a decibel level of 130, and a
normal conversation at 1 meter has a decibel level of 60. What is the ratio of the
intensities of the two sounds?
b) Suppose that the intensity of sound is proportional to the inverse square
of the distance from the sound. Based on this rule, calculate the decibel level of
the sound from the jet at a distance of 100 meters, at distance of 1 km.1
1The inverse square law is justied by the fact that the intensity is measured in energy per

unit time per unit area. When the sound has travelled a distance r, the energy of a sound spread
over a sphere of radius r centered at the source. The area of that sphere is proportional to r2 , so
the average intensity is proportional to 1/r2 . Fortunately for people who live near airports, sound

1. Dierentiation

E. 18.01 Exercises

1H-8* The mean distance of each of the planets to the Sun and their mean period
of revolution is as follows.2 (Distance is measured in millions of kilometers and time
in Earth years.)
9Mercury
57.9
0.241

Venus
108
0.615

Earth
150
1.00

Mars
228
1.88

Jupiter
778
11.9

Saturn
1, 430
29.5

Uranus
2, 870
84.0

Neptune
4, 500
165

Pluto
5, 900
248

a) Find the pattern in these data by making a graph of (ln x, ln y) where x is


the distance to the Sun and y is the period of revolution for the rst four points
(Mercury through Mars). Observe that these points are nearly on a straight line.
Plot a line with ruler and estimate its slope. (You can check your estimated slope
by calculating slopes of lines connecting consecutive data points.)
b) Using an approximation to the slope m that you found in part (a) accurate
to two signicant gures, give a formula for y in the form
ln y = m ln x + c
(Use the Earth to evaluate c.)
c) Solve for y and make a table for the predicted values of the periods of
revolutions of all the planets based on their distance to the Sun. (Your answers
should be accurate to one percent.)
d) The Earth has radius approximately 6, 000 km and the Moon is at a
distance of about 382, 000 km. The period of revolution of the Moon is a lunar
month, say 28 days. Assume that the slope m is the same for revolution around the
Earth as the on you found for revolution around the Sun in (a). Find the distance
above the surface of the Earth of geosynchronous orbit, that is, the altitude of the
orbit of a satellite that stays above one place on the equator. (For satellites this
close to Earth it is important to know that y is predicting the distance from the
satellite to the center of the Earth. This is why you need to know the radius of the
Earth.)
e) Find the period of revolution of a satellite that circles at an altitude of
1, 000 km.

1I. Exponential and Logarithms: Calculus

1I-1 Calculate the derivatives


b) (2x 1)e2x
a) xex

c) ex

doesnt travel as well as this. Part of the energy is dissipated into heating the air and another
part into vibration of insulating materials on the way to the listeners ear.
2
from Fundamentals of Physics, vol. 1, by D. Halliday and R. Resnick

10

E. 18.01 Exercises

1. Dierentiation

d) x ln x x
e) ln(x2 )
x2 2
g) (e )
h) xx
x
x
k) ln(1/x)
j) (e e )/2
m) (1 ex )/(1 + ex )
1I-2 Graph the function y = (ex + ex )/2.
1
1I-3 a) Evaluate lim n ln(1 + ).
n
n
Hint: Let h = 1/n, and use (d/dx) ln(1 + x)|x=0 = 1.
1
b) Deduce that lim (1 + )n = e.
n
n
1
1I-4 Using lim (1 + )n = e, calculate
n
n
1
2
a) lim (1 + )3n
b) lim (1 + )5n
n
n
n
n

f) (ln x)2
i) (ex + ex )/2
l) 1/ ln x

c) lim (1 +
n

1 5n
)
2n

1I-5* If you invest P dollars at the annual interest rate r, then after one year the
interest is I = rP dollars, and the total amount is A = P + I = P (1 + r). This is
simple interest.
For compound interest, the year is divided into k equal time periods and the
interest is calculated and added to the account at the end of each period. So at
the end of the rst period, A = P (1 + r( k1 )); this is the new amount for the second
period, at the end of which A = P (1 + r( k1 ))(1 + r( k1 )), and continuing this way, at
the end of the year the amount is

r k
A=P 1+
.
k
The compound interest rate r thus earns the same in a year as the simple interest
rate of

r k
1+
1 ;
k
this equivalent simple interest rate is in bank jargon the annual percentage rate
or APR.3
a) Compute the APR of 5% compounded monthly, daily,4 and continuously.
Continuous compounding means the limit as k tends to innity.
b) As in part (a), compute the APR of 10% compounded monthly, biweekly
(k=26), daily, and continuously. (We have thrown in the biweekly rate because
loans can be paid o biweekly.)
3Banks are required to reveal this so-called APR when they oer loans. The APR also takes
into account certain bank fees known as points. Unfortunately, not all fees are included in it, and
the true costs are higher if the loan is paid o early.
4For daily compounding assume that the year has 365 days, not 365.25. Banks are quite careful
about these subtle dierences. If you look at ocial tables of rates from precalculator days you
will nd that they are o by small amounts because U.S. regulations permitted banks to pretend
that a year has 360 days.

10

11

1. Dierentiation

E. 18.01 Exercises

1J. Trigonometric functions

1J-1 Calculate the derivatives of the following functions


b) sin2 (3x)
c) ln(cos(2x))
sin x
e)
f) cos(x + y); y = f (x)
d) ln(2 cos x)
x2
h) esin x
g) cos(x + y); y constant
i) ln(x2 sin x)
2
2x
j) e sin(10x)
l) sec 1 x2
k) tan (3x)
m) The following three functions have the same derivative: cos(2x), cos2 x sin2 x,
and 2 cos2 x. Verify this. Are the three functions equal? Explain.

2
2
n) sec(5x)
tan(5x)
o)
sec
(3x)

tan
(3x)
p)
sin(
x2 + 1)

x
2
2
2
q) cos ( 1 x )
r) tan (
)
x+1
a) sin(5x2 )

1J-2 Calculate lim

x/2

cos x
by relating it to a value of (cos x) .
x /2

1J-3 a) Let a > 0 be a given constant. Find in terms of a the value of k > 0 for
which y = sin(kx) and y = cos(kx) both satisfy the equation
y + ay = 0.

Use this value of k in each of the following parts.


b) Show that y = c1 sin(kx) + c2 cos(kx) is also a solution to the equation in
(a), for any constants c1 and c2 .
c) Show that the function y = sin(kx + ) (whose graph is a sine wave with
phase shift ) also satises the equation in (a), for any constant .
d) Show that the function in (c) is already included among the functions of
part (b), by using the trigonometric addition formula for the sine function. In other
words, given k and , nd values of c1 and c2 for which
sin(kx + ) = c1 sin(kx) + c2 cos(kx)
1J-4 a) Show that a chord of the unit circle with angle has length
Deduce from the half-angle formula

1 cos
sin(/2) =
2
that the length of the chord is
2 sin(/2)
11

2 2 cos .

12

E. 18.01 Exercises

1. Dierentiation

b) Calculate the perimeter of an equilateral n-gon with vertices at a distance


1 from the center. Show that as n tends to innity, the perimeter tends to 2, the
circumference of the unit circle.

12

13

MIT OpenCourseWare
http://ocw.mit.edu

18.01SC Single Variable Calculus


Fall 2010

For information about citing these materials or our Terms of Use, visit: http://ocw.mit.edu/terms.

14

SOLUTIONS TO 18.01 EXERCISES

Unit 1. Dierentiation
1A. Graphing
1A-1,2 a) y = (x 1)2 2
b) y = 3(x2 + 2x) + 2 = 3(x + 1)2 1

2
1

-2

1b

1a

1A-3 a) f (x) =

-2

-1

2a

2b

(x)3 3x
x3 3x
=
= f (x), so it is odd.
1 (x)4
1 x4

b) (sin(x))2 = (sin x)2 , so it is even.

c)

odd

, so it is odd
even

(1 + x)4 : neither.
d) (1 x)4 =
e) J0 ((x)2 ) = J0 (x2 ), so it is even.
1A-4 a) p(x) = pe (x) + po (x), where pe (x) is the sum of the even powers and po (x)
is the sum of the odd powers

b) f (x) =

F (x) =

f (x) + f (x) f (x) f (x)


+
2
2

f (x) f (x)
f (x) + f (x)
is even and G(x) =
is odd because
2
2

COPYRIGHT DAVID JERISON AND MIT 1996, 2003


1

15

E. Solutions to 18.01 Exercises

F (x) =

1. Dierentiation

f (x) + f ((x))
= F (x);
2

G(x) =

f (x) f (x)
= G(x).
2

c) Use part b:

1
1
2a
2a

+
=
= 2
(x + a)(x + a)
a x2
x + a x + a

even

2x
2x
1
1

=
= 2
(x + a)(x + a)
a x2

x + a x + a

odd

1
a
x

2
= 2
2
a x
x+a
a x2

x1
3y + 1
. Crossmultiply and solve for x, getting x =
, so the
1 2y
2x + 3
3x + 1
inverse function is
.
1 2x
1A-5 a) y =

b) y = x2 + 2x = (x + 1)2 1
y = x,
(Restrict domain to x 1, so when its ipped about the diagonal

youll still get the graph ofa function.) Solving for x, we get x = y + 1 1, so
the inverse function is y = x + 1 1 .

g(x)

g(x)

f(x)
f(x)

5b

5a

1A-6 a) A = 1 + 3 = 2, tan c =

b)

3
,c=
1

3.

So sin x +

3 cos x = 2 sin(x + 3 ) .

2 sin(x )
4

1A-7 a) 3 sin(2x ) = 3 sin 2(x

b) 4 cos(x +

), amplitude 3, period , phase angle /2.


2

) = 4 sin x amplitude 4, period 2, phase angle 0.


2
2

16

1. Dierentiation

E. Solutions to 18.01 Exercises

-4

-3

7a

7b

1A-8
f (x) odd = f (0) = f (0) = f (0) = 0.
So f (c) = f (2c) = = 0, also (by periodicity, where c is the period).
1A-9
3
2
-8
-7

-5

-3

-1

-4

12

-1

9c

9ab period = 4

-6

c) The graph is made up of segments joining (0, 6) to (4, 3) to (8, 6). It


repeats in a zigzag with period 8. * This can be derived using:
(1)

x/2 1 = 1 = x = 0 and g(0) = 3f (1) 3 = 6

(2)

x/2 1 = 1 = x = 4 and g(4) = 3f (1) 3 = 3

(3)

x/2 1 = 3 = x = 8 and g(8) = 3f (3) 3 = 6

(4)
1B. Velocity and rates of change
1B-1 a) h = height of tube = 400 16t2 .
average speed

h(2) h(0)
(400 16 22 ) 400
=
= 32ft/sec
2
2

(The minus sign means the test tube is going down. You can also do this whole
problem using the function s(t) = 16t2 , representing the distance down measured
from the top. Then all the speeds are positive instead of negative.)
b) Solve h(t) = 0 (or s(t) = 400) to nd landing time t = 5. Hence the average
speed for the last two seconds is
h(5) h(3)
0 (400 16 32 )
=
= 128ft/sec
2
2
3

17

E. Solutions to 18.01 Exercises

1. Dierentiation

c)
16(5 t)(5 + t)
h(t) h(5)
400 16t2 0
=
=
t5
t5
t5
= 16(5 + t) 160ft/sec as t 5

(5)
(6)

1B-2 A tennis ball bounces so that its initial speed straight upwards is b feet per
second. Its height s in feet at time t seconds is
s = bt 16t2

a)
(7)
(8)
(9)
(10)

b(t + h) 16(t + h)2 (bt 16t2 )


s(t + h) s(t)
=
h
h
bt + bh 16t2 32th 16h2 bt + 16t2
=
h
bh 32th 16h2
=
h
= b 32t 16h b 32t as h 0

Therefore, v = b 32t.
b) The ball reaches its maximum height exactly when the ball has nished
going up. This is time at which v(t) = 0, namely, t = b/32.
c) The maximum height is s(b/32) = b2 /64.
d) The graph of v is a straight line with slope 32. The graph of s is a parabola
with maximum at place where v = 0 at t = b/32 and landing time at t = b/16.
v
b
s
b/32
t
graph of velocity

b/32
b/16
graph of position

e) If the initial velocity on the rst bounce was b1 = b, and the


velocity of the
second bounce is b2 , then b22 /64 = (1/2)b21 /64. Therefore, b2 = b1 / 2. The second
(continued )
bounce is at b1 /16 + b2 /16.
4

18

1. Dierentiation

E. Solutions to 18.01 Exercises

f) If the ball continues to bounce then the landing times form a geometric
series

(11)
b1 /16 + b2 /16 + b3 /16 + = b/16 + b/16 2 + b/16( 2)2 +

(12)
= (b/16)(1 + (1/ 2) + (1/ 2)2 + )
(13)

b/16

1 (1/ 2)

Put another way, the ball stops bouncing after 1/(1 (1/ 2)) 3.4 times the
length of time the rst bounce.
1C. Slope and derivative.
1C-1 a)
(r + h)2 r2
(r2 + 2rh + h2 ) r2
(2rh + h2 )
=
=
h
h
h
= (2r + h)

(14)
(15)

2r as h 0

(16)

b)

(19)

(4/3)(r + h)3 (4/3)r3


(4/3)(r3 + 3r2 h + 3rh2 + h3 ) (4/3)r3
=
h
h
(4/3)(3r2 h + 3rh2 + h3 )
=
h
= (4/3)(3r2 + 3rh + h2 )

(20)

4r2 as h 0

(17)
(18)

1C-2

f (x) f (a)
(x a)g(x) 0
=
= g(x) g(a) as x a.
xa
xa

1C-3 a)
(21)
(22)
(23)
(24)

1
1
1
1 2x + 1 (2(x + h) + 1)

=
h 2(x + h) + 1 2x + 1
h (2(x + h) + 1)(2x + 1)

1
2h
=
h (2(x + h) + 1)(2x + 1)
2
=
(2(x + h) + 1)(2x + 1)
2
as h 0

(2x + 1)2
5

19

E. Solutions to 18.01 Exercises

1. Dierentiation

b)
(25)
2x2 + 4xh + 2h2 + 5x + 5h 2x2 5x
2(x + h)2 + 5(x + h) + 4 (2x2 + 5x + 4)
=
h
h

4xh + 2h2 + 5h

(26)
=
= 4x + 2h + 5
h
(27)
4x + 5 as h 0

c)
(28)
(29)
(30)
(31)
(32)

1
1
1
1 (x2 + 1) ((x + h)2 + 1)

=
h
((x + h)2 + 1)(x2 + 1)
h (x + h)2 + 1 x2 + 1
2

1 x + 1 x2 2xh h2 1
=
h
((x + h)2 + 1)(x2 + 1)

1
2xh h2
=
h ((x + h)2 + 1)(x2 + 1)
2x h
=
((x + h)2 + 1)(x2 + 1)
2x
2
as h 0
(x + 1)2

d) Common denominator:

x x+h
1
1
1
1

h
h
x
x+h
x + h x

numerator and denominator by


Now
simplify the numerator by multiplying
x + h, and using (a b)(a + b) = a2 b2 :

1
x (x + h)
1
( x)2 ( x + h)2

=
(33)


h
h
x + h x( x + x + h)
x + h x( x + x + h)

h
1

(34)
=

h
x + h x( x + x + h)

(35)
=

x + h x( x + x + h)
1
1
3 = x3/2 as h 0
(36)
2
2( x)

x +

e) For part (a), 2/(2x + 1)2 < 0, so there are no points where the slope is 1
or 0. For slope 1,

2/(2x + 1)2 = 1 = (2x + 1)2 = 2 = 2x + 1 = 2 = x = 1/2 2/2

For part (b), the slope is 0 at x = 5/4, 1 at x = 1 and 1 at x = 3/2.


6

20

1. Dierentiation

E. Solutions to 18.01 Exercises

1C-4 Using Problem 3,


a) f (1) = 2/9 and f (1) = 1/3, so y = (2/9)(x 1) + 1/3 = (2x + 5)/9
b) f (a) = 2a2 + 5a + 4 and f (a) = 4a + 5, so

y = (4a + 5)(x a) + 2a2 + 5a + 4 = (4a + 5)x 2a2 + 4

c) f (0) = 1 and f (0) = 0, so y = 0(x 0) + 1, or y = 1.

d) f (a) = 1/ a and f (a) = (1/2)a3/2 , so

y = (1/2)a3/2 (x a) + 1/ a = a3/2 x + (3/2)a1/2

1C-5 Method 1. y (x) = 2(x 1), so the tangent line through (a, 1 + (a 1)2 ) is

y = 2(a 1)(x a) + 1 + (a 1)2

In order to see if the origin is on this line, plug in x = 0 and y = 0, to get the

following equation for a.


0 = 2(a 1)(a) + 1 + (a 1)2 = 2a2 + 2a + 1 + a2 2a + 1 = a2 + 2

Therefore a = 2 and the two tangent lines through the origin are

y = 2( 2 1)x and y = 2( 2 + 1)x


(Because these are lines throught the origin, the constant terms must cancel: this
is a good check of your algebra!)
Method 2. Seek tangent lines of the form y = mx. Suppose that y = mx
meets y = 1 + (x 1)2 , at x = a, then ma = 1 + (a 1)2 . In addition we want the
slope y (a) = 2(a 1) to be equal to m, so m = 2(a 1). Substituting for m we
nd
2(a 1)a = 1 + (a 1)2

Thisis the same equation as in method 1: a2 2 = 0, so a = 2 and m =


2( 2 1), and the two tangent lines through the origin are as above,

y = 2( 2 1)x and y = 2( 2 + 1)x

1D. Limits and continuity


1D-1 Calculate the following limits if they exist. If they do not exist, then indicate
whether they are +, or undened.
a) 4
b) 8/3

21

E. Solutions to 18.01 Exercises

1. Dierentiation

c) undened (both are possible)


d) Note that 2 x is negative when x > 2, so the limit is
e) Note that 2 x is positive when x < 2, so the limit is + (can also be
written )

f)

4x2
4x

= as x
=
1
x2
1 (2/x)

g)

4x2
4x2 4x(x 2)
8x
8
8 as x
4x =
=
=
x2
x2
x2
1 (2/x)

i)

1 + (2/x) + (3/x2 )
1
x2 + 2x + 3
as x
=
3 (2/x) + 4/x2 )
3
3x2 2x + 4

j)

x2
x2
1
1
as x 2
=
=
2
(x 2)(x + 2)
x+2
4
x 4

1D-2 a) lim

x 0

x=0

b) lim

x1+

1
=
x1

lim

x1

1
=
x1

c) lim (x 1)4 = (left and right hand limits are same)


x 1

d) lim | sin x| = 0 (left and right hand limits are same)


x0

e) lim

x0+

|x|
=1
x

lim

x0

1D-3 a) x = 2 removable
innite
c) x = 0 removable
removable

|x|
= 1
x
x = 2 innite

d) x = 0 removable

b) x = 0, , 2, ...

e) x = 0 jump

(1,1)
(0,.5)

4b

4a
8

f) x = 0

22

1. Dierentiation

E. Solutions to 18.01 Exercises

1D-5 a) for continuity, want ax + b = 1 when x = 1.


a+b=1

Ans.: all a, b such that

dy
d(x2 )
d(ax + b)
= 2x = 2 when x = 1 . We have also
= a.
=
dx
dx
dx
Therefore, to make f (x) continuous, we want a = 2.
b)

Combining this with the condition a + b=1 from part (a), we get nally b = 1,
a = 2.
1D-6 a) f (0) = 02 + 4 0 + 1 = 1. Match the function values:
f (0 ) = lim ax + b = b,
x 0

so b = 1 by continuity.

Next match the slopes:


f (0+ ) = lim 2x + 4 = 4

x 0

and f (0 ) = a. Therefore, a = 4, since f (0) exists.


b)

f (1) = 12 + 4 1 + 1 = 6 and f (1 ) = lim ax + b = a + b

x 1

Therefore continuity implies a + b = 6. The slope from the right is


f (1+ ) = lim 2x + 4 = 6
x 1

Therefore, this must equal the slope from the left, which is a. Thus, a = 6 and
b = 0.
1D-7
f (1) = c12 + 4 1 + 1 = c + 5 and f (1 ) = lim ax + b = a + b
x 1

Therefore, by continuity, c + 5 = a + b. Next, match the slopes from left and right:
f (1+ ) = lim 2cx + 4 = 2c + 4 and f (1 ) = lim a = a
x 1

x 1

Therefore,
a = 2c + 4 and b = c + 1.
1D-8
a)

f (0) = sin(2 0) = 0 and f (0+ ) = lim ax + b = b

x 0

Therefore, continuity implies b = 0. The slope from each side is


f (0 ) = lim 2 cos(2x) = 2 and f (0+ ) = lim a = a
x 0

x 0

Therefore, we need a =
2 in order that f not be dierentiable.
9

23

E. Solutions to 18.01 Exercises

1. Dierentiation

b)

f (0) = cos(2 0) = 1 and f (0+ ) = lim ax + b = b

x 0

Therefore, continuity implies b = 1. The slope from each side is


f (0 ) = lim 2 sin(2x) = 0 and f (0+ ) = lim a = a
x0

x 0

Therefore, we need a =
0 in order that f not be dierentiable.
1D-9 There cannot be any such values because every dierentiable function is
continuous.
1E: Dierentiation formulas: polynomials, products, quotients
1E-1 Find the derivative of the following polynomials
a) 10x9 + 15x4 + 6x2
b) 0 (e2 + 1 8.4 is a constant and the derivative of a constant is zero.)
c) 1/2
d) By the product rule: (3x2 + 1)(x5 + x2 ) + (x3 + x)(5x4 + 2x) = 8x7 + 6x5 +
5x + 3x2 . Alternatively, multiply out the polynomial rst to get x8 + x6 + x5 + x3
and then dierentiate.
4

1E-2 Find the antiderivative of the following polynomials


a) ax2 /2 + bx + c, where a and b are the given constants and c is a third
constant.
b) x7 /7 + (5/6)x6 + x4 + c
c) The only way to get at this is to multiply it out: x6 + 2x3 + 1. Now you
can take the antiderivative of each separate term to get
x7
x4
+
+x+c
7
2
Warning: The answer is not (1/3)(x3 + 1)3 . (The derivative does not match if you
apply the chain rule, the rule to be treated below in E4.)
1E-3 y = 3x2 + 2x 1 = 0 = (3x 1)(x + 1) = 0. Hence x = 1/3 or x = 1
and the points are (1/3, 49/27) and (1, 3)
10

24

1. Dierentiation

E. Solutions to 18.01 Exercises

1E-4 a) f (0) = 4, and f (0 ) = lim 5x5 + 3x4 + 7x2 + 8x + 4 = 4. Therefore the


x 0

function is continuous for all values of the parameters.


f (0+ ) = lim 2ax + b = b and f (0 ) = lim 25x4 + 12x3 + 14x + 8 = 8
x 0

x 0

Therefore, b = 8 and a can have any value.


b) f (1) = a + b + 4 and f (1+ ) = 5 + 3 + 7 + 8 + 4 = 27. So by continuity,
a + b = 23
f (1 ) = lim 2ax + b = 2a + b;

f (1+ ) = lim 25x4 + 12x3 + 14x + 8 = 59.

x 1

x1

Therefore, dierentiability implies


2a + b = 59
Subtracting the rst equation, a = 59 23 = 36 and hence b = 13.

1E-5 a)

1
(1 + x)2

b)

1 2ax x2
(x2 + 1)2

c)

x2 4x 1
(x2 1)2

d) 3x2 1/x2
1F. Chain rule, implicit dierentiation
1F-1 a) Let u = (x2 + 2)
d 2
du d 2
u = (2x)(2u) = 4x(x2 + 2) = 4x3 + 8x
u =
dx
dx du
Alternatively,
d 2
d 4
(x + 2)2 =
(x + 4x2 + 4) = 4x3 + 8x
dx
dx
d 100
du d 100
u
=
u
= (2x)(100u99 ) = (200x)(x2 + 2)99 .
b) Let u = (x2 + 2); then
dx
dx du
1F-2 Product rule and chain rule:
10x9 (x2 + 1)10 + x10 [10(x2 + 1)9 (2x)] = 10(3x2 + 1)x9 (x2 + 1)9
1F-3 y = x1/n = y n = x = ny n1 y = 1. Therefore,
y =

1
1
1 1
= y 1n = x n 1
ny n1
n
n

1F-4 (1/3)x2/3 + (1/3)y 2/3 y = 0 implies


y = x2/3 y 2/3
Put u = 1 x1/3 . Then y = u3 , and the chain rule implies
du
dy
= 3(1 x1/3 )2 ((1/3)x2/3 ) = x2/3 (1 x1/3 )2
= 3u2
dx
dx
11

25

E. Solutions to 18.01 Exercises

1. Dierentiation

The chain rule answer is the same as the one using implicit dierentiation because
y = (1 x1/3 )3 = y 2/3 = (1 x1/3 )2
1F-5 Implicit dierentiation gives cos x + y cos y = 0. Horizontal slope means
y = 0, so that cos x = 0. These are the points x = /2 + k for every integer k.
Recall that sin(/2 + k) = (1)k , i.e., 1 if k is even and 1 if k is odd. Thus
at x = /2 + k, 1 + sin y = 1/2, or sin y = 1 + 1/2. But sin y = 3/2 has no
solution, so the only solutions are when k is even and in that case sin y = 1 + 1/2,
so that y = /6 + 2n or y = 7/6 + 2n. In all there are two grids of points at
the vertices of squares of side 2, namely the points
(/2 + 2k, /6 + 2n) and (/2 + 2k, 7/6 + 2n);

k, n any integers.

1F-6 Following the hint, let z = x. If f is even, then f (x) = f (z) Dierentiating
and using the chain rule:
f (x) = f (z)(dz/dx) = f (z)

because dz/dx = 1

But this means that f is odd. Similarly, if g is odd, then g(x = g(z). Dierenti
ating and using the chain rule:
g (x) = g (z)(dz/dx) = g (z)
1F-7 a)

because dz/dx = 1

dD
1
xa
= ((x a)2 + y0 2 )1/2 (2(x a)) =
dx
2
(x a)2 + y0 2

b)

dm
1
2v
m0 v
= m0
(1 v 2 /c2 )3/2 2 = 2
dv
2
c
c (1 v 2 /c2 )3/2

c)

dF
3
3mgr
= mg ( )(1 + r2 )5/2 2r =
dr
2
(1 + r2 )5/2

d)

dQ
6bt
a
a(1 5bt2 )
+
=
= at
(1 + bt2 )4
(1 + bt2 )3
(1 + bt2 )4
dt

1F-8 a) V =

1
r2
r
1 2
=
r h = 0 = (2rr h + r2 ) = r =
3
3
2rh
2h

b) P V c = nRT = P V c + P cV c1 = 0 = P =

cP V c1
cP
=
c
V
V

c) c2 = a2 + b2 2ab cos implies


0 = 2aa + 2b 2(cos (a b + a)) = a =

1G. Higher derivatives


12

a cos b
2b + 2 cos a
=
a b cos
2a 2 cos b

26

1. Dierentiation

E. Solutions to 18.01 Exercises

1G-1 a) 6 x3/2

b)

10
(x + 5)3

c)

10
(x + 5)3

d) 0

1G-2 If y = 0, then y = c0 , a constant. Hence y = c0 x + c1 , where c1 is some


other constant. Next, y = c0 x2 /2 + c1 x + c2 , where c2 is yet another constant.
Thus, y must be a quadratic polynomial, and any quadratic polynomial will have
the property that its third derivative is identically zero.
1G-3

y2
2x 2yy
x2
+
=
1
=

+ 2 = 0 = y = (b2 /a2 )(x/y)


b2
b
a2
a2

Thus,

(37)
(38)

b2
y xy
b
y + x(b2 /a2 )(x/y)
=
y =
y2
y2
a2
a2
4
4
b
b
=
(y 2 /b2 + x2 /a2 ) = 2 3
3
2
y a
a y

1G-4 y = (x + 1)1 , so y (1) = (x + 1)2 , y (2) = (1)(2)(x + 1)3 , and

y (3) = (1)(2)(3)(x + 1)4 .

The pattern is

y (n) = (1)n (n!)(x + 1)n1

13

27

E. Solutions to 18.01 Exercises


1G-5 a) y = u v + uv

1. Dierentiation
y = u v + 2u v + uv

b) Formulas above do coincide with Leibnizs formula for n = 1 and n = 2.


To calculate y (p+q) where y = xp (1 + x)q , useu = xp and v = (1 + x)q . The only
n (p) (q)
term in the Leibniz formula that is not 0 is
u v , since in all other terms
k
either one factor or the other is 0. If u = xp , u(p) = p!, so

y (p+q) =


n
n!
p!q! = n!
p!q! =
p
p!q!

1H. Exponentials and Logarithms: Algebra


1H-1 a) To see when y = y0 /2, we must solve the equation
1
2

= ekt .

Take ln of both sides: ln 2 = kt, from which t =

b) y1 = y0 ekt1 by assumption, =
1
e ln 2 = y1
2

y0
= y0 ekt , or
2

ln 2
.
k

ln 2
y0 ek(t1 +) = y0 ekt1 ek = y1
k

1H-2 pH = log10 [H + ]; by assumption, [H + ]dil = 12 [H + ]orig . Take log10 of


both sides (note that log 2 .3):
log [H + ]dil = log 2 log [H + ]orig = pHdil = pHorig + log2 .
1H-3 a) ln(y + 1) + ln(y 1) = 2x + ln x; exponentiating both sides and solving
for y:
(y + 1) (y 1) = e2x x = y 2 1 = xe2x = y =

xe2x + 1, since y > 0.

2
y+1
= 10x . Solve for
y1
2
y; to simplify the algebra, let A = 10x . Crossmultiplying, y + 1 = Ay A =
2
A+1
10x + 1
y=
= x2
A1
10
1

b) log(y + 1) log(y 1) = x 2 ; exponentiating,

c) 2 ln y ln(y + 1) = x; exponentiating both sides and solving for y:


14

28

1. Dierentiation

E. Solutions to 18.01 Exercises

y2
ex e2x + 4ex
= e x = y 2 ex y ex = 0 = y =
, since y 1 > 0.
y+1
2

1H-4

c
ln a
= c ln a = c ln b
a = ec ln b = eln b = bc . Similarly,
ln b
log a

= c a = bc .
log b

u2 + 1
= y; this
u2 1
1
y+1
x = ln(
)
2
y1

1H-5 a) Put u = ex (multiply top and bottom by ex rst):


gives u2 =

y+1
y+1
= e2x ; taking ln: 2x = ln(
),
y 1
y1

1
= y ; solving for u gives
b) ex + ex = y; putting u = ex gives u +
u

y y2 4
y y2 4
x
2
u yu+1 = 0 so that u =
= e ; taking ln: x = ln(
)
2
2
1H-6 A = log e ln 10 = ln(10log e ) = ln(e) = 1 ;

similarly, logb a loga b = 1

1H-7 a) If I1 is the intensity of the jet and I2 is the intensity of the conversation,
then

I1 /I0
log10 (I1 /I2 ) = log10
= log10 (I1 /I0 ) log10 (I2 /I0 ) = 13 6 = 7
I2 /I0
Therefore, I1 /I2 = 107 .
b) I = C/r2 and I = I1 when r = 50 implies

I1 = C/502 = C = I1 502 = I = I1 502 /r2

This shows that when r = 100, we have I = I1 502 /1002 = I1 /4 . It follows that
10 log10 (I/I0 ) = 10 log10 (I1 /4I0 ) = 10 log10 (I1 /I0 ) 10 log10 4 130 6.0 124
The sound at 100 meters is 124 decibels.
The sound at 1 km has 1/100 the intensity of the sound at 100 meters, because
100m/1km = 1/10.
10 log10 (1/100) = 10(2) = 20
so the decibel level is 124 20 = 104.
1I. Exponentials and Logarithms: Calculus
1I-1 a) (x + 1)ex
2
4xe2x

b) 4xe2x

c) (2x)ex
15

d) ln x

e) 2/x

f) 2(ln x)/x

g)

29

E. Solutions to 18.01 Exercises

1. Dierentiation

h) (xx ) = ex ln x = (x ln x) ex ln x = (ln x + 1)ex ln x = (1 + ln x)xx


i) (ex ex )/2

j) (ex + ex )/2

k) 1/x

l) 1/x(ln x)2

m) 2ex /(1 +

x 2

e )

(even)

1I-3 a) As n , h = 1/n 0.

1
ln(1 + h)
ln(1 + h) ln(1)
d

=

ln(1 + x)
=1
n ln(1 + ) =
h0
n
h
h
dx
x=0
Therefore,
lim n ln(1 +

1
)=1
n

b) Take the logarithm of both sides. We need to show


lim ln(1 +

1 n
) = ln e = 1
n

But

1 n
1
) = n ln(1 + )
n
n
so the limit is the same as the one in part (a).
ln(1 +

1I-4 a)

3n
n 3
1
1
e3 as n ,
1+
=
1+
n
n
b) Put m = n/2. Then

5n
10m
m 10
2
1
1
e10 as m
1+
= 1+
=
1+
m
n
m

c) Put m = 2n. Then

5n
5m/2
m 5/2
1
1
1
e5/2 as m
1+
= 1+
=
1+
m
2n
m

1J. Trigonometric functions


1J-1 a) 10x cos(5x2 )

b) 6 sin(3x) cos(3x)
16

c) 2 sin(2x)/ cos(2x) = 2 tan(2x)

30

1. Dierentiation

E. Solutions to 18.01 Exercises

d) 2 sin x/(2 cos x) = tan x. (Why did the factor 2 disappear? Because
ln(2 cos x) = ln 2 + ln(cos x), and the derivative of the constant ln 2 is zero.)

e)

x cos x sin x
x2

i)

(x2 sin x)
2x sin x + x2 cos x 2
= + cot x. Alternatively,
=
x2 sin x
x
x2 sin x

f) (1+y ) sin(x+y)

g) sin(x+y)

h) 2 sin x cos xesin

ln(x2 sin x) = ln(x2 ) + ln(sin x) = 2 ln x + ln sin x


Dierentiating gives

2 cos x
2
+
= + cot x
x
x
sin x

j) 2e2x sin(10x) + 10e2x cos(10x)

k) 6 tan(3x) sec2 (3x) = 6 sin x/ cos3 x

l) x(1 x2 )1/2 sec( 1 x2 ) tan( 1 x2 )


m) Using the chain rule repeatedly and the trigonometric double angle formu
las,
(39)

(cos2 x sin2 x) = 2 cos x sin x 2 sin x cos x = 4 cos x sin x;

(40)

(2 cos2 x) = 4 cos x sin x;

(41)

(cos(2x)) = 2 sin(2x) = 2(2 sin x cos x).

The three functions have the same derivative, so they dier by constants. And
indeed,
cos(2x) = cos2 x sin2 x = 2 cos2 x 1,

(using sin2 x = 1 cos2 x).

n)
5(sec(5x) tan(5x)) tan(5x) + 5(sec(5x)(sec2 (5x)) = 5 sec(5x)(sec2 (5x) + tan2 (5x))
Other forms:

5 sec(5x)(2 sec2 (5x) 1);

10 sec3 (5x) 5 sec(5x)

o) 0 because sec2 (3x)tan2 (3x) = 1, a constant or carry it out for practice.


p) Successive use of the chain rule:

(42)
(43)

(sin ( x2 + 1)) = cos ( x2 + 1) (x2 + 1)1/2 2x


2

x
=
cos ( x2 + 1)
x2 + 1
17

31

E. Solutions to 18.01 Exercises

1. Dierentiation

q) Chain rule several times in succession:

x
(44)
(cos2 1 x2 ) = 2 cos 1 x2 ( sin 1 x2 )
1 x2

x
sin(2 1 x2 )
(45)
=
1 x2
r) Chain rule again:

x
x
x
x+1x
(46)
tan2 (
) = 2 tan(
) sec2 (
)
x+1
x+1
x+1
(x + 1)2
2
x
x
(47)
=
tan(
) sec2 (
)
x+1
x+1
(x + 1)2
1J-2 Because cos(/2) = 0,
cos x cos(/2)
cos x
d
= lim
=
cos x|x=/2 = sin x|x=/2 = 1
x /2
dx
x /2 x /2
x/2
lim

1J-3 a) (sin(kx)) = k cos(kx). Hence


(sin(kx)) = (k cos(kx)) = k 2 sin(kx).
Similarly, dierentiating cosine twice switches from sine and then back to cosine
with only one sign change, so
(cos(kx) = k 2 cos(kx)
Therefore,
sin(kx) + k 2 sin(kx) = 0 and cos(kx) + k 2 cos(kx) = 0

Since we are assuming k > 0, k = a.


b) This follows from the linearity of the operation of dierentiation. With
k 2 = a,
(48)

(c1 sin(kx) + c2 cos(kx)) + k 2 (c1 sin(kx) + c2 cos(kx))

(49)

= c1 (sin(kx)) + c2 (cos(kx)) + k 2 c1 sin(kx) + k 2 c2 cos(kx)

(50)

= c1 [(sin(kx)) + k 2 sin(kx)] + c2 [(cos(kx)) + k 2 cos(kx)]

(51)

= c1 0 + c2 0 = 0

c) Since is a constant, d(kx + )/dx = k, and (sin(kx + ) = k cos(kx + ),


(sin(kx + ) = (k cos(kx + )) = k 2 sin(kx + )
Therefore, if a = k 2 ,
(sin(kx + ) + a sin(kx + ) = 0

d) The sum formula for the sine function says

sin(kx + ) = sin(kx) cos() + cos(kx) sin()

In other words
sin(kx + ) = c1 sin(kx) + c2 cos(kx)
18

32

1. Dierentiation

E. Solutions to 18.01 Exercises

with c1 = cos() and c2 = sin().


1J-4 a) The Pythagorean theorem implies that
c2 = sin2 + (1 cos )2 = sin2 + 1 2 cos + cos2 = 2 2 cos
Thus,

1 cos
c = 2 2 cos = 2
= 2 sin(/2)
2
b) Each angle is = 2/n, so the perimeter of the n-gon is
n sin(2/n)
As n , h = 2/n tends to 0, so
2
sin h sin 0
d
sin h = 2
2
sin x|x=0 = 2 cos x|x=0 = 2
n sin(2/n) =
h
h
dx

19

33

MIT OpenCourseWare
http://ocw.mit.edu

18.01SC Single Variable Calculus


Fall 2010

For information about citing these materials or our Terms of Use, visit: http://ocw.mit.edu/terms.

34

18.01 Exercises

December 23, 2010

Unit 2. Applications of Dierentiation


2A. Approximation

2A-1 Find the linearization of a + bx at 0, by using (2), and also by using


the basic approximation formulas. (Here a and b are constants; assume a > 0.
Do not confuse this a with the one in (2), which has the value 0.)

2A-2 Repeat Exercise A-1 for the function

2A-3 Find the linearization at 0 of


formulas, and also by using (2).

1
,
a + bx

a = 0 .

(1 + x)3/2
by using the basic approximation
1 + 2x

gW
, the
(1 + h/R)2
weight of a body at altitude h above the earths surface, where W is the surface
weight and R is the radius of the earth. (Do this without referring to the notes.)
2A-4 Find the linear approximation for h 0 for w =

2A-5 Making reasonable assumptions, if a person 5 feet tall weighs on the


average 120 lbs., approximately how much does a person 5 1 tall weigh?
2A-6 Find a quadratic approximation to tan , for 0.
sec x
2A-7 Find a quadratic approximation to
, for x 0.
1 x2
Copyright

David Jerison and MIT 1996, 2003

35

2. Applications of Dierentiation

E. 18.01 Exercises

2A-8 Find the quadratic approximation to 1/(1 x), for x 1/2 . (Either use
(13), or put x = 12 + h and use the basic approximations.)
2A-9* Derive (12) algebraically as suggested in the Notes.
2A-10 Derive (9), (10), (12) by using formula (13).
2A-11 For an ideal gas at constant temperature, the variables p (pressure) and
v (volume) are related by the equation pv k = C, where k and C are constants. If
what quadratic approximation
the volume is changed slightly from v0 to v0 + Dv,
would you use? (Find the approximation valid for
expressing p in terms of Dv
0.)
Dv
2A-12 Give the indicated type of approximation at the point indicated. (This
is to be done after studying the log and exponential functions.)
ex
ln(1 + x)
b)
(linear, x 0)
(quadratic, x 0)
xex
1 2x
c) ex (quadratic, x 0)
d) ln cos x (quadratic, x 0)
e) x ln x (quadratic, x 1) (Hint: put x = 1 + h.)
a)

2A-13 Find the linear and quadratic approximation to the following functions

a) sin(2x), near 0
b) cos(2x), near 0
c) sec(x), near 0
x2
d) e , near 0
e) 1/(a + bx), near 0; assuming a = 0;
f) 1/(a + bx), near 1; what do you have to assume about the consants a and b?

2A-14* Suppose that a piece of bubble gum has volume 4 cubic centimeters.
a) Use a linear approximation to calculate the thickness of a bubble of
inner radius 10 centimeters.
(Hint: Start with the relation between the volume V of a sphere and the radius
r, and derive the approximate relation between V and r.)
b) Find the exact answer.

36

2. Applications of Dierentiation

E. 18.01 Exercises

c) To how many signicant gures is the linear approximation accurate?


In other words, nd the order of magnitude of the dierence between the ap
proximation and the exact answer. (Be sure you use enough digits of to reect
correctly this accuracy!)
d) Use a quadratic approximation to the exact formula for the thickness
that you found in part (b) to get an even more accurate estimate.
e) Why is the quadratic term comparable to the error in the accuracy of
the linear approximation?
2A-15 Find the linear and quadratic approximations to cos(3x) near x = 0,
/6, and /3.

2A-16 a) Use the law of cosines to nd the formula 2 cos(2/n) for the
length of the side of the equilateral n-gon inscribed in the unit circle.
b) Compute the perimeter and then compute the limit as n tends to
innity using the quadratic approximation to cos near = 0. (Compare with
1J-4.)

2B. Curve Sketching


2B-1 Sketch the graphs of the following. Find the intervals on which it is
increasing and decreasing and decide how many solutions there are to y = 0.
(Graphs need not reect inection points, which are discussed in 9-2).

a) y = x3 3x + 1
c) y (x) = 1/(1 + x2 ) and y(0) = 0.
e) y = x/(x + 4)
g) y = 3x4 16x3 + 18x2 + 1
2
i) y = ex and y(0) = 0.

b) y = x4 4x + 1
d) y =
x2 /(x 1)
f) y = x + 1/(x 3)
2
h) y = ex

2B-2 Find the inection points of the graphs in problem 1.


2B-3 Find the conditions on a, b and c for which the cubic
y = x3 + ax2 + bx + c
3

37

2. Applications of Dierentiation

E. 18.01 Exercises

has a local maximum and a local minimum. Use the following two methods:
a) Find the condition under which y has two distinct real roots. Which
of these roots is at the local maximum and which is at the local minimum?
(Draw a picture.)
b) Find the condition under which y < 0 at the inection point. Why
does this property imply that there is a local maximum and a local minimum?
2B-4 Suppose that f is a continuous function on 0 x 10. Sketch the graph
from the following description: f is zero at 4, 7, and 9. f (x) > 0 on 0 < x < 5
and 8 < x < 10 and f (x) < 0 on 5 < x < 8. With the given information, can
you say anything for certain about the maximum value, the minimum value of
f ? Can you say anything about the place where the maximum is attained or
the place where the minimum is attained?
2B-5 a) Trace a copy of the graph of the function below and draw the graph
of the derivative directly underneath. Connect the inection point to the cor
responding point on the graph of the derivative with a vertical dotted line.
b) Find a rational function with a graph resembling the one below.

-7

-3

-1

2B-6 a) Find a cubic polynomial with a local maximum at x = 1 and a local


minimum at x = 1.
b) Draw the graph of the cubic on 3 x 3.

38

2. Applications of Dierentiation

E. 18.01 Exercises

c) Draw a dierentiable function on 3 x 3 that has an absolute


maximum at x = 1 and an absolute minimum at x = 1.
2B-7 a) Prove that if f (x) is increasing and it has a derivative at a, then
f (a) 0. (You may use the fact that a positive function has a limit 0.)
b) If the conclusion of part (a) is changed to : f (a) > 0, the statement
becomes false. Indicate why the proof of part (a) fails to show that f (a) > 0,
and give a counterexample to the conclusion f (a) > 0 (i.e., an example for
which it is false).
c) Prove that if f (x) has relative maximum at a and it has a derivative
Dx;
apply
at a, then f (a) = 0. (Consider the right-and left-hand limits of Dy/
the ideas of part (a).)

2C. Max-min problems


2C-1 Cut four identical squares out of the corners of a 12 by 12 inch piece of
cardboard and fold the sides so as to make a box without a top. Find the size
of the corner square that maximizes the volume of the box.
2C-2 You are asked to design a rectangular barnyard enclosing 20,000 square
feet with fencing on three sides and the wall of a long barn on the fourth. Find
the shortest length of fence needed.
2C-3 What is the largest value of the product xy, if x and y are related by
x + 2y = a, where a is a xed positive constant?
2C-4 The U. S. Postal Service accepts boxes whose length plus girth equals at
most 108 inches. What are the dimensions of the box of largest volume that is
accepted? What is its volume (in cubic feet)?
(Length is the longest of the three dimensions and girth is the sum of
the lengths of the four sides of the face perpendicular to the length, that is,
the waist-line measurement of the box. Note that the best shape for this
rectangular face perpendicular to the length must be a square.)
2C-5 Find the proportions of the open (i.e., topless) cylindrical can having the
largest volume inside, among all those having a xed surface area A. (Use as
the variable the radius r.)

39

2. Applications of Dierentiation

E. 18.01 Exercises

2C-6 What are the largest and the smallest possible values taken on by the

product of three distinct numbers spaced so the central number x has distance

1 from each of the other two, if x lies between 2 and 2 inclusive?

2C-7 Find the dimensions of the rectangle of largest area that can be inscribed

in a semicircle of radius a.

2C-8 Find the dimensions of the rectangle of largest area in a right triangle, if

a) the sides of the rectangle are parallel to the legs;

b) one side of the rectangle is parallel to the hypotenuse.

(Of the two placements, which gives the rectangle of larger area?)

2C-9 A light ray reected in a mirror travels the shortest distance between its

starting point and its endpoint.

Suppose the ray starts at (0, 1), and ends at the point (a, b) inside the rst

quadrant, and being reected when it hits the x-axis (at the point (x, 0), say).

Show that the two line segments forming its path make equal angles with the

x-axis, i.e., the angle of incidence equals the angle of reection.

2C-10 A swimmer is on the beach at a point A. The closest point on the

straight shoreline to A is called P . There is a platform in the water at B, and

the nearest pont on the shoreline to B is called Q. Suppose that the distance

from A to P is 100 meters, the distance from B to Q is 100 meters and the

distance from P to Q is a meters. Finally suppose that the swimmer can run

at 5 meters per second on the beach and swim at 2 meters per second in the

water.

B
100
P

a x

100
A

Show that the path the swimmer should take to get to the platform in the
least time has the property that the ratio of the sines of the angles the path
makes with the shoreline is the reciprocal of the ratio of the speeds in the two

40

2. Applications of Dierentiation

E. 18.01 Exercises

regions:

sin a
5
=

2
sin b
(In optics, this is known as Snells law describing the path taken by a light ray
through two successive media. Snell discovered experimentally that the above
ratio of sines was a constant, not depending on the starting point and endpoint
of the path. This problem shows that Snells law follows from a minimum
principle: the light ray takes the path minimizing its total travel time. The
ratio of sines is constant since it depends only on the speeds of light in the two
media.)
2C-11 A beam with a rectangular cross-section is cut from a log with a circular
cross-section. The strength S of the beam is proportional to the horizontal
dimension x of the rectangle and to the cube of the vertical dimension y of the
rectangle, S = cxy 3 . Find the ratio y/x which gives the strongest beam.
2C-12 You are going to mount a light on the wall behind your desk. The
light at S illuminates a point P on the horizontal surface of the desk with an
intensity inversely proportional to the square of the distance from P to S and
proportional to sine of the angle between the ray from P to S and the horizontal
surface. Fix a point P on the desk 1 foot from the wall. Find the height of S
above the desk for which the intensity at P is largest.
S

2C-13 a) An airline will ll 100 seats of its aircraft at a fare of $200. For every
$5 increase in the fare, the plane loses two passengers. For every decrease of $5,
the company gains two passengers. What price maximizes revenue?
b) A utility company has a small power plant that can produce x kilowatt
hours of electricity daily at a cost of 10 x/105 cents each for 0 x 8 105 .
Consumers will use 105 (10 p/2) kilowatt hours of electricity daily at a price
of p cents per kilowatt hour. What price should the utility charge to maximize
its prot?
2C-14 Find the maximum value and the location of the maximum for the
7

41

2. Applications of Dierentiation

E. 18.01 Exercises

following functions. (When a > 0, xa ln x 0 as x 0+ . This follows from


a) x2 ln(1/x), for x > 0
2G-8 or from LHospitals rule in Section 6A.)
b) x ln(2x), for x > 0
2C-15 Find the minimum of (x + 1)ex , for 0 x < .

2D. More Max-min Problems


2D-1* Consider a supersonic airplane wing with a cross-section in the shape
of a thin diamond (rhombus) in which the half-angle of the opening is and
the attack angle a. (The attack angle is the angle that the long diagonal of
the rhombus makes with the horizontal direction of motion of the plane. See
the picture.) The ratio of the lift to the drag is given by the formula (from the
Course 16 Unied Engineering notes on aerodynamics):

=
direction of plane
lift
a
= 2
drag
a + 2
a) For a given xed , nd the best attack angle a, that is, the one the
maximizes the ratio of lift to drag.
b) Find the minimum (largest negative) ratio. (This attack angle could
be used in the design of a winged car attempting to break the sound barrier, to
prevent it from ying.)
2D-2* Consider a paper cup in the shape of a cone obtained by rotating the
line segment y = ax, 0 x r, around the y axis. For which slope a 0 will
the paper cup hold the most water, assuming its surface area A is held
xed.
(Use the formulas: volume V = 13 (area of the base)(height); A = r2 1 + a2 .)
2D-3 Coee in a cup at a temperature y(t0 ) at time t0 in a room at tem
perature a cools according to the formula (derived in 3F-4); assume a = 20 C
and c = 1/10:
y(t) = (y(t0 ) a)ec(tt0 ) + a, t t0
You are going to add milk so that the cup has 10% milk and 90% coee. If
the coee has temperature T1 and the milk T2 , the temperature of the mixture
8

42

2. Applications of Dierentiation

will be

9
10 T1

E. 18.01 Exercises

1
10 T2 .

The coee temperature is 100 C at time t = 0, and you will drink the mixture
at time t = 10. The milk is refrigerated at 5 C. What is the best moment to
add the milk so that the coee will be hottest when you drink it?
2D-4* a) Show that the shortest collection of roads joining four towns at four
corners of a unit square is given by roads that meet at 120 angles. Use the
variable x as indicated on the picture.
b) Find the shortest collection of roads in the shape indicated for towns
at the four corners of a rectangle. Write down the formula for the length of the
roads as a function of a. Hint: Sometimes the answer is that the roads meet at
120 angles, but only for certain values of a.
1

x
a

10b

10a

2D-5* Find the triangle of smallest area in the half-plane to the right of the
y-axis whose three sides are respectively segments of the x-axis, the diagonal
y = x, and a line through (2, 1).
2D-6* Find the point of the ellipse
73x2 72xy + 52y 2 = 2500
closest to (0, 0).
Hint: Use implicit dierentiation to nd a quadratic equation in x and y.
This is the type of equation (known as a homogeneous equation) that you faced
in Problem 2C-11. The homogeneous form makes it possible to solve for y/x.
You can then nd (x, y) by substituting in the original equation.
2D-7* Find two positive numbers whose product is 10 and whose sum is as
large as possible.
9

43

2. Applications of Dierentiation

E. 18.01 Exercises

2E. Related Rates


2E-1 A robot going 20 ft/sec passes under a street light that is 30 feet above
the ground. If the robot is 5 feet tall, how fast is the tip of its shadow moving
two seconds after passing under the street light? How fast is the length of the
shadow increasing at that moment?
2E-2 A beacon light 4 miles oshore (measured perpendicularly from a straight
shoreline) is rotating at 3 revolutions per minute. How fast is the spot of light on
the shoreline moving when the beam makes an angle of 60 with the shoreline?
2E-3 Two boats are travelling at 30 miles/hr, the rst going north and the
second going east. The second crosses the path of the rst 10 minutes after the
rst one was there. At what rate is their distance increasing when the second
has gone 10 miles beyond the crossing point?
2E-4 Sand is pouring on a conical pile at a rate of 12 m3 per minute, in such
a way that the diameter of the base of the pile is always 3/2 the height. Find
the rate at which the height is increasing when the pile is 2 m tall.
2E-5 A person walks away from a pulley pulling a rope slung over it. The rope
is being held at a height 10 feet below the pulley. Suppose that the weight at
the opposite end of the rope is rising at 4 feet per second. At what rate is the
person walking when s/he is 20 feet from being directly under the pulley?
2E-6 An airplane passes directly over a boat at a height of 2 miles. The plane
is going north at 400 mph and does not change its altitude. The boat is going
west at 50 mph. How rapidly is their distance from each other increasing after
one hour?
2E-7 A trough is lled with water at a rate of 1 cubic meter per second. The
trough has a trapezoidal cross section with the lower base of length half a meter
and one meter sides opening outwards at an angle of 45 from the base. The
length of the trough is 4 meters. What is the rate at which the water level h is
rising when h is one half meter?
2E-8 One ship is sailing east at 60km an hour and another is sailing south at
50km an hour. The slower ship crosses the path of the faster ship at noon when
the faster ship was there one hour earlier. Find the time at which the two ships
were closest to each other.
10

44

2. Applications of Dierentiation

E. 18.01 Exercises

2E-9 A girl slides down a slide in the shape of the parabola y = (x 1)2 for
0 x 1. Her vertical speed is dy/dt = y(1 y). Find her horizontal speed
dx/dt when y = 1/2.
2E-10 Oil spreads on a frying pan so that its radius is proportional to t1/2 ,
where t represents the time from the moment when the oil is poured. Find the
rate of change dT /dt of the thickness T of the oil.

2F. Locating zeros; Newtons method


2F-1 a) Graph the function y = cos x x. Show using y that there is exactly
one root to the equation cos x = x, and give upper and lower bounds on the
root.
b) Use Newtons method to nd the root to 3 decimal places.
c) Another way to nd the root of cos x = x is to use what is called
the xed point method. Starting with the value z1 = 1, press the cosine key on
your calculator until the answer stabilizes, i.e., until zn+1 = cos zn . How many
iterations do you need until the rst nine digits stabilize? Which method takes
fewer steps?
2F-2 Graph the function
y = 2x 4 +

1
(x 1)2

<x<

In particular, count how many times y vanishes. (In math jargon, vanishes
means y = 0, not y . Another name for the values of x at which y = 0 is
the zeros of y.) Give reasons, based on the sign of y , that the zeros you have
found must be there and that there cannot be any more. (For example, There
must be exactly one zero in the interval (, 1) because . . . .)
2F-3 Same problem as 2 for y = x2 + x1
2F-4 The equation x5 x = x(x2 1)(x2 + 1) = 0 has three roots, x = 0, 1, 1.
How many roots does the equation x5 x 1/2 = 0 have?
2F-5 a) Find an initial value x1 for the zero of x x3 = 0 for which Newtons
method gives an undened quantity for x2 .
11

45

2. Applications of Dierentiation

E. 18.01 Exercises

b) Find an initial value x1 for the zero of x x3 = 0 such that Newtons


method bounces back and forth between two values forever. Hint: use symmetry.
c) Find the largest interval around each of the roots of x x3 = 0 such
that Newtons method converges to that root for every initial value x1 in the
interval. Hint: Parts (a) and (b) should help.
2F-6 a) Suppose that a company manufacturing cylindrical beakers (of uniform
thickness on the bottom and the sides) is willing to use up to 10 percent more
glass than the minimum required to hold a particular volume. What proportions
are permitted? (You will need to use Newtons method.)
b) What is the connection with Problem 2C-5?
2F-7 Find the point of the curve y = cos x closest to the origin. (Minimize the
square of the distance to the origin; prove youve found all the critical points.)

2G. Mean-value Theorem


2G-1 For each of these functions, on the indicated interval, nd explicitly the
point c whose existence is predicted by the Mean-value Theorem; if there is
more than one such c, nd all of them. Use the form (1).
(a) x2 on [0, 1]
(b) ln x on [1, 2]
(c) x3 x on [2, 2]
2G-2 Using the form (2), show that
(a) sin x < x, if x > 0

(b)

1 + x < 1 + x/2

if x > 0.

2G-3 The Mass Turnpike is 121 miles long. An SUV enters at the Boston end
at noon and and emerges at the west end at 1:50. Prove that at some moment
during the trip it was speeding (i.e., over the 65 mph limit).
2G-4 A polynomial p(x) of degree n has at most n distinct real roots, but it
may have fewer for instance, x2 + 1 has no real roots at all. However, show
that if p(x) does have n distinct real roots, then p (x) has n 1 distinct real
roots.
2G-5 a) Suppose f (x) exists on an interval I and f (x) has a zero at three
distinct points a < b < c on I. Show there is a point p on [a, c] where f (p) = 0.
12

46

2. Applications of Dierentiation

E. 18.01 Exercises

b) Illustrate part (a) on the cubic f (x) = (x a)(x b)(x c).


2G-6 Using the form (2) of the Mean-value Theorem, prove that on an interval
[a, b],
a) f (x) > 0
constant.

(b) f (x) = 0

f (x) increasing;

f (x)

2G-7* In what follows, use the following consequence of the Mean-value The
orem:
if f (a) g(a) and f (x) > g (x) for all x > a, then f (x) > g(x) for all x > a.
a) Starting with ex > 0, show that for all x > 0, we have ex > 1.
b) Use the same method as in part (a) to show that ex > 1 + x, and
then from this deduce that ex > 1 + x + x2 /2.
Remark This process can be continued. In the limit it leads to the innite
series which represents ex :
ex = 1 + x +

x2
x3
+
+
2!
3!

This innite series can be used to dene ex , and it is a good way to compute it
to high accuracy.
c) Show that for each number n > 0
nx1/n < ln x

for x > 1

2G-8* An analogous principle to the one in 2G-7 can be used if x < a:


if f (a) g(a) and f (x) < g (x) for all x < a, then f (x) > g(x) for all x < a.
(Why did the inequalities get reversed when x < a? Draw a graph to see.)
Use this principle with f (x) = ln x and a = 1 to show that

ln x > nx1/n

for 0 < x < 1, n > 0.

13

47

MIT OpenCourseWare
http://ocw.mit.edu

18.01SC Single Variable Calculus


Fall 2010

For information about citing these materials or our Terms of Use, visit: http://ocw.mit.edu/terms.

48

SOLUTIONS TO 18.01 EXERCISES

2. Applications of Dierentiation
2A. Approximation

2A-1

d
b
b
a + bx =
f (x) a + x by formula.
dx
2 a
2 a + bx

By algebra:

2A-2 D(

a + bx = a

1+

bx
bx
a(1 + ), same as above.
a
2a

1
b
1
b
1
1/a

)=
f (x) 2 x; OR:
=
a + bx
(a + bx)2
a + bx
1 + b/ax
a a

1
b
(1 x).
a
a

2A-3 D(

(1 + 2x)
(1 + x)3/2
)=
1 + 2x

3
2

(1 + x)3/2 (1 + x)3/2 2
1
f (0) =
2
(1 + 2x)
2

1
f (x) 1 x; OR, by algebra,
2

2A-4 Put

3
1
(1 + x)3/2
(1+ x)(12x) 1 x.
1 + 2x
2
2

2h
g
h
g(1 )2 g(1 2) = g(1
).
= ; then w =
R
R
(1 + )2

2A-5 A reasonable assumption is that w is propotional to volume v, which is in


turn proportional to the cube of a linear dimension, i.e., a given person remains
similar to him/herself, for small weight changes.) Thus w = Ch3 ; since 5 feet = 60
inches, we get
C(60 + )3

3
1
w(60 + )
=
= (1+ )3 w(60+) w(60)(1+ ) 120(1+ ) 126.
w(60)
C(60)3
60
60
20
[Or you can calculate the linearization of w(h) arround h = 60 using derivatives,
and using the value w(60) to determine C. getting w(h) 120 + 6(h 60)
COPYRIGHT DAVID JERISON AND MIT 1996, 2003
1

49

E. Solutions to 18.01 Exercises


2A-6 tan =

2. Applications of Dierentiation

sin

(1 + 2 //2)
1 2 /2
cos

1
1
1
sec x
2

2A-7
=

1 2
1 2 1 x2 1 + x
2
2
)(1

x
)
(1

x
1x
cos x 1 x
2
2

2A-8

1
1
=
=
1
1x
1 ( 2 + x)

1
2

1
2
=
1

2x
x

2(1 + 2x + 4(x)2 ) 2 + 4(x 21 ) + 8(x 21 )2


2A-10 y = (1 + x)r , y = r(1 + x)r1 , y = r(r 1)(1 + x)r2
Therefore y(0) = 1, y (0) = r, y (0) = r(r 1), giving (1 + x)r 1 + rx +
r(r 1) 2
x .
2

2A-11 pv k = c p = cv k = c((v0 + v)k = cv0 k (1 +

2A-12 a)

b)

c
v0

(1 k

v k
)
v0

v k(k + 1) v 2
+
(
) )
v0
v0
2

ex
x2
5
(1 + x + )(1 + x + x2 ) 1 + 2x + x2
1x
2
2

ln(1 + x)
x

1 x

x
x(1 + x)
xe
2

c) ex 1 x2 [Substitute into ex 1 + x]

d) ln(cos x) ln(1

x2
x2
)
[since ln(1 + h) h]
2
2

e) x ln x = (1 + h) ln(1 + h) (1 + h)(h
2

(x 1) +

(x 1)
2

h2
h2
) h+
2
2

2A-13 Finding the linear and quadratic approximation


2

x ln x

50

2. Applications of Dierentiation

E. Solutions to 18.01 Exercises

a) 2x (both linear and quadratic)


b) 1, 1 2x2
c) 1, 1 + x2 /2 (Use (1 + u)1 1 u with u = x2 /2:
sec x = 1/ cos x 1/(1 x2 /2) = (1 x2 /2)1 1 + x2 /2

d) 1, 1 + x2
e) Use (1 + u)1 1 u + u2 :
(a + bx)1 = a1 (1 + (bx/a))1 a1 (1 bx/a + (bx/a)2 )
Linear approximation: (1/a) (b/a2 )x
Quadratic approximation: (1/a) (b/a2 )x + (b2 /a3 )x2
f) f (x) = 1/(a + bx) so that f (1) = b(a + b)2 and f (1) = 2b2 /(a + b)3 .
0.
We need to assume that these numbers are dened, in other words that a + b =
Then the linear approximation is
1/(a + b) (b/(a + b)2 )(x 1)
and the quadratic approximation is
1/(a + b) (b/(a + b)2 )(x 1) + (b/(a + b)3 )(x 1)2

Method 2: Write
1/(a + bx) = 1/(a + b + b(x 1))
Then use the expansion of problem (e) with a+b in place of a and b in place of b and
0 in (e) corresponds to the restriction
(x 1) in place of x. The requirement a =
a + b = 0 in (f).
f (x) = 3 sin(3x),

2A-15 f (x) = cos(3x),


(1)

f (0) = 1,

f (/6) = cos(/2) = 0,

(2) f (0) = 3 sin 0 = 0,


(3)

f (x) = 9 cos(3x). Thus,


f (/3) = cos = 1

f (/6) = 3 sin(/2) = 3,

f (0) = 9,

f (/6) = 0,

f (/3) = 3 sin = 0

f (/3) = 9

Using these values, the linear and quadratic approximations are respectively:
(4)

for x 0 : f (x) 1 and f (x) 1 (9/2)x2

(5)

for x /6 : both are f (x) 3(x /6)

(6)

for x /3 : f (x) 1 and f (x) 1 + (9/2)(x /3)2


3

51

E. Solutions to 18.01 Exercises

2. Applications of Dierentiation

2A-16 a) The law of cosines says that for a triangle with sides a, b, and c, with
opposite the side of length c,
c2 = a2 + b2 2ab cos
Apply it to one of the n triangles with vertex at the origin: a = b = 1 and = 2/n.
So the formula is

c = 2 2 cos(2/n)

b) The perimeter is n 2 2 cos(2/n). The quadratic approximation to cos


near 0 is
cos 1 2 /2
Therefore, as n and = 2/n 0,

n 2 2 cos(2/n) n 2 2(1 (1/2)(2/n)2 ) = n (2/n)2 = n(2/n) = 2


In other words,
lim n

2 2 cos(2/n) = 2,

the circumference of the circle of radius 1.

2B. Curve Sketching


2B-1 a) y = x3 3x + 1, y = 3x2 3 = 3(x 1)(x + 1). y = 0 = x = 1.
Endpoint values: y as x , and y as x .
Critical values: y(1) = 3, y(1) = 1.
Increasing on: < x < 1, 1 < x < .
Decreasing on: 1 < x < 1.
Graph: (, ) (1, 3) (1, 1) (, ), crossing the x-axis three
times.
(-1, 3)

1a

(1,-1)

b) y = x4 4x + 1, y = x3 4. y = 0 = x = 41/3 .
4

52

2. Applications of Dierentiation

E. Solutions to 18.01 Exercises

Increasing on: 41/3 < x < ; decreasing on: < x < 41/3 .

Endpoint values: y as x ; critical value: y(41/3 ) = 1.

Graph: (, ) (41//3 , 1) (, ), never crossing the x-axis. (See below.)

c) y (x) = 1/(1 + x2 ) and y(0) = 0. By inspection, y > 0 for all x, hence


always increasing.
Endpoint values: y c as x and by symmetry y c as x . (But
it is not clear at this point in the course whether c = or some nite value. It
turns out (in Lecture 26) that y c = /2.
Graph: (, c) (, c), crossing the x-axis once (at x = 0). (See below.)

(2,4)
(4/3, 1)

(odd)

1
-4

1b

1c

1d

1e

d) y = x2 /(x 1), y = (2x(x 1) x2 )/(x 1)2 = (x2 2x)/(x 1)2 =


(x 2)x/(x 1)2 .
Endpoint values: y as x and y as x .
Singular values: y(1+ ) = + and y(1 ) = .
Critical values: y(0) = 0 and y(2) = 4.
New feature: Pay attention to sign changes in the denominator of y .
Increasing on: < x < 0 and 2 < x <
Decreasing on: 0 < x < 1 and 1 < x < 2
Graph: (, ) (0, 0) (1, ) (1, ) (2, 4) (, ), crossing
the x-axis once (at x = 0).
5

53

E. Solutions to 18.01 Exercises

2. Applications of Dierentiation

Commentary on singularities: Look out for sign changes both where y is zero and
also where y is undened: y = 0 indicates a possible sign change in the numerator
and y undened indicates a possible sign change in the denominator. In this case
there was no sign change in y at x = 1, but there would have been a sign change,
if there had been an odd power of (x 1) in the denominator. e) y = x/(x + 4),
y = ((x + 4) x)/(x + 4)2 = 4/(x + 4)2 . No critical points.
Endpoint values: y 1 as x .
Increasing on: 4 < x < .
Decreasing on: < x < 4.
Singular values: y(4+ ) = , y(4 ) = +.
Graph: (, 1) (4, ) (4, ) (, 1), crossing the x-axis once (at
x = 0).

f) y = x + 1/(x 3), y = (1/2)(x + 5)(x + 1)1/2 (x 3)2 No critical


points because x = 5 is outside of the domain of denition, x 1.
Endpoint values: y(1) = 0, and as x ,

1 + x1
1

y=
3 =0

x x

Singular values: y(3+ ) = +, y(3 ) = .

Increasing on: nowhere

Decreasing on: 1 < x < 3 and 3 < x < .

Graph: (1, 0) (3, ) (3, ) (, 0), crossing the x-axis once (at

x = 1).

g) y = 3x4 16x3 + 18x2 + 1, y = 12x3 48x2 + 36x = 12x(x 1)(x 3).


y = 0 = x = 0, 1, 3.
Endpoint values: y as x .

54

2. Applications of Dierentiation

E. Solutions to 18.01 Exercises

Critical values: y(0) = 1, y(1) = 6, and y(3) = 188.

Increasing on: 0 < x < 1 and 3 < x < .

Decreasing on: < x < 0 and 1 < x < 3.

Graph: (, ) (0, 1) (1, 6) (3, 188) (, ), crossing the x-axis

once.
(1,6)
1
-1

1g

1f

1h

1i

(3,-26)

h) y = ex , y = 2xex . y = 0 = x = 0.

Endpoint values: y 0 as x .

Critical value: y(0) = 1.


Increasing on: < x < 0
Decreasing on: 0 < x <
Graph: (, 0) (0, 1) (, 0), never crossing the x-axis. (The function is
even.)

i) y = ex and y(0) = 0. Because y is even and y(0) = 0, y is odd. No


critical points.
Endpoint values: y c as x and by symmetry y c as x . It is
not clear at this point in the course whether c is nite or innite. But we will be
able to show that c is nite when we discuss improper integrals in Unit 6.
(Using
a trick with iterated integrals, a subject in 18.02, one can show that c = /2.)
Graph: (, c) (, c), crossing the x-axis once (at x = 0).
2B-2 a) One inection point at x = 0. (y = 6x)
7

55

E. Solutions to 18.01 Exercises

2. Applications of Dierentiation

b) No inection points. y = 3x2 , so the function is convex. x = 0 is not a


point of inection because y > 0 on both sides of x = 0.
c) Inection point at x = 0. (y = 2x/(1 + x2 )2 )
d) No inection points. Reasoning: y = 2/(x 1)3 . Thus y > 0 and the
function is concave up when x > 1, and y < 0 and the function is concave down
when x < 1. But x = 1 is not called an inection point because the function is not
continuous there. In fact, x = 1 is a singular point.
e) No inection points. y = 8/(x+1)3 . As in part (d) there is a sign change
in y , but at a singular point not an inection point.

1)

f) y = (1/2)[(x + 1)(x 3) (1/2)(x + 5)(x 3) 2(x + 5)(x + 1)](x +


(x 3)3

3/2

= (1/2)[(3/2)x2 15x 11/2](x + 1)3/2 (x 3)3


Therefore there are two inection points, x = (30

768)/6, 9.6, .38.

g) y = 12(3x2 8x + 36). Therefore there are no inection points. The


quadratic equation has no real roots.
2

2 x
. Therefore there are two inection points at x =
h) y = (2 + 4x )e
1/ 2.

i) One inection point at x = 0. (y = 2xex )


2B-3 a) y = 3x2 + 2ax + b. The roots of the quadratic polynomial are distinct real
numbers if the discriminant is positive. (The discriminant is dened as the number
under the square root in the quadratic formula.) Therefore there are distinct real
roots if and only if
(2a)2 4(3)b > 0,

or

a2 3b > 0.

From the picture, since y as x and y as x , the larger


root of 3x2 + 2ax + b = 0 (with the plus sign in the quadratic formula) must be the
local min, and the smaller root must be the local max.

Since y <<-1 when x << -1


and y >> 1 when x >> 1, the
local max. is to the left of the local min.

56

2. Applications of Dierentiation

E. Solutions to 18.01 Exercises

b) y = 6x + 2a, so the inection point is at a/3. Therefore the condition


y < 0 at the inection point is

y (a/3) = 3(a/3)2 + 2a(a/3) + b = a2 /3 + b < 0,


which is the same as
a2 3b > 0.
If y < 0 at some point x0 , then the function is decreasing at that point. But
y as x , so there must be a local minimum at a point x > x0 . Similarly,
since y as x , there must be a local maximum at a point x < x0 .
Comment: We evaluate y at the inection point of y (x = a/3) since we are
trying to decide (cf. part (b)) whether y is ever negative. To do this, we nd the
minimum of y (which occurs where y = 0).

7 8

10

Max is at x = 5 or x = 10;
Min is at x = 0 or x = 8.

2B4

Graph of function

Graph of derivative; note that


local maximum point above corresponds
to zero below;
point of inflection above corresponds to
local minimum below.

2B5

2B-6 a) Try y = (x + 1)(x 1) = x2 1. Then y = x3 /3 x + c. The constant c


wont matter so set c = 0. Its also more convenient to multiply by 3:
y = x3 3x
b) This is an odd function with local min and max: y(1) = 2 and y(1) = 2.
The endpoints values are y(3) = 18 and y(3) = 18. It is very steep: y (3) = 8 .
9

57

E. Solutions to 18.01 Exercises

2. Applications of Dierentiation

c)

-3

-1

y
x0 x

2B-7 a) f (a) = lim

If y increasing, then

Therefore, lim

x0

y > 0 x > 0
y < 0 x < 0

So in both cases

y
> 0.
x

y
0.
x

y
> 0 doesnt imply
b) Proof breaks down at the last step. Namely,
x
x
lim
>0
x0 y
[Limits dont preserve strict inequalities, only weak ones. For example, u2 > 0
for u = 0, but lim u2 = 0 0, not > 0.]
u0

Counterexample: f (x) = x3 is increasing for all x, but f (0) = 0.


c) Use f (a) f (x) to show that

lim y/x 0 and

x0+

lim y/x 0.

x0

Since the left and right limits are equal, the derivative must be zero.

2C. Max-min problems

2C-1 The base of the box has sidelength 12 2x and the height is x, so the volume
is V = x(12 2x)2 .
At the endpoints x = 0 and x = 6, the volume is 0, so the maximum must occur
in between at a critical point.
V = (12 2x)2 + x(2)(12 2x)(2) = (12 2x)(12 2x 4x) = (12 2x)(12 6x).
10

58

2. Applications of Dierentiation

E. Solutions to 18.01 Exercises


x
12-2x
x

It follows that V = 0 when x = 6 or x = 2. At the endpoints x = 0 and x = 6 the


volume is 0, so the maximum occurs when x = 2.
2C-2 We want to minimize the fence length L = 2x + y, where the variables x and
y are related by xy = A = 20, 000.
y
x

Choosing x as the independent variable, we have y = A/x, so that L = 2x+A/x.


At the endpoints x = 0 and x = (its a long barn), we get L = , so the
minimumof L must occur at a critical point.
A
A
L = 2 2 ; L = 0 = x2 =
= 10, 000 = x = 100 feet
x
2
2C-3 We have y = (a x)/2, so xy = x(a x)/2. At the endpoints x = 0 and
x = a, the product xy is zero (and beyond it is negative). Therefore, the maximum
occurs at a critical point. Taking the derivative,
a 2x
d x(a x)
=
; this is 0 when x = a/2.
dx
2
2
2C-4 If the length is y and the cross-section is a square with sidelength x, then
4x + y = 108. Therxefore the volume is V = x2 y = 108x2 4x3 . Find the critical
points:
(108x2 4x3 ) = 216x 12x2 = 0 = x = 18 or x = 0.
The critical point x = 18 (3/2 ft.) corresponds to the length y = 36 (3 ft.), giving
therefore a volume of (3/2)2 (3) = 27/4 = 6.75 cubic feet.
The endpoints are x = 0, which gives zero volume, and when x = y, i.e., x = 9/5
feet, which gives a volume of (9/5)3 cubic feet, which is less than 6 cubic feet. So
the critical point gives the maximum volume.
2C-5 We let r = radius of bottom and h = height, then the volume is V = r2 h,
and the area is A = r2 + 2rh.
Using r as the independent variable, we have using the above formulas,

A r2
h=
,
2r

V = r h =

A
3
r r ;
2
2
11

dV
A 3 2
r .
=
2
dr
2

59

E. Solutions to 18.01 Exercises

2. Applications of Dierentiation

h
r

Therefore, dV /dr = 0 implies A = 3r2 , from which h =

A r2
= r.
2r

Checking the endpoints, at one h = 0 and V = 0; at the other, lim V = 0


r 0+

(using the expression above for V in terms of r); thus the critical point must occur
at a maximum.
(Another way to do this problem is to use implicit dierentiation with respect
to r. Briey, since A is xed, dA/dr = 0, and therefore
(7)
(8)

dA
r+h
;
= 2r + 2h + 2rh = 0 = h =
r
dr
dV
= 2rh + r2 h = 2rh r(r + h) = r(h r).
dr

It follows that V = 0 when r = h or r = 0, and the latter is a rejected endpoint.

2C-6 To get max and min of y = x(x + 1)(x 1) = x3 x, rst


1
y = 3x2 1 = 0 if x = ;
points:
3
2
1
1
1
1
2
2
y( ) = ( ) =
y( ) = ( ) = , rel. min.
3 3
3
3 3
3
3 3
Check endpoints:
absolute min.

y(2) = 6 2 is absolute max.;

nd the critical
2
, rel. max.
3 3

y(2) = 6 2 is

(This is an endpoint problem. The endpoints should be tested unless the physical
or geometric picture already makes clear whether the max or min occurs at an
endpoint.)

2C-7 Let r be the radius, which is


xed. Then the height a of the rectangle is in
the interval 0 a r. Since b = 2 r2 a2 , the area A is given in terms of a by

A = 2a r2 a2 .
The value of A at the endpoints a = 0 and a = r is zero, so the maximum occurs
at a critical point in between.
12

60

2. Applications of Dierentiation

E. Solutions to 18.01 Exercises

a
b

dA
2a2
2(r2 a2 ) 2a2

= 2 r2 a2
=
da
r2 a2
r 2 a2

r
2
2
Thus dA/da = 0 implies 2r = a , from which we get a = , b = r 2.
2
(We use the positive square root since a 0. Note that b = 2a and A = r2 .)
2C-8 a) Letting a and b be the two legs and x and y the sides of the rectangle,
we have y = (b/a)(x a) and the area A = xy = (b/a)x(a x). The area is zero
at the two ends x = 0 and x = a, so the maximum occurs in between at a critical
point:
A = (b/a)((a x) x); = 0 if x = a/2.
Thus y = (b/a)(a x) = b/2 and A = ab/4.
b) This time let x be the point shown on the accompanying gure; using
similar triangles, the sides of the rectangle are

b
y

x
a

1 =

x 2
b
a + b2 and 2 =
(a x)
2
a
a + b2

Therefore the area is


l
b
x

l2

a-x

A = 1 2 = (b/a)x(a x)
This is the same formula for area as in part (a), so the largest area is the same,
occurring when x = a/2, and the two maximal rectangles both have the same area;
they have dierent dimensions though, since in the present case, one side length is
half the hypotenuse:

1 = a2 + b2 /2 and 2 = ab/2 a2 + b2 .
13

61

E. Solutions to 18.01 Exercises

2. Applications of Dierentiation

2C-9 The distance is

L=

x2 + 1 +

(a x)2 + b2

The endpoint values are x , for L , so the minimum value is at a critical


point.

a-x

(a x)2 + b2 (a x) x2 + 1

=
L =

x2 + 1
x2 + 1 (a x)2 + b2
(a x)2 + b2

ax

Thus L = 0 implies (after squaring both sides),


x2 ((a x)2 + b2 ) = (a x)2 (x2 + 1),

or

x2 b2 = (a x)2

or

bx = (a x);

we used the positive square roots since both sides must be positive. Rewriting the
above,
b
1
= ,
or
tan 1 = tan 2 .
ax
x
Thus 1 = 2 : the angle of incidence equals the angle of reection.
2C-10 The total time is

1002 + (a x)2
1002 + x2
T =
+
5
2
As x , T , so the minimum value will be at a critical point.
B
100
P

a x

100
A

x
(a x)
sin sin


=
.
2
2
2
2
5
2
5 100 + x
2 100 + (a x)
Therefore, if T = 0 , it follows that
sin
sin
sin
5
=
or
= .
sin
2
5
2
T =

2C-11 Use implicit dierentiation:


14

62

2. Applications of Dierentiation

E. Solutions to 18.01 Exercises

x2 + y 2 = d2 = 2x + 2yy = 0 = y = x/y.
We want to maximize xy 3 . At the endpoints x = 0 and y = 0, the strength is zero,
so there is a maximum at a critical point. Dierentiating,

0 = (xy 3 ) = y 3 + 3xy 2 y = y 3 + 3xy 2 (x/y) = y 3 3x2 y


Dividing by x3 ,
(y/x)3 3(y/x) = 0 = (y/x)2 = 3 = y/x =

3.

2C-12 The intensity is proportional to

sin
x/ 1 + x2
y=
=
= x(1 + x2 )3/2
1 + x2
1 + x2
Endpoints: y(0) = 0 and y 0 as x , so the maximum will be at a critical
point. Critical points satisfy
S

y = (1 2x2 )(1 + x2 )5/2 = 0 = 1 2x2 = 0 = x = 1/ 2

The best height is 1/ 2 feet above the desk. (Its not worth it. Use a desk lamp.)
2C-13 a) Let p denote the price in dollars. Then there will be 100 + (2/5)(200 p)
passengers. Therefore the total revenue is
R = p(100 + (2/5)(200 p) = p(180 (2/5)p)
At the ends zero price p = 0, and no passengers p = (5/2)180 = 450, the revenue
is zero. So the maximum occurs in between at a critical point.
R = (180 (2/5)p) (2/5)p = 180 (4/5)p = 0 = p = (5/4)180 = $225
b)
P = xp x(10 x/105 ) with x = 105 (10 p/2)
Therefore, the prot is cents is
P = 105 (10 p/2)(p 10 + (10 p/2)) = 105 (10 p/2)(p/2) = (105 /4)p(20 p)
dP
= (105 /2)(10 p)
dp
15

63

E. Solutions to 18.01 Exercises

2. Applications of Dierentiation

The critical point at p = 10. This is x = 105 (10 5) = 5 105 kilowatt hours,
which is within the range available to the utility company. The function P has
second derivative 105 /2, so it is concave down and the critical point must be the
maximum. (This is one of those cases where checking the second derivative is easier
than checking the endpoints.)
Alternatively, the endpoint values are:
x = 0 = 105 (10 p/2) = 0 = p = 20 = P = 0.

(9)

x = 8 105 = 8 105 = 105 (10 p/2)

(10)

= 10 p/2 = 8 = p = 4

(11)

= P = (105 /4)4(20 4) = 16 105 cents = $160, 000

The prot at the crit. pt. was (105 /4)10(20 10) = 2.5 106 cents = $250, 000

2C-14 a) Endpoints: y = x2 ln(x) 0 as x 0+ and y as x .

Critical points: y = 2x ln x x = 0 = ln x = 1/2 = x = 1/ e.

Critical value: y(1/ e) = 1/2e.

Maximum value: 1/2e, attained when x = 1/ e. (min is not attained)


b) Endpoints: y = x ln(2x) 0 as x 0+ and y as x .
Critical points: y = ln(2x) 1 = 0 = x = 1/2e.
Critical value: y(1/2e) = (1/2e) ln(1/e) = 1/2e.
Maximum value: 1/2e, attained at x = 1/2e. (min is not attained)

2C-15 No minimum. The derivative is xex < 0, so the function decreases.


(Not needed here, but it will follows from 2G-8 or from LHospitals rule in E31
that xex 0 as x .)
16

64

2. Applications of Dierentiation

E. Solutions to 18.01 Exercises

(1//e, 1/2e)
1

14a

(1/2e, 1/2e)
1/2

14b

2D. More Max-min Problems


2D-3 The milk will be added at some time t1 , such that 0 t1 10. In the
interval 0 t < t1 the temperature is
y(t) = (100 20)e(t0)/10 + 20 = 80et/10 + 20
Therefore,
t1 /10
T1 = y(t
+ 20
1 ) = 80e

We are adding milk at a temperature T2 = 5, so the temperature as we start the


second interval of cooling is
9
1
1
T1 + T2 = 72et1 /10 + 18 +
2
10
10
Let Y (t) be the coee temperature in the interval t1 t 10. We have just
calculated Y (t1 ), so
5Y (t) = (Y (t1 ) 20)e(tt1 )/10 + 20 = (72et1 /10 1.5)e(tt1 )/10 + 20
The nal temperature is

T = Y (10) = (72et1 /10 1.5)e(10t1 )/10 + 20 = e1 72 (1.5)et1 /10 + 20


We want to maximize this temperature, so we look for critical points:
dT
= (1.5/10e)et1 /10 < 0
dt1
Therefore the function T (t1 ) is decreasing and its maximum occurs at the left
endpoint: t1 = 0.
Conclusion: The coee will be hottest if you put the milk in as soon as possible.

2E. Related Rates


17

65

E. Solutions to 18.01 Exercises

2. Applications of Dierentiation

2E-1 The distance from robot to the point on the ground directly below the street
lamp is x = 20t. Therefore, x = 20.
y
x+y
=
(similar triangles)
30
5
Therefore,

30
5
x

(x + y )/30 = y /5 = y = 4 and (x + y) = 24
The tip of the shadow is moving at 24 feet per second and the length of the shadow
is increasing at 4 feet per second.

18

66

2. Applications of Dierentiation

E. Solutions to 18.01 Exercises

2E-2

x
and d/dt = 3(2) = 6
4
with t is measured in minutes and measured in radians. The light makes an angle
of 60 with the shore when is 30 or = /6. Dierentiate with respect to t to
get
(sec2 )(d/dt) = (1/4)(dx/dt)
Since sec2 (/6) = 4/3, we get dx/dt = 32 miles per minute.
tan =

y
shoreline

2E-3 The distance is x = 10, y = 15, x = 30 and y = 30. Therefore,

(12)
(x2 + y 2 )1/2 = (1/2)(2xx + 2yy )(x2 + y 2 )1/2

(13)
= (10(30) + 15(30))/ 102 + 152

(14)
= 150/ 13miles per hour
2E-4 V = (/3)r2 h and 2r = d = (3/2)h implies h = (4/3)r.

/x 2+ y 2

y
x

Therefore,
V = (/3)r2 h = (4/9)r3
Moreover, dV /dt = 12, hence

dr
dV dr
d
dV
3
(4/9)r )
=
=
= (4/3)r2 (12) = 16r2 .
dt
dr dt
dr
dt
dV
= 36m3 /minute. 2E-5 The information is
dt

When h = 2, r = 3/2, so that

h = (4/3)r

19

67

E. Solutions to 18.01 Exercises

2. Applications of Dierentiation
z = 4

x2 + 102 = z 2 ,

We want to evaluate x at x = 20. (Derivatives are with respect to time.) Thus


2xx = 2zz and z 2 = 202 + 102 = 500
Therefore,

z
10
x
weight

x = (zz )/x = 4 500/20 = 2 5


2E-6 x = 50 and y = 400 and
z 2 = x2 + y 2 + 22
The problem is to evaluate z when x = 50 and y = 400. Thus

y
x

x+y

boat

2zz = 2xx + 2yy = z = (xx + yy )/z

and z = 502 + 4002 + 4 = 162504. So z = 162500/ 162504 403mph.


(The fact that the plane is 2 miles up rather than at sea level changes the answer
by only about 4/1000. Even the boat speed only aects the answer by about 3 miles
per hour.)
2E-7 V = 4(h2 + h/2), V = 1. To evaluate h at h = 1/2,
1 = V = 8hh + 2h = 8(1/2)h + 2h = 6h
Therefore,
h

1/2

h
h

Cross-sectional area = h 2+ h/2

20

68

2. Applications of Dierentiation

E. Solutions to 18.01 Exercises

h = 1/6 meters per second


60t, y = 50t. Noon is t = 0 and t is measured
2E-8 x = 60, y = 50 and x = 60 +
in hours. To nd the time when z = x2 + y 2 is smallest, we may as well minimize
z 2 = x2 + y 2 . We know that there will be a minimum at a critical point because
when t the distance tends to innity. Taking the derivative with respect to
t, the critical points satisfy

z
x
y

2xx + 2yy = 0
This equation says
2((60 + 60t)60 + (50t)50) = 0 = (602 + 502 )t = 602
Hence
t = 36/61 35min
The ships were closest at around 11 : 25 am.
2E-9 dy/dt = 2(x 1)dx/dt. Notice that in the range x < 1, x 1 is negative and

so (x 1) = y. Therefore,

dx/dt = (1/2(x 1))(dy/dt) = (1/2 y)(dy/dt) = +( y)(1 y)/2 = 1/4 2

Method 2: Doing this directly turns out to be faster:

x = 1 y = dx/dt = 1 (1/2)y 1/2 dy/dt


and the rest is as before.
2E-10 r = Ct1/2 . The implicit assumption is that the volume of oil is constant:
r2 T = V or r2 T = (V /) = const
Therefore, dierentiating with respect to time t,
(r2 T ) = 2rr T + r2 T = 0 = T = 2r /r
But r = (1/2)Ct1/2 , so that r /r = 1/2t. Therefore
T = 1/t
(Although we only know the rate of change of r up to a constant of proportionality,
we can compute the absolute rate of change of T .)
21

69

E. Solutions to 18.01 Exercises

2. Applications of Dierentiation

2F. Locating zeros; Newtons method


2F-1 a) y = sin x 1 0. Also, y < 0 except at a discrete list of points (where
sin x = 1). Therefore y is strictly decreasing, that is, x1 < x2 = y(x1 ) < y(x2 ).
Thus y crosses zero only once.
Upper and lower bounds for z such that y(z) = 0:
y(0) = 1 and y(/2) = /2. Therefore, 0 < z < /2.
b) xn+1 = xn (cos xn xn )/(sin xn + 1)

(15)

x1 = 1,

x2 = .750363868,

(16)

x3 = .739112891,

x4 = .739085133

Accurate to three decimals at x3 , the second step. Answer .739.


5/2

Slopes at the points


are 0 or -2.

3/2

/2
5/2

3/2

3/2

5/2

/2

3/2
5/2

c) Fixed point method takes 53 steps to stabilize at .739085133. Newtons


method takes only three steps to get to 9 digits of accuracy. (See x4 .)
2F-2
y = 2x 4 +
y = 2

1
(x 1)2

<x<

2
2((x 1)3 1)
=
(x 1)3
(x 1)3

y = 0 implies (x 1)3 = 1, which implies x 1 = 1 and hence that x = 2. The


sign changes of y are at the critical point x = 2 and at the singularity x = 1. For
x < 1, the numerator and denominator are negative, so y > 0. For 1 < x < 2, the
22

70

2. Applications of Dierentiation

E. Solutions to 18.01 Exercises

numerator is still negative, but the denominator is positive, so y < 0. For 2 < x,
both numerator and denominator are positive, so y > 0.
6
y =
(x 1)4
Therefore, y > 0 for x = 1.
Critical value: y(2) = 1

Singular values: y(1 ) = y(1+ ) =

Endpoint values: y as x and y as x .

(2,1)
1

-3

Conclusion: The function increases from to on the interval (, 1).


Therefore, the function vanishes exactly once in this interval. The function de
creases from to 1 on the interval (1, 2) and increases from 1 to on the interval
(2, ). Therefore, the function does not vanish at all in the interval (1, ). Finally,
the function is concave up on the intervals (, 1) and (1, )

23

71

E. Solutions to 18.01 Exercises

2. Applications of Dierentiation

2F-3
2x3 1
x2
Therefore y = 0 implies x3 = 1/2 or x = 21/3 . Moreover, y > 0 when x > 21/3 ,
and y < 0 when x < 21/3 and x = 0. The sign does not change across the singular
(continued )
point x = 0 because the power in the denominator is even.
y = 2x x2 =

2(x3 + 1)
x3

3
Therefore y = 0 implies x = 1, or x = 1. Keeping track of the sign change
in the denominator as well as the numerator we have that y > 0 when x > 0 and
y < 0 when 1 < x < 0. Finally, y > 0 when x < 1, and both numerator and
denominator are negative.
y = 2 + 2x3 =

Critical value: y(21/3 ) = 22/3 + 21/3 1.9


Singular value: y(0+ ) = + and y(0 ) =
Endpoint values: y as x

P
P is the critical point

Conclusions: The function decreases from to in the interval (, 0).


Therefore it vanishes exactly once in this interval. It jumps to at 0 and decreases
from to 22/3 + 21/3 in the interval (0, 21/3 ). Finally it increases from 22/3 +
21/3 to in the interval (21/3 , ). Thus it does not vanish on the interval (0, ).
The function is concave up in the intervals (, 1) and (0, ) and concave down
in the interval (1, 0), with an inection point at 1.
2F-4 From the graph, x5 x c = 0 has three roots for any small value of c. The
value of c gets too large if it exceeds the local maximum of x5 x labelled. To
calculate that local maximum, consider y = 5x4 1 = 0, with solutions x = 51/4 .
The local maximum is at x = 51/4 and the value is
(51/4 )5 (51/4 ) = 51/4 55/4 .535
Since .535 > 1/2, there are three roots.
24

72

2. Applications of Dierentiation

E. Solutions to 18.01 Exercises

P
y=c

2F-5 a) Answer: x1 = 1/ 3. f (x) = x x3 , so f (x) = 1 3x2 and


xn+1 = xn f (xn )/f (xn )

So x2 is undened if f (x1 ) = 0, that is x1 = 1/ 3.

b) Answer: x1 = 1/ 5. (This value can be found by experimentation. It


can be also be found by iterating the inverse of the Newton method function.)
Here is an explanation: Using the fact that f is odd and that x3 = x1 suggests
that x2 = x1 . This greatly simplies the equation.
xn+1 = xn (xn xn3 )/(1 3xn2 ) =

2x3n
1 3x2n

Therefore we want to nd x satisfying


2x 3
1 3x 2
2
3
2
This equation is the same
as x(1 3x ) = 2x , which implies
x = 0 or 5x = 1.In
other words,
x = 1/ 5. Now one can check that if x1 = 1/ 5, then x2 = 1/ 5,
x3 = +1/ 5, etc.
x =

c) Answers: If x1 < 1/ 3, then xn 1. If x1 > 1/ 3, then xn 1. If


1/ 5 < x1 < 1/ 5, then xn 0. This can be found experimentally, numerically.
For a complete analysis and proof one needs the methods of an upper level course
like 18.100.
2F-6 a) To simplify this problem to its essence, let V = . (We are looking for
ratio r/h and this will be the same no matter what value we pick for V .) Thus
r2 h = 1 and
A = r2 + 2/r
Minimize B = A/ instead.
B = r2 + 2r1 = B = 2r 2r2
and B = 0 implies r = 1. Endpoints: B as r 0 and as r , so we have
found the minimum at r = 1. (The constraint r2 h = 1 shows that this minimum
is achieved when r = h = 1. As a doublecheck, the fact that the minimum area is
achieved for r/h = 1 follows from 2C/5; see part (b).)
The minimum of B is 3 attained at r = 1. Ten percent more than the

25

73

E. Solutions to 18.01 Exercises

2. Applications of Dierentiation

minimum is 3.3, so we need to nd all r such that


B(r) 3.3
Use Newtons method with F (r) = B(r) 3.3. (It is unwise to start Newtons
method at r = 1. Why?) The roots of F are approximately r = 1.35 and r = .72.

B=3.3
(1,3)

allowable
r-interval

Since r2 h = 1, h = 1/r2 and the ratio,


r/h = r3
Compute (1.35)3 2.5 and (.72)3 .37. Therefore, the proportions with at most
10 percent extra glass are approximately
.37 < r/h < 2.5

b) The connection with Problem 2C-5 is that the minimum area r = h


is not entirely obvious, and not just because we are dealing with glass beakers
instead of tin cans. In 2C/5 the area is xed whereas here the volume is held xed.
But because one needs a larger surface area to hold a larger volume, maximizing
volume with xed area is the same problem as minimizing surface area with xed
volume. This is an important so-called duality principle often used in optimization
problems. In Problem 2C-5 the answer was r = h, which is the proportion with
minimum surface area as conrmed in part (a).

26

74

2. Applications of Dierentiation

E. Solutions to 18.01 Exercises

2F-7 Minimize the distance squared, x2 + y 2 . The critical points satisfy


2x + 2yy = 0
The constraint y = cos x implies y = sin x. Therefore,
0 = x + yy = x cos x sin x
There is one obvious solution x = 0. The reason why this problem is in this section
is that one needs the tools of inequalities to make sure that there are no other
solutions. Indeed, if
f (x) = x cos x sin x, then f (x) = 1 cos2 x + sin2 x = 2 sin2 x 0
Furthermore, f (x)0 is strictly positive except at the points x = k, so f is increas
ing and crosses zero exactly once.
There is only one critical point and the distance tends to innity at the endpoints
x , so this point is the minimum. The point on the graph closest to the origin
is (0, 1).
y = cos x

Alternative method: To show that (0, 1) is closest it suces to show that for
1 x < 0 and 0 < x 1,

1 x2 < cos x
2
2
Squaring gives 1 x < cos x. This can be proved using the principles of problems
6 and 7. The derivative of cos2 x (1 x2 ) is twice the function f above, so the
methods are very similar.

27

75

E. Solutions to 18.01 Exercises

2. Applications of Dierentiation

2G. Mean-value Theorem


2G-1 a) slope chord = 1; f (x) = 2x f (c) = 1 if c =

b) slope chord = ln 2; f (x) =

c) for x3 x: slope chord =

1
x

1
.
2

f (c) = ln 2 if c =

1
.
ln 2

6 (6)
f (2) f (2)
=
= 3;
2 (2)
4

f (x) = 3x2 1 f (c) = 3c2 1 = 3 c = 23


From the graph, it is clear you should get two values for c. (The axes are not drawn
to the same scale.)
(2,6)

-c

(-2,-6)

2G-2 a) f (x) = f (a) + f (c)(x a) ; Take a = 0; f () = sin x, f (x) = cos x


f (x) = 0 + cos c x sin x < x (since cos c < 1 for 0 < c < 2)
Thus the inequality is valid for 0 < x 2; since the function is periodic, it is
also valid for all x > 0.
b)

d
1
1
1
1+x=1+
x < 1 + x, since
1+x=
2
dx
2 1+x
2 1+c

c > 0.

2G-3. Let s(t) = distance; then average velocity = slope of chord =

121
= 66.
11/6

Therefore, by MVT, there is some time t = c such that s (c) = 66 > 65.
(An application of the mean-value theorem to trac enforcement...)
28

76

2. Applications of Dierentiation

E. Solutions to 18.01 Exercises

2G-4 According to Rolles Theorem (Thm.1 p.800 : an important special case of


the M.V.T, and a step in its proof), between two roots of p(x) lies at least one root
of p (x). Therefore, between the n roots a1 , . . . , an of p(x), lie at least n 1 roots
of p (x).
There are no more than n 1 roots, since degree of p (x) = n 1; thus p (x) has
exactly n 1 roots.
2G-5 Assume f (x) = 0 at a, b, c.
By Rolles theorem (as in MVT-4), there are two points q1 , q2 where f (q1 ) =
0, f (q2 ) = 0.
By Rolles theorem again, applied to q1 and q2 and f (x), there is a point p where
f (p) = 0. Since p is between q1 and q2 , it is also between a and c.

2G-6 a) Given two points xi such that a x1 < x2 b, we have


f (x2 ) = f (x1 ) + f (c)(x2 x1 ), where x1 < c < x2 .
Since f (x) > 0 on [a, b], f (c) > 0; also x2 x1 > 0. Therefore f (x2 ) > f (x1 ),
which shows f (x) is increasing .
b) We have f (x) = f (a) + f (c)(x a) where a < c < x.
Since f (c) = 0, f (x) = f (a) for a x b, which shows f (x) is constant on [a, b].

29

77

MIT OpenCourseWare
http://ocw.mit.edu

18.01SC Single Variable Calculus


Fall 2010

For information about citing these materials or our Terms of Use, visit: http://ocw.mit.edu/terms.

78

18.01 EXERCISES

Unit 3. Integration
3A. Dierentials, indenite integration
3A-1 Compute the dierentials df (x) of the
following functions.
a) d(x7 + sin 1)
b) d x
c) d(x10 8x
+ 6)
3x
e) Express dy in terms of x and dx if x + y = 1
d) d(e sin x)
3A-2 Compute
the following indenite integrals

1
4
2
dx
c)
b)
x+
8 + 9xdx
a) (2x + 3x + x + 8)dx
x

d) x3 (1 12x4 )1/8 dx
dx
f) e7x dx
e)
2

2x
8

e
dx
4 x5
dx
g)
7x e dx
i)
h)
3x
+2
x

x
x
x+5
dx. (Write
= 1 + . . ..)
dx.
k)
j)
x
x+5
x+5

dx
ln x
l)
dx
m)
x
x ln x
3A-3 Compute
the following indenite integrals.

a)
sin(5x)dx
b) sin(x) cos(x)dx
c) cos2 x sin xdx

cos x
2
d)
sec
(x/5)dx
f)
tan6 x sec2 xdx
dx
e)
3
sin x
g)
sec9 x tan xdx

3B. Denite Integrals


3B-1 Evaluate
4

a)
n2

b)

n=1
5

c)
(1)j j 2
j=1

d)

2j

j=1
4

1
n
n=1

3B-2 Find a notation expression for


a) 3 5 + 7 9 + 11 13
b) 1 + 1/4 + 1/9 + + 1/n2
c) sin x/n + sin(2x/n) + + sin((n 1)x/n) + sin x
3B-3 Write the upper, lower, left and right Riemann sums for the following
COPYRIGHT DAVID JERISON AND MIT 1996, 2003
1

79

E. 18.01 Exercises

3. Integration

integrals, using 4 equal subintervals:


3
1
3
a)
x dx
b)
x2 dx

c)

sin xdx

3B-4 Calculate the dierence between the upper and lower Riemann sums
for the following integrals with n intervals
b
b
a)
x2 dx
b)
x3 dx

Does the dierence tend to zero as n tends to innity?

3B-5 Evaluate the limit, by relating it to a Riemann sum.


sin(b/n) + sin(2b/n) + + sin((n 1)b/n) + sin(nb/n)
n
1
3B-6* Calculate
ex dx by using upper Riemann sums.
lim

Hints: The sum is a geometric progression. You will need the limit lim n(e1/n
n

1). This can be evaluated putting h = 1/n and relating the limit to the derivative
of ex at x = 0.
3B-7* Evaluate the limit
2b/n + 22b/n + + 2(n1)b/n + 2nb/n
n
n
lim

(See 3B-6.)

3C. Fundamental theorem of calculus

3C-1 Find the area under the graph of y = 1/ x 2 for 3 x 6


3C-2 Calculate
2

2

sin xdx
n
a)
3x + 5dx
b)
(3x + 5) dx
c)
3
0
0
3/4 cos x
3C-3 Calculate
2b
2
xdx
xdx
a)
b)
2+1
2 + b2
x
x
1
b

x10 dx. What area

3C-4 Calculate lim

does this integral describe? 3C-5 Find the area


a) under one arch of sin x.
b) under one arch of sin ax for a positive constant a.
3C-6 Find the area between the x-axis and
a) the curve y = x2 4b) the curve y = x2 a for a > 0.

3D. Second fundamental theorem


2

80

3. Integration

E. 18.01 Exercises
x

dt

= ln(x + x2 + a2 ) ln a, a > 0, x > 0.


2
2
t +a

3D-1 a) Prove that


0

b) For what c is
c

dt

= ln(x + x2 + a2 ) ?
t 2 + a2
x

3D-2* Show that the function y =

1 t2 dt satises the dierential equation

with side conditions:


y y = x;

y(0) = 0, y (0) = 1.

3D-3 Discuss the function F (x) =


0

1 t2
dt, including a sketch; describe
1 + t2

a) domain
b) relative maxima and minima, where increasing or decreasing, points of
inection
c) behavior as x (Hint: Evaluate the integrand as t .)
d) symmetry about y-axis or origin
3D-4 Find a function whose derivative is sin(x3 ) and whose value at 0 is
a) 0

c) nd one whose value at 1 is 1

b) 2

1
3D-5 Evaluate lim
x0 x

1+x

dt two ways:
1 + t4

a) by interpreting the integral as the area under a curve


b) by relating the limit to F (1), where F (x) =

3D-6 For dierent values of a, the functions F (x) =

dt dier from each other


a

by constants. Show this two ways:


a) directly

dt

1 + t4

81

E. 18.01 Exercises

3. Integration

b) using the corollary to the mean-value theorem quoted ((8), p.FT.5)

3D-7 Evaluate F (x) if F (x) =


x2

u sin udu
a)
0x2
c)
tan udu

sin x

b)
0

dt

1 t2

3D-8 Let f (x) be continuous. Find f (/2) if:


x/2
x
a)
f (t)dt = 2x(sin x + 1)
b)
f (t)dt = 2x(sin x + 1)
0

3E. Change of variables; Estimating integrals


x

3E-1 Prove directly from the denition L(x) =


1

dt
1
that L( ) = L(a), by
t
a

making a change of variables in the denite integral.

x
2
1

3E-2 The function dened by E(x) =


eu /2 du is used in probability
2 0
and statistics and has the same importance as sine and cosine functions have to
trigonometry.

a) Express E(x) in terms of the function of example 5 of Notes FT, F (x) =

et dt, by making a change of variable. It is known that lim F (x) = /2.


2

What is lim E(x)?


x

1
b) Evaluate lim
N
2
1
c) Express
2
a and b, where a < b.

eu

/2

eu

/2

du and

lim E(x).

du in terms of the function E and the constants

3E-3 Evaluate by making a substitution and changing both the variable and the
limits of integration.

a)
11
c)
0

ln x
dx (u = ln x)
x
dx

(x = sin u)
1 x2

sin x
dx
(2
+
cos x)3

0 19
dx

d)
(x = z + 17)
2 34x + 289

x
18
4

b)

82

3. Integration

E. 18.01 Exercises

3E-4 From the denite integral

1 x2 dx = /2 deduce the value of

a2 x2 dx

by making a suitable change of variable of the form x = ct (c constant).

3E-5 Let F (x) =

f (t)dt.
0

a) Prove that if f (t) is even, then F (x) is odd.


b) Prove that if f (t) is odd, then F (x) is even.
Hint: Make the change of variable u = t in the denite integral. (Compare
with 4-6 .)
3E-6 By comparing the given integral with an integral that is easier to evaluate.
establish each of the following estimations:
1

a)
0

dx
> 0.65
1 + x3

b)

sin2 x dx < 2

20

c)

x2 + 1 dx > 150

10

N sin x

3E-7 Show
dx < 1
1

x2

3F. Dierential equations: separation of variables


3F-1 Solve the following dierential equations
a) dy/dx = (2x + 5)4

c) dy/dx = 3/ y

b) dy/dx = (y + 1)1
d) dy/dx = xy 2

3F-2 Solve each dierential equation with the given initial condition, and evaluate
the solution at the given value of x:
a) dy/dx = 4xy,
b) dy/dx =

y(1) = 3. Find y(3).

y + 1,

y(0) = 1. Find y(3).

c) dy/dx = x2 y 1 ,

y(0) = 10. Find y(5).

d) dy/dx = 3y + 2,

y(0) = 0. Find y(8).


5

83

E. 18.01 Exercises
e) dy/dx = ey ,
y dened?

3. Integration
y(3) = 0. Find y(0). For which values of x is the solution

3F-3 a) Solve dy/dx = y 2 with y = 1 at x = 0. Evaluate y at x = 1/2, at x = 1,


and at x = 1.
b) Graph the solution and use the graph to discuss the range of validity of the
formula for y. In particular, explain why the apparent value at x = 3/2 is suspect.

3F-4 Newtons law of cooling says that the rate of change of temperature is
proportional to the temperature dierence. In symbols, if a body is at a temperature
T at time t and the surrounding region is at a constant temperature Te (e for
external), then the rate of change of T is given by
dT /dt = k(Te T ).
The constant k > 0 is a constant of proportionality that depends properties of the
body like specic heat and surface area.
a) Why is k > 0 the only physically realistic choice?
b) Find the formula for T if the initial temperature at time t = 0 is T0 .
c) Show that T Te as t .
d) Suppose that an ingot leaves the forge at a temperature of 680 Celsius in
a room at 40 Celsius. It cools to 200 in eight hours. How many hours does it take
to cool from 680 to 50 ? (It is simplest to keep track of the temperature dierence
T Te , rather than T . The temperature dierence undergoes exponential decay.)
e) Suppose that an ingot at 1000 cools to 800 in one hour and to 700 in
two hours. Find the temperature of the surrounding air.
f) Show that y(t) = T (t t0 ) also satises Newtons law of cooling for any
constant t0 . Write out the formula for T (t t0 ) and show that it is the same as
the formula in E10/17 for y(t) by identifying the constants k, Te and T0 with their
corresponding values in the displayed formula in E10/17.

3F-5* Air pressure satises the dierential equation dp/dh = (.13)p,


h is the altitude from sea level measured in kilometers.
6

where

84

3. Integration

E. 18.01 Exercises
2

a) At sea level the pressure is1 1 kg/cm . Solve the equation and nd the
pressure at the top of Mt. Everest (10 km).
b) Find the dierence in pressure between the top and bottom of the Green
Building. (Pretend its 100 meters tall starting at sea level.) Compute the numerical
value using a calculator. Then use instead the linear approximation to ex near x = 0
to estimate the percentage drop in pressure from the bottom to the top of the Green
Building.
c) Use the linear approximation p p (0)h and compute p (0) directly
from the dierential equation to nd the drop in pressure from the bottom to top
of the Green Building. Notice that this gives an answer without even knowing the
solution to the dierential equation. Compare with the approximation in part (b).
What does the linear approximation p (0)h give for the pressure at the top of Mt.
Everest?
d) What is the dierential equation for p if altitude is measured in meters
instead of kilometers?
3F-6 Let y = cos3 u 3 cos u, x = sin4 u. Find dy, dx, and dy/dx. Simplify.
3F-7 Solve:

a) y = xy, y(0) = 1

b) cos x sin y dy = sin x dx,

y(0) = 0.

3F-8 a) Find all plane curves such that the tangent line at P intersects the x-axis
1 unit to the left of the projection of P on the the x-axis.
b) Find all plane curves in the rst quadrant such that for every point P on
the curve, P bisects the part of the tangent line at P that lies in the rst quadrant.

3G. Numerical Integration


3G-1 Find approximations to the following integrals using four intervals using
Riemann sums with left endpoints, using the trapezoidal rule, and using Simpsons
rule. Also give numerical approximations to the exact values of the integrals given
to see how good these approximation methods are.

a)
01
c)
0

xdx (= 2/3.)
dx
(= /4; cf. unit 5)
1 + x2

b)

sin xdx (= 2.)


0 2

d)
1

dx
(= ln 2)
x

1using the correspondence between weight and mass on Earth of F = ma with a = 10m/sec2

85

E. 18.01 Exercises

3. Integration

3G-2 Show that the value given by Simpsons rule for two intervals for the integral
b
f (x)dx
0

gives the exact answer when f (x) = x3 . (Since a cubic polynomial is a sum of a
quadratic polynomial and a polynomial ax3 , and Simpsons rule is exact for any
quadratic polynomial, the result of this exercise implies by linearity (cf. Notes PI)
that Simpsons rule will also be exact for any cubic polynomial.)
3G-3 Use the trapezoidal rule to estimate
estimate too high or too low?

1+

2 + 3 + ... + 10, 000. Is your

3G-4 Use the trapezoidal rule to estimate the sum of the reciprocals of the rst n
integers. Is your estimate too high or too low?
b

3G-5 If the trapezoidal rule is used to estimate the value of

f (x)dx under what


a

hypotheses on f (x) will the estimate be too low? too high?

86

MIT OpenCourseWare
http://ocw.mit.edu

18.01SC Single Variable Calculus


Fall 2010

For information about citing these materials or our Terms of Use, visit: http://ocw.mit.edu/terms.

87

SOLUTIONS TO 18.01 EXERCISES

Unit 3. Integration
3A. Dierentials, indenite integration

3A-1 a) 7x6 dx. (d(sin 1) = 0 because sin 1 is a constant.)


b) (1/2)x1/2 dx
c) (10x9 8)dx
d) (3e3x sin x + e3x cos x)dx

e) (1/2 x)dx + (1/2 y)dy = 0 implies

y
1/2 xdx
1 x
1
dx = 1
dx
dy =
= dx =
1/2 y
x
x
x
3A-2 a) (2/5)x5 + x3 + x2 /2 + 8x + c
b) (2/3)x3/2 + 2x1/2 + c

c) Method 1 (slow way) Substitute: u = 8 + 9x, du = 9dx. Therefore

8 + 9xdx = u1/2 (1/9)du = (1/9)(2/3)u3/2 + c = (2/27)(8 + 9x)3/2 + c

Method 2 (guess and check): Its often faster to guess the form of the antideriv
ative and work out the constant factor afterwards:
Guess (8 + 9x)3/2 ;

d
27
(8 + 9x)3/2 = (3/2)(9)(8 + 9x)1/2 =
(8 + 9x)1/2 .
dx
2

2
So multiply the guess by
to make the derivative come out right; the answer is
27
then
2
(8 + 9x)3/2 + c
27
COPYRIGHT DAVID JERISON AND MIT 1996, 2003
1

88

E. Solutions to 18.01 Exercises

3. Integration

d) Method 1 (slow way) Use the substitution: u = 1 12x4 , du = 48x3 dx.

1
1
x3 (112x4 )1/8 dx = u1/8 (1/48)du = (8/9)u9/8 +c = (112x4 )9/8 +c
48
54

Method 2 (guess and check): guess (1 12x4 )9/8 ;


d
9
(1 12x4 )9/8 = (48x3 )(1 12x4 )1/8 = 54(1 12x4 )1/8 .
dx
8

to make the derivative come out right, getting the


So multiply the guess by
54
previous answer.
e) Method 1 (slow way): Use substitution: u = 8 2x2 , du = 4xdx.

x
12
1

dx = u1/2 (1/4)du = u3/2 + c = (8 2x2 )3/2 + c


43
6
8 2x2

Method 2 (guess and check): guess (8 2x2 )3/2 ; dierentiating it:


d
3
(8 2x2 )3/2 = (4x2 )(8 2x2 )1/2 = 6(8 2x2 )1/2 ;
dx
2

so multiply the guess by to make the derivative come out right.


6
The next four questions you should try to do (by Method 2) in your head. Write
down the correct form of the solution and correct the factor in front.
f) (1/7)e7x + c
5

g) (7/5)ex + c

h) 2e

+c

i) (1/3) ln(3x + 2) + c. For comparison, lets see how much slower substitution
is:
u = 3x + 2,

dx
=
3x + 2

du = 3dx,

so

(1/3)du
= (1/3) ln u + c = (1/3) ln(3x + 2) + c
u

j)

x+5
dx =
x


5
1+
dx = x + 5 ln x + c
x
2

89

3. Integration

E. Solutions to 18.01 Exercises

k)

x
dx =
x+5

5
x+5

dx = x 5 ln(x + 5) + c

In Unit 5 this sort of algebraic trick will be explained in detail as part of a general
method. What underlies the algebra in both (j) and (k) is the algorithm of long
division for polynomials.
l) u = ln x, du = dx/x, so

ln x
dx = udu = (1/2)u2 + c = (1/2)(ln x)2 + c
x

m) u = ln x, du = dx/x.

dx
du
=
= ln u + c = ln(ln x) + c
u
x ln x
3A-3 a) (1/5) cos(5x) + c
b) (1/2) sin2 x + c, coming from the substitution u = sin x or (1/2) cos2 x +
c, coming from the substitution u = cos x. The two functions (1/2) sin2 x and
(1/2) cos2 x are not the same. Nevertheless the two answers given are the same.
Why? (See 1J-1(m).)
c) (1/3) cos3 x + c
d) (1/2)(sin x)2 + c = (1/2) csc2 x + c
e) 5 tan(x/5) + c
f) (1/7) tan7 x + c.
g) u = sec x, du = sec x tan xdx,

sec9 x tan xdx (sec x)8 sec x tan xdx = (1/9) sec9 x + c

3B. Denite Integrals

3B-1 a) 1 + 4 + 9 + 16 = 30

b) 2 + 4 + 8 + 16 + 32 + 64 = 126

c) 1 + 4 9 + 16 25 = 15

d) 1 + 1/2 + 1/3 + 1/4 = 25/12


3

90

E. Solutions to 18.01 Exercises

3B-2 a)

(1)n+1 (2n + 1)

n=1

3. Integration

b)

1/k 2

c)

k=1

sin(kx/n)

k=1

3B-3 a) upper sum = right sum = (1/4)[(1/4)3 +(2/4)3 +(3/4)3 +(4/4)3 ] = 15/128
lower sum = left sum = (1/4)[03 + (1/4)3 + (2/4)3 + (3/4)3 ] = 7/128
b) left sum = (1)2 + 02 + 12 + 22 = 6;

right sum = 02 + 12 + 22 + 32 = 14;

upper sum = (1)2 + 12 + 22 + 32 = 15;

lower sum = 02 + 02 + 12 + 22 = 5.

c) left sum = (/2)[sin 0+sin(/2)+sin()+sin(3/2)] = (/2)[0+1+01] =


0;
right sum = (/2)[sin(/2)+sin()+sin(3/2)+sin(2)] = (/2)[1+01+0] = 0;
upper sum = (/2)[sin(/2)+sin(/2)+sin()+sin(2)] = (/2)[1+1+0+0] = ;
lower sum = (/2)[sin(0)+sin()+sin(3/2)+sin(3/2)] = (/2)[0+011] =
.
3B-4 Both x2 and x3 are increasing functions on 0 x b, so the upper sum is
the right sum and the lower sum is the left sum. The dierence between the right
and left Riemann sums is
(b/n)[f (x1 + + f (xn )] (b/n)[f (x0 + + f (xn1 )] = (b/n)[f (xn ) f (x0 )]
In both cases xn = b and x0 = 0, so the formula is
(b/n)(f (b) f (0))
a) (b/n)(b2 0) = b3 /n. Yes, this tends to zero as n .
b) (b/n)(b3 0) = b4 /n. Yes, this tends to zero as n .
3B-5 The expression is the right Riemann sum for the integral
1
sin(bx)dx = (1/b) cos(bx)|10 = (1 cos b)/b
0

so this is the limit.

3C. Fundamental theorem of calculus


4

91

3. Integration

E. Solutions to 18.01 Exercises

3C-1

(x 2)1/2 dx = 2(x 2)1/2 = 2[(4)1/2 11/2 ] = 2


3
3
2

3C-2 a) (2/3)(1/3)(3x + 5)3/2 = (2/9)(113/2 53/2 )


0

b) If n =
1, then
2
(1/(n + 1))(1/3)(3x + 5)n+1 0 = (1/3(n + 1))((11n+1 5n+1 )
If n = 1, then the answer is (1/3) ln(11/5).

c) (1/2)(cos x)2 3/4 = (1/2)[(1)2 (1/ 2)2 ] = 1/2


2
3C-3 a) (1/2) ln(x2 + 1)1 = (1/2)[ln 5 ln 2] = (1/2) ln(5/2)
2b
b) (1/2) ln(x2 + b2 )b = (1/2)[ln(5b2 ) ln(2b2 )] = (1/2) ln(5/2)
3C-4 As b ,
b
b
x10 dx = (1/9)x9 1 = (1/9)(b9 1) (1/9)(0 1) = 1/9.
1

This integral is the area of the innite region between the curve y = x10 and the
x-axis for x > 0.

3C-5 a)
0

b)
0

sin xdx = cos x|0 = (cos cos 0) = 2

/a

/a

sin(ax)dx = (1/a) cos(ax)|0

= (1/a)(cos cos 0) = 2/a

3C-6 a) x2 4 = 0 implies x = 2. So the area is


2
2
2

x3
8
(x2 4)dx =
4x = 4 2 = 16/3
(x2 4)dx = 2
3
3
2
0
0
(We changed to the interval (0, 2) and doubled the integral because x2 4 is even.)
Notice that the integral gave the wrong answer! Its negative. This is because the
graph y = x2 4 is concave up and is below the x-axis in the interval 2 < x < 2.
So the correct answer is 16/3.

b) Following part (a), x2 a = 0 implies x = a. The area is


a
a
a

x3
a3/2 4 3/2
2
2
(a x )dx = 2
(a x )dx = 2ax = 2 a3/2
= a

3 0
3
3
a
0
5

92

E. Solutions to 18.01 Exercises

3. Integration

3D. Second fundamental theorem


3D-1 Dierentiate both sides;
left side L(x): L (x) =

d
dx

dt
1

=
, by FT2;
a2 + x2
a2 + x2

d
R (x) =
(ln(x + a2 + x2 ) ln a) =
dx

right side R(x):


1

a2 + x2

1 + a2x+x2

x + a 2 + x2

Since L (x) = R (x), we have L(x) = R(x) + C for some constant C = L(x)
R(x). The constant C may be evaluated by assigning a value to x; the most
convenient choice is x = 0, which gives
0

L(0) =

= 0;

R(0) = ln(0 +

0 + a2 ) ln a = 0;

therefore C = 0 and

L(x) = R(x).

b) Put x = c; the equation becomes0 = ln(c + c2 + a2 ); solve this for c by


rst exponentiating both sides: 1 = c + c2 + a2 ; then subtract c and square both
sides; after some algebra one gets c = 12 (1 a2 ).

3D-3 Sketch y =

1 t2
rst, as shown at the right.
1 + t2
-t 2
y = 11 +
t2

y
-1

1
-1

3D-4 a)

x
3

sin(t )dt, by the FT2.

b)

x
3

sin(t )dt + 2
0

3D-5 This problem reviews the idea of the proof of the FT2.
t
a) f (t) =
1 + t4
6

c)
1

sin(t3 )dt 1

93

3. Integration

E. Solutions to 18.01 Exercises


1+x

1
x

f (t)dt =
1

shaded area
height .
width

f(t)

2/2

1 1+x

lim

x0

1
x

1+x

f (t)dt =
1

lim

x0

shaded area
1
= height = f (1) = .
width
2

b) By denition of derivative,
1+x
F (1 + x) F (1)
1
F (1) = lim
f (t)dt;
= lim
x0
x0 x 1
x
1
by FT2, F (1) = f (1) = .
2

3D-6 a) If F1 (x) =

dt, then F1 (x) = x a1 and F2 (x) =

dt and F2 (x) =
a1

a2

x a2 . Thus F1 (x) F2 (x) = a2 a1 , a constant.


b) By the FT2, F1 (x) = f (x) and F2 (x) = f (x); therefore F1 = F2 + C, for
some constant C.
3D-7 a) Using the FT2 and the chain rule, as in the Notes,
d
dx

x2

d(x2 )
u sin udu = x2 sin(x2 )
= 2x2 sin(x2 )
dx

b) =

d
c)
dx

1
2

1 sin x

sin x

cos x = 1. (So
0

dt
= x)

1 t2

x2

tan udu = tan(x2 ) 2x tan x

3D-8 a) Dierentiate both sides using FT2, and substitute x = /2: f (/2) = 4.
7

94

E. Solutions to 18.01 Exercises

3. Integration

b) Substitute x = 2u and follow the method of part (a); put u = , get nally
f (/2) = 4 4.

3E. Change of Variables; Estimating Integrals


1
3E-1 L( ) =
a

1/a

dt
1
1
. Put t = , dt = 2 du. Then
t
u
u
a
1/a
dt
u
1
dt
du
= 2 du = L( ) =
=
= L(a)
t
u
t
u
a
1
1

3E-2 a) We want t2 = u2 /2, so u = t 2, du = 2dt.


1

x/2
x/2
2
2
1
1
eu /2 du =
et 2dt =
et dt

2 0
0
0

1
1
=
= E(x) = F (x/ 2) and
lim E(x) =
x

2
2

b) The integrand is even, so


N
N
2
1
2
u2 /2

e
du =
eu /2 du = 2E(N ) 1
2 N
2 0
lim E(x) = 1/2

as N

because E(x) is odd.

b
2
1

eu /2 du = E(b)E(a) by FT1 or by interval addition Notes PI (3).


2 a
Commentary: The answer is consistent with the limit,
N
2
1

eu /2 du = E(N ) E(N ) = 2E(N ) 1 as N


2 N

dx
3E-3 a) Using u = ln x, du =
,
x

2
.
3

1
1

ln x
2

dx =
udu = u3/2 =
x
3
0
0

b) Using u = cos x, du = sin x,

sin x
dx =
(2 + cos x)3

1
du
1
1 1
1
4

=
= ( 2 2) =
..

3
2
(2 + u)
2(2 + u) 1
2 1
3
9

c) Using x = sin u, dx = cos udu,


.

dx

=
1 x2

/2
cos u

du = u
=
2
cos u
0

95

3. Integration

E. Solutions to 18.01 Exercises

3E-4 Substitute x = t/a; then x = 1 t = a. We then have


1
a
a

1
t2 dt
2
=
1 x dx =
1 2
= 2
a2 t2 dt. Multiply
a a
2
a a
1
a
ing by a2 gives the value a2 /2 for the integral, which checks, since the integral
represents the area of the semicircle.

-a

3E-5 One can use informal reasoning based on areas (as in Ex. 5, Notes FT), but
it is better to use change of variable.
a) Goal: F (x) = F (x). Let t = u, dt = du, then
x
x
F (x) =
f (t)dt =
f (u)(du)
0

0
x

Since f is even (f (u) = f (u)), F (x) =

f (u)du = F (x).
0

b) Goal: F (x) = F (x). Let t = u, dt = du, then


x
x
F (x) =
f (t)dt =
f (u)(du)
0

0
x

Since f is odd ((f (u) = f (u)), F (x) =

f (u)du = F (x).
0

1
1
>
on (0,1); therefore
3
1+x
1+x

3E-6 a) x3 < x on (0,1)

dx
>
1 + x3

1
dx

= ln(1 + x) = ln 2 = .69
1+x
0

b) 0 < sin x < 1 on (0, ) sin2 x < sin x on (0, ); therefore

sin xdx <


0

sin xdx = cos x = (1 1) = 2.


0

96

E. Solutions to 18.01 Exercises

20

c)

x2 + 1dx >

10

3. Integration
20

10

N sin x
N

3E-7
dx 1
2
1

x2 20
1
x2 dx =
= (400 100) = 150
2
2 10

| sin x|
x2 dx

N
1

1
x2 dx

= x1 = N1 + 1 < 1.
1

3F. Dierential Equations: Separation of Variables. Applications


3F-1 a) y = (1/10)(2x + 5)5 + c

dx = (1/2)(y + 1)2 = x + c. You

can leave this in implicit form or solve for y: y = 1 2x + a for any constant a
(a = 2c)
b) (y + 1)dy = dx =

(y + 1)dy =

c) y 1/2 dy = 3dx = (2/3)y 3/2 = 3x + c = y = (9x/2 + a)2/3 , with


a = (3/2)c.
d) y 2 dy = xdx = y 1 = x2 /2 + c = y = 1/(x2 /2 + c)
3F-2 a) Answer: 3e16 .
y 1 dy = 4xdx =
ln y = 2x2 + c
y(1) = 3 = ln 3 = 2 + c = c = ln 3 2.

Therefore
ln y = 2x2 + (ln 3 2)
18+ln 32

At x = 3, y = e

16

= 3e

b) Answer: y = 11/2 + 3 2.

(y + 1)1/2 dy = dx = 2(y + 1)1/2 = x + c

y(0) = 1 = 2(1 + 1)1/2 = c = c = 2 2


At x = 3,

2(y + 1)1/2 = 3 + 2 2 = y + 1 = (3/2 + 2)2 = 13/2 + 3 2

Thus, y = 11/2 + 3 2.
c) Answer: y =

550/3

ydy = x2 dx = y 2 /2 = (1/3)x3 + c
y(0) = 10 = c = 102 /2 = 50
Therefore, at x = 5,
y 2 /2 = (1/3)53 + 50 = y =
10

550/3

97

3. Integration

E. Solutions to 18.01 Exercises

d) Answer: y = (2/3)(e24 1)
(3y + 2)1 dy = dx = (1/3) ln(3y + 2) = x + c
y(0) = 0 = (1/3) ln 2 = c
Therefore, at x = 8,
(1/3) ln(3y + 2) = 8 + (1/3) ln 2 = ln(3y + 2) = 24 + ln 2 = (3y + 2) = 2e24
Therefore, y = (2e24 2)/3
e) Answer: y = ln 4 at x = 0. Dened for < x < 4.
ey dy = dx = ey = x + c
y(3) = 0 = e0 = 3 + c = c = 4
Therefore,
y = ln(4 x),
The solution y is dened only if x < 4.

y(0) = ln 4

3F-3 a) Answers: y(1/2) = 2, y(1) = 1/2, y(1) is undened.


y 2 dy = dx = y 1 = x + c
y(0) = 1 = 1 = 0 + c = c = 1
Therefore, 1/y = x 1 and
1
y=
1x
The values are y(1/2) = 2, y(1) = 1/2 and y is undened at x = 1.
b) Although the formula for y makes sense at x = 3/2, (y(3/2) = 1/(1 3/2) =
2), it is not consistent with the rate of change interpretation of the dierential
equation. The function is dened, continuous and dierentiable for < x < 1.
But at x = 1, y and dy/dx are undened. Since y = 1/(1 x) is the only solution
to the dierential equation in the interval (0, 1) that satises the initial condition
y(0) = 1, it is impossible to dene a function that has the initial condition y(0) = 1
and also satises the dierential equation in any longer interval containing x = 1.
To ask what happens to y after x = 1, say at x = 3/2, is something like asking
what happened to a rocket ship after it fell into a black hole. There is no obvious
reason why one has to choose the formula y = 1/(1 x) after the explosion.
For example, one could dene y = 1/(2 x) for 1 x < 2. In fact, any formula
y = 1/(c x) for c 1 satises the dierential equation at every point x > 1.
3F-4 a) If the surrounding air is cooler (Te T < 0), then the object will cool, so
dT /dt < 0. Thus k > 0.
b) Separate variables and integrate.
(T Te )1 dT = kdt = ln |T Te | = kt + c
11

98

E. Solutions to 18.01 Exercises

3. Integration

Exponentiating,
T Te = ec ekt = Aekt
The initial condition T (0) = T0 implies A = T0 Te . Thus
T = Te + (T0 Te )ekt
c) Since k > 0, ekt 0 as t . Therefore,

T = Te + (T0 Te )ekt Te as t

d)
T Te = (T0 Te )ekt
The data are T0 = 680, Te = 40 and T (8) = 200. Therefore,
200 40 = (680 40)e8k = e8k = 160/640 = 1/4 = 8k = ln 4.
The number of hours t that it takes to cool to 50 satises the equation
50 40 = (640)ekt = ekt = 1/64 = kt = 3 ln 4.
To solve the two equations on the right above simultaneously for t, it is easiest just
to divide the bottom equation by the top equation, which gives
t
= 3, t = 24.
8
e)
T Te = (T0 Te )ekt
The data at t = 1 and t = 2 are
800 Te = (1000 Te )ek

and

700 Te = (1000 Te )e2k

Eliminating ek from these two equations gives

700 Te
=
1000 Te

800 Te
1000 Te

(800 Te )2 = (1000 Te )(700 Te )


8002 1600Te + Te2 = (1000)(700) 1700Te + Te2
100Te = (1000)(700) 8002
Te = 7000 6400 = 600
f) To conrm the dierential equation:
y (t) = T (t t0 ) = k(Te T (t t0 )) = k(Te y(t))
The formula for y is
y(t) = T (t t0 ) = Te + (T0 Te )ek(tt0 ) = a + (y(t0 ) a)ec(tt0 )
with k = c, Te = a and T0 = T (0) = y(t0 ).
3F-6 y = cos3 u 3 cos u, x = sin4 u
12

99

3. Integration

E. Solutions to 18.01 Exercises

dy = (3 cos2 u ( sin u) + 3 sin u)du, dx = 4 sin3 u cos udu


3
dy
3 sin u(1 cos2 u)
=
=
3
4 cos u

dx
4 sin u cos u
3F-7 a) y = xy; y(0) = 1

dy
1
= xdx = ln y = x2 + c
y
2
To nd c, put x = 0, y = 1: ln 1 = 0 + c = c = 0.
2
1
= ln y = x2 =
y = ex /2
2

b) cos x sin ydy = sin xdx; y(0) = 0

sin ydy =

sin x

dx = cos y = ln(cos x) + c
cos x

Find c: put x = 0, y = 0: cos 0 = ln(cos 0) + c = c = 1


= cos y = ln(cos x) + 1

3F-8 a) From the triangle, y = slope tangent =

y
1

dy
= dx = ln y = x + c1 = y = ex+c1 = Aex (A = ec1 )
y

b) If P bisects tangent, then P0 bisects OQ (by euclidean geometry)


So P0 Q = x ( since OP0 = x).
13

100

E. Solutions to 18.01 Exercises


Slope tangent = y =

3. Integration

y
dy
dx
=
=
x
y
x

= ln y = ln x + c1

Exponentiate: y =

1 c1
c
e = ,c > 0
x
x

Ans: The hyperbolas y =

c
,c > 0
x

3G. Numerical Integration


3G-1 Left Riemann sum: (x)(y0 + y1 + y2 + y3 )
Trapezoidal rule: (x)((1/2)y0 + y1 + y2 + y3 + (1/2)y4 )
Simpsons rule: (x/3)(y0 + 4y1 + 2y2 + 4y3 + y4 )

a) x = 1/4 and

y0 = 0, y1 = 1/2, y2 = 1/ 2, y3 = 3/2, y4 = 1.

Left Riemann sum: (1/4)(0 + 1/2 + 1/ 2 + 3/2) .518

Trapezoidal rule: (1/4)((1/2) 0 + 1/2 + 1/ 2 + 3/2 + (1/2)1) .643

Simpsons rule: (1/12)(1 0 + 4(1/2) + 2(1/ 2) + 4( 3/2) + 1) .657


as compared to the exact answer .6666 . . .

b) x = /4

y0 = 0, y1 = 1/ 2, y2 = 1, y3 = 1/ 2, y4 = 0.
14

101

3. Integration

E. Solutions to 18.01 Exercises

Left Riemann sum: (/4)(0 + 1/ 2 + 1 + 1/ 2) 1.896

Trapezoidal rule: (/4)((1/2) 0 + 1/ 2 + 1 + 1/ 2 + (1/2) 0) 1.896 (same


as Riemann sum)

Simpsons rule: (/12)(1 0 + 4(1/ 2) + 2(1) + 4(1/ 2) + 1 0) 2.005


as compared to the exact answer 2

c) x = 1/4

y0 = 1, y1 = 16/17, y2 = 4/5, y3 = 16/25, y4 = 1/2.

Left Riemann sum: (1/4)(1 + 16/17 + 4/5 + 16/25) .845


Trapezoidal rule: (1/4)((1/2) 1 + 16/17 + 4/5 + 16/25 + (1/2)(1/2)) .8128
Simpsons rule: (1/12)(1 1 + 4(16/17) + 2(4/5) + 4(16/25) + 1(1/2)) .785392
as compared to the exact answer /4 .785398
(Multiplying the Simpsons rule answer by 4 gives a passable approximation to
, of 3.14157, accurate to about 2 105 .)
d) x = 1/4

y0 = 1, y1 = 4/5, y2 = 2/3, y3 = 4/7, y4 = 1/2.

Left Riemann sum: (1/4)(1 + 4/5 + 2/3 + 4/7) .76

Trapezoidal rule: (1/4)((1/2) 1 + 4/5 + 2/3 + 4/7(1/2)(1/2)) .697

Simpsons rule: (1/12)(1 1 + 4(4/5) + 2(2/3) + 4(4/7) + 1(1/2)) .69325


Compared with the exact answer ln 2 .69315, Simpsons rule is accurate to
about 104 .
15

102

E. Solutions to 18.01 Exercises

3. Integration

b4
. Using Simpsons rule with two subintervals, x =
4
0
b/2, so that we get the same answer as above:1

b
b 3 3
b4
3
3
3
S(x ) = (0 + 4(b/2) + b ) =
b = .
4
6
6 2

3G-2 We have

x3 dx =

3G-3 The sum

1 + 2 + ... + 10, 000


104

is related to the trapezoidal estimate of


xdx :
S=

y= x
1

104

(1)
0

10,000

1
1 4
1 4
0 + 1 + ... +
xdx
10 = S
10
2
2
2

But
104

104

2 3/2
2
xdx = x
= 106

3
3
0

From (1),
(2)

2
106 S 50
3

Hence
S 666, 717

(3)

In (1), we have >, as in the picture. Hence in (2), we have >, so in (3), we have
<, Too high.

3G-4 As in Problem 3 above, let


S=

1 1
1
+ + ... +
n
1 2

Then by trapezoidal rule,


1The fact that Simpsons rule is exact on cubic polynomials is very signicant to its eectiveness
as a numerical approximation. It implies that the approximation converges at a rate proportional
to the the fourth derivative of the function times (x)4 , which is fast enough for many practical
purposes.

16

103

3. Integration

E. Solutions to 18.01 Exercises

y= x

Since
1

n-1

dx
1 1 1 1
1 1
1
1

+ + + ... + = S
2 2 2 3
2 n
2 2n

1
x
dx
1
1

= ln n, we have S ln n + +
. (Estimate is too low.)
x
2 2n

3G-5 Referring to the two pictures above, one can see that if f (x) is concave down
on [a, b], the trapezoidal rule gives too low an estimate; if f (x) is concave up, the
trapezoidal rule gives too high an estimate..

17

104

MIT OpenCourseWare
http://ocw.mit.edu

18.01SC Single Variable Calculus


Fall 2010

For information about citing these materials or our Terms of Use, visit: http://ocw.mit.edu/terms.

105

18.01 EXERCISES

Unit 4. Applications of integration


4A. Areas between curves.
4A-1 Find the area between the following curves
a) y = 2x2 and y = 3x 1
c) y = x + 1/x and y = 5/2.

b) y = x3 and y = ax; assume a > 0


d) x = y 2 y and the y axis.

4A-2 Find the area under the curve y = 1 x2 in two ways.


4A-3 Find the area between the curves y = 4 x2 and y = 3x in two ways.
4A-4 Find the area between y = sin x and y = cos x from one crossing to the next.

4B. Volumes by slicing; volumes of revolution


4B-1 Find the volume of the solid of revolution generated by rotating the regions
bounded by the curves given around the x-axis.
a) y = 1 x2 , y = 0
b) y = a2 x2 , y = 0
c) y = x, y = 0, x = 1
d) y =
x, y = 0, x = a e) y = 2x x2 , y = 0
f) y = 2ax x2 , y = 0
2
2
2 2
g) y = ax, y = 0, x = ah) x /a + y /b = 1, x = 0

4B-2 Find the volume of the solid of revolution generated by rotating the regions
in 4B-1 around the y-axis.
4B-3 Show that the volume of a pyramid with a rectangular base is bh/3, where b
is the area of the base and h is the height. (Show in the process that the proportions
of the rectangle do not matter.)
4B-4 Consider (x, y, z) such that x2 + y 2 < 1, x > 0 and 0 z 5. This describes
one half of cylinder (a split log). Chop out a wedge out of the log along z = 2x.
Find the volume of the wedge.
COPYRIGHT DAVID JERISON AND MIT 1996, 2003
1

106

E. 18.01 Exercises

4. Applications of ntegration

4B-5 Find the volume of the solid obtained by revolving an equilateral triangle of
sidelength a around one of its sides.
4B-6 The base of a solid is the disk x2 + y 2 a2 . Planes perpendicular to the
xy-plane and perpendicular to the x-axis slice the solid in isoceles right triangles.
The hypotenuse of these trianglesis the segment where the plane meets the disk.
What is the volume of the solid?

4B-7 A tower is constructed with a square base and square horizontal crosssections. Viewed from any direction perpendicular to a side, the tower has base
y = 0 and prole lines y = (x1)2 and y = (x+1)2 . (See shaded region in picture.)
Find the volume of the solid.

4C. Volumes by shells

4C-1 Assume that 0 < a < b. Revolve the disk (x b)2 + y 2 a2 around the y
axis. This doughnut shape is known as a torus.
a) Set up the integral for volume using integration dx
b) Set up the integral for volume using integration dy
c) Evaluate (b).
d) (optional) Show that the (a) and (b) are the same using the substitution
z = x b.
4C-2 Find the volume of the region 0 y x2 , x 1 revolved around the y-axis.

4C-3 Find the volume of the region x y 1, x 0 revolved around the y-axis
by both the method of shells and the method of disks and washers.

4C-4 Set up the integrals for the volumes of the regions in 4B-1 by the method of
shells. (Do not evaluate.)
2

107

4. Applications of ntegration

E. 18.01 Exercises

4C-5 Set up the integrals for the volumes of the regions in 4B-2 by the method of
shells. (Do not evaluate.)
4C-6 Let 0 < a < b. Consider a ball of radius b and a cylinder of radius a whose
axis passes through the center of the ball. Find the volume of the ball with the
cylinder removed.

4D. Average value

4D-1 What is the average cross-sectional area of the solid obtained by revolving
the region bounded by x = 2, the x-axis, and the curve y = x2 about the x-axis?
(Cross-sections are taken perpendicular to the x-axis.)
4D-2 Show that the average value of 1/x over the interval [a, 2a] is of the form
C/a, where C is a constant independent of a. (Assume a > 0.)
4D-3 A point is moving along the x-axis, with distance function given by x = s(t).
Show that over a time interval [a, b], the average value of its velocity v(t) is the
same as its average velocity over this interval.
4D-4 What is the average value of the square of the distance of a point P from a
xed point Q on the unit circle, where P is chosen at random on the circle? (Use
coordinates; place Q on the x-axis.) Check your answer for reasonableness.
4D-5 If the average value of f (t) between 0 and x is given by the function g(x),
express f (x) in terms of g(x).
4D-6 An amount of money A compounded continuously at interest rate r increases
according to the law
(t = time in years)
A(t) = A0 ert
a) What is the average amount of money in the bank over the course of T years?
b) Suppose r and T are small. Give an approximate answer to part (a) by using
the quadratic approximation to your exact answer; check it for reasonableness.
4D-7 Find the average value of x2 in 0 x b.
4D-8 Find the average distance from a point on the perimeter of a square of
sidelength a to the center. Find the average of the square of the distance.
3

108

E. 18.01 Exercises

4. Applications of ntegration

4D-9 Find the average value of sin ax in its rst hump.

4D. Work
4D-1 An extremely sti spring is 12 inches long, and a force of 2,000 pounds
extends it 1/2 inch. How many foot-pounds of work would be done in stretching it
to 18 inches?
4D-2 A heavy metal 2 pound pail initially is lled with 10 pounds of paint.
Immediately after it is lled, it is pulled up at a steady rate to the top of a building
30 feet high. While being pulled, the paint leaks out through a hole in the pail at
a steady rate so that by the time it reaches the top, 1/5 of the paint has leaked
out. How many foot-pounds of work were done pulling the pail to the top of the
building?
4D-3 A heavy-duty rubber rehose hanging over the side of a building is 50 feet
long and weighs 2 lb./foot. How much work is done winding it up on a windlass on
the top of the building?
4D-4 Two point-particles having respective masses m1 and m2 are at d units
distance. How much work is required to move them n times as far apart (i.e., to
distance nd)? What is the work to move them innitely far apart?

4E. Parametric equations


4E-1 Find the rectangular equation for x = t + t2 , y = t + 2t2 .
4E-2 Find the rectangular equation for x = t + 1/t and y = t 1/t (compute x2
and y 2 ).
4E-3 Find the rectangular equation for x = 1 + sin t, y = 4 + cos t.
4E-4 Find the rectangular equation for x = tan t, y = sec t.
4E-5 Find the rectangular equation for x = sin 2t, y = cos t.
4E-6 Consider the parabola y = x2 . Find the parametrization using the slope of
the curve at a point (x, y) as the parameter.
4E-7 Find the parametrization of the circle x2 + y 2 = a2 using the slope as the
parameter. Which portion of the circle do you obtain in this way?
4

109

4. Applications of ntegration

E. 18.01 Exercises

4E-8 At noon, a snail starts at the center of an open clock face. It creeps at a
steady rate along the hour hand, reaching the end of the hand at 1:00 PM. The
hour hand is 1 meter long. Write parametric equations for the position of the snail
at time t, in some reasonable xy-coordinate system.
4E-9* a) What part of a train is moving backwards when the train moves forwards?
b) A circular disc has inner radius a and outer radius b. Its inner circle
rolls along the positive x-axis without slipping . Find parametric equations for
the motion of a point P on its outer edge, assuming P starts at (0, b). Use as
parameter. (Your equations should reduce to those of the cycloid when a = b. Do
they?)
c) Sketch the curve that P traces out.
d) Show from the parametric equations you found that P is moving backwards
whenever it lies below the x-axis.

4F. Arclength

4F-1 Find the arclength of the following curves


a) y = 5x + 2, 0 x 1.
c) y = (1 x2/3 )3/2 , 0 x 1.

b) y = x3/2 , 0 x 1.
d) y = (1/3)(2 + x2 )3/2 , 1 x 2.

4F-2 Find the length of the curve y = (ex + ex )/2 for 0 x b. Hint:
x
2
x
2
e ex
e + ex
+1=
2
2
4F-3 Express the length of the parabola y = x2 for 0 x b as an integral. (Do
not evaluate.)
4F-4 Find the length of the curve x = t2 , y = t3 for 0 t 2.
4F-5 Find an integral for the length of the curve given parametrically in Exercise
4E-2 for 1 t 2. Simplify the integrand as much as possible but do not evaluate.
4F-6 a) The cycloid given parametrically by x = tsin t, y = 1cos t describes the
path of a point on a rolling wheel. If t represents time, then the wheel is rotating
5

110

E. 18.01 Exercises

4. Applications of ntegration

at a constant speed. How fast is the point moving at each time t? When is the
forward motion (dx/dt) largest and when is it smallest?
b) Find the length of the cycloid for one turn of the wheel. (Use a half angle
formula.)
4F-7 Express the length of the ellipse x2 /a2 + y 2 /b2 = 1 using the parametrization
x = a cos t and y = b sin t. (Do not evaluate.)
4F-8 Find the length of the curve x = et cos t, y = et sin t for 0 t 10.

4G. Surface Area


4G-1 Consider the sphere of radius R formed by revolving the circle x2 + y 2 = R2
around the x-axis. Show that for R a < b R, the portion of the sphere
a x b has surface area 2R(b a). For example, the hemisphere, a = 0, b = R
has area 2R2 .
R

4G-2 Find the area of the segment of y = 1 2x in the rst quadrant revolved
around the x-axis.

4G-3 Find the area of the segment of y = 1 2x in the rst quadrant revolved

around the y-axis.

4G-4 Find an integral formula for the area of y = x2 , 0 x 4 revolved around


the x-axis. (Do not evaluate.)
4G-5 Find the area of y = x2 , 0 x 4 revolved around the y-axis.
4G-6 Find the area of the astroid x2/3 + y 2/3 = a2/3 revolved around the x-axis.
4G-7 Conside the torus of Problem E22/1.
a) Set up the integral for surface area using integration dx
6

111

4. Applications of ntegration

E. 18.01 Exercises

b) Set up the integral for surface area using integration dy


c) Evaluate (b) using the substitution y = a sin .

4H. Polar coordinate graphs


4H-1 For each of the following points given in rectangular coordinates, give its
polar coordinates. (For points below the x-axis, give two expressions for its polar
coordinates, using respectively positive and negative values for .)

a) (0, 3) b) (2, 0) c) (1,


3) d) (2, 2)
e) (1, 1)f) (0, 2) g) ( 3, 1)
h) (2, 2)

4H-2
a) Find using two dierent methods the equation in polar coordinates for the
circle of radius a with center at (a, 0) on the x-axis, as follows:
(i) write its equation in rectangular coordinates, and then change it to polar
coordinates (substitute x = r cos and y = r sin , and then simplify).
(ii) treat it as a locus problem: let OQ be the diameter lying along the x-axis,
and P : (r, ) a point on the circle; use OP Q and trigonometry to nd the relation
connecting r and .
b) Carry out the analogue of 4H-2a for the circle of radius a with center at (0, a)
on the y-axis; OQ is now the diameter lying along the y-axis.
c) (i) Find the polar equation for the line intersecting the positive x- and y-axes
respectively at A and B, and having perpendicular distance a from the origin.
(Let = DOA; use the right triangle DOP to get the equation connecting
r, , and a.
(ii) Convert your polar equation to the usual rectangular equation involving A
and B, by using trigonometry.

fig/4H-2c-eps-converted-to.pdf

d) In the accompanying gure, the point Q moves around the circle of radius a
centered at the origin; QR is a perpendicular to the x-axis. P is a point on ray OQ
7

112

E. 18.01 Exercises

4. Applications of ntegration

such that |QP | = |QR|: P is the point inside the circle in the rst two quadrants,
but outside the circle in the last two quadrants.
(i) Sketch the locus of P ; the locus is called a cardioid (cf. 4H-3c).
(ii) nd the polar equation of this locus.

a
O

e) The point P moves in a locus so that the product of its distances from the two
points Q : (a, 0) and R : (a, 0) is constant. Assuming the locus of P goes through
the origin, determine the value of the constant, and derive the polar equation of
the locus of P .
(Work with the squares of the distances, rather than the distances themselves,
and use the law of cosines; the identities (A + B)(A B) = A2 B 2 and cos 2 =
2 cos2 1 simplify the algebra and produce a simple answer at the end. The
resulting curve is a lemniscate, cf. 4H-3g.)
4H-3 For each of the following,
(i) give the corresponding equation in rectangular coordinates;
(ii) draw the graph; indicate the direction of increasing .
b) r = 2a cos
c) r = (a+b cos ) (This gure is a cardioid
a) r = sec
for a = b, a limacon with a loop for 0 < a < b, and a limacon without a loop for
a > b > 0.)
d) r = a/(b + c cos ) (Assume the constants a and b are positive. This gure is an
ellipse for b > |c| > 0, a circle for c = 0, a parabola for b = |c|, and a hyperbola for
b < |c|.)
e) r = a sin(2) (4-leaf rose)
g) r2 = a2 sin(2) (lemniscate)
i) r = ea (logarithmic spiral)

f) r = a cos(2) (4-leaf rose)


h) r2 = a2 cos(2) (lemniscate)

4I. Area and arclength in polar coordinates

4I-1 Find the arclength element ds = w()d for the curves of 4H-3.
4I-2 Find the area of one leaf of a three-leaf rose r = a cos(3).
4I-3 Find the area of the region 0 r e3 for 0
8

113

4. Applications of ntegration

E. 18.01 Exercises

4I-4 Find the area of one loop of the lemniscate r2 = a2 sin(2)


4I-5 What is the average distance of a point on a circle of radius a from a xed point
Q on the circle? (Place the circle so Q is at the origin and use polar coordinates.)
4I-6 What is the average distance from the x-axis of a point chosen at random on
the cardioid r = a(1 cos ) , if the point is chosen
a) by letting a ray = c sweep around at uniform velocity, stopping at
random and taking the point where it intersects the cardioid;
b) by letting a point P travel around the cardioid at uniform velocity, stopping
at random; (the answers to (a) and (b) are dierent...)
4I-7 Calculate the area and arclength of a circle, parameterized by x = a cos , y =
a sin .

4J. Other Applications


4J-1 Suppose it takes k units of energy to lift a cubic meter of water one meter.
About how much energy E will it take to pump dry a circular hole one meter in
diameter and 100 meters deep that is lled with water? (Give reasoning.)
4J-2 The amount x (in grams) of a radioactive material declines exponentially
over time (in minutes), according to the law x = x0 ekt , where x0 is the amount
initially present at time t = 0. If one gram of the material produces r units of
radiation/minute, about how much radiation R is produced over one hour by x0
grams of the material? (Give reasoning.)
4J-3 A very shallow circular reecting pool has uniform depth D, and radius R
(meters). A disinfecting chemical is released at its center, and after a few hours of
symmetrical diusion outwards, the concentration of chemical at a point r meters
k
from the center is
g/m3 .
1 + r2
What amount A of the chemical was released into the pool? (Give reasoning.)
4J-4 Assume a heated outdoor pool requires k units of heat/hour for each degree
F it is maintained above the external air temperature.
If the external temperature T varies between
50o and 70o over a 24 hour period

starting at midnight, according to T = 10 6cos(t/12) , how many heat units will


be required to maintain the pool at a steady 75o temperature? (Give reasoning.)
9

114

E. 18.01 Exercises

4. Applications of ntegration

4J-5 A manufacturers cost for storing one unit of inventory is c dollars/day for
space and insurance. Over the course of 30 days, production P rises from 10 to
40 units/day according to P = 10 + t. Assuming no units are sold, what is the
inventory cost for this period? (Give reasoning.)
4J-6 A water tank for a town has the shape of a sphere of radius r feet, and its
center is at a height h above the ground. If the weight of a cubic foot of water is
w lbs., how much work is required to ll the tank when empty by pumping water
from the ground? (Give reasoning using innitesimals.)
dy.
4J-6 Divide the water in the tank into thin horizontal slices of width
If the slice is at height y above the center of the tank, its radius is r2 y 2 .
volume of water in the slice = (r2 y 2 ) dy
weight of water in the slice = w(r2 y 2 ) dy
work to lift this slice from the ground = w(r2 y 2 ) dy (h + y).
r

r
r
r2 y 2
y3 y4
2
2
2
2
2
3
2
.
Total work =
w(r y )(h+y) dy = w
(r h+r yhy y ) = w r hy+
h
2
3 4 r
r
r
The even powers of y have the same value at r and r, so contribute 0 to the value;
we get

y3
r3
4
2
3
= 2wh r
= whr3 .
= wh r y
3 r
3
3

10

115

MIT OpenCourseWare
http://ocw.mit.edu

18.01SC Single Variable Calculus


Fall 2010

For information about citing these materials or our Terms of Use, visit: http://ocw.mit.edu/terms.

116

SOLUTIONS TO 18.01 EXERCISES

Unit 4. Applications of integration


4A. Areas between curves.
1

4A-1 a)

1/2

1
(3x 1 2x2 )dx = (3/2)x2 x (2/3)x3 1/2 = 1/24

b) x3 = ax = x = a or x = 0. There are two enclosed pieces (a < x < 0


and 0 < x < a) with the same area by symmetry. Thus the total area is:
a
a
2
(ax x3 )dx = ax2 (1/2)x4 0 = a2 /2
0

5/2

a3/2

(1,2)

a1/2

(1/2,1/2)

a1/2

a 3/2

1a

1c

1b

1d

c) x + 1/x = 5/2 = x2 + 1 = 5x/2 = x = 2 or 1/2. Therefore, the area


is

1/2

d)
0

2
[5/2 (x + 1/x)]dx = 5x/2 x2 /2 ln x1/2 = 15/8 2 ln 2

1
(y y 2 )dy = y 2 /2 y 3 /30 = 1/6

4A-2 First way (dx):


1
1
1
(1 x2 )dx = 2
(1 x2 )dx = 2x 2x3 /30 = 4/3
1

Second way (dy): (x = 1 y)


1
1

2 1 ydy = (4/3)(1 y)3/2 = 4/3


0

COPYRIGHT DAVID JERISON AND MIT 1996, 2003


1

117

E. Solutions to 18.01 Exercises

4. Applications of integration

y = 1-x 2
-1

x = - 1-y

x = 1-y

4A-3 4 x2 = 3x = x = 1 or 4. First way (dx):


1
1
(4 x2 3x)dx = 4x x3 /3 3x2 /24 = 125/6
4

Second way (dy): Lower section has area


3
3

(y/3 + 4 ydy = y 2 /6 (2/3)(4 y)3/2

12

12

= 117/6

4
(1,3)
x=- 4-y

x= 4-y

x= y
3
(-4,-12)

Upper section has area


4
4

2 4 ydy = (4/3)(4 y)3/2 = 4/3


3

(See picture for limits of integration.) Note that 117/6 + 4/3 = 125/6.
4A-4 sin x = cos x = x = /4 + k. So the area is
5/4

5/4
(sin x cos x)dx = ( cos x sin x)|/4 = 2 2
/4

118

4. Applications of integration

E. Solutions to 18.01 Exercises

5/4

/4

4B. Volumes by slicing; volumes of revolution


1

y 2 dx =

4B-1 a)

(1 x2 )2 dx = 2

(1 2x2 + x4 )dx

1
= 2(x 2x3 /3 + x5 /5)0 = 16/15
b)

y 2 dx =

a
a

(a2 x2 )2 dx = 2

a
0

(a4 2a2 x2 + x4 )dx

a
= 2(a4 x 2a2 x3 /3 + x5 /5)0 = 16a5 /15
1

x2 dx = /3

c)
0

x2 dx = a3 /3

d)
0

(2x x2 )2 dx =

e)
0

2
(4x2 4x3 + x4 )dx = (4x3 /3 x4 + x5 /5)0 =

16/15
(Why (e) the same as (a)? Complete the square and translate.)
1b

1d

1f

1g

1h
b

y = a2 x

(for 1a, set a = 1)

f)

2a
0

y = 2ax x2

y=x
(for 1c, set a = 1)

(2ax x2 )2 dx =

2a
0

y= ax

(for 1e, set a = 1)

(4a2 x2 4ax3 + x4 )dx


3

y 2= b 2(1 x 2/a 2)

119

E. Solutions to 18.01 Exercises

4. Applications of integration

2
= (4a2 x3 /3 ax4 + x5 /5)0 = 16a5 /15
(Why is (f) the same as (b)? Complete the square and translate.)
a

axdx = a3 /2

g)
0

y 2 dx =

h)
0

a
b2 (1 x2 /a2 )dx = b2 (x x3 /3a2 )0 = 2b2 a/3

(1 y)dy = /2

4B-2 a)
01
c)

a2

(a2 y)dy = a4 /2

b)

(1 y 2 )dy = 2/3

d)

e) x2 2x + y = 0 = x = 1

(a2 y 2 )dy = 2a3 /3

0
1 y. Using the method of washers:

[(1 +

1 y)2 (1

1 y)2 ]dy =

1 ydy

= (8/3)(1 y)3/2 = 8/3


0

(In contrast with 1(e) and 1(a), rotation around the y-axis makes the solid in
2(e) dierent from 2(a).)

a 2 2b

a,2fa2)

2d

(a,a)

2g

2h
b

x = a2 y
(for 2a, set a = 1)

x=y

x = a + a 2 y

(for 2c, set a = 1)

f) x2 2ax + y = 0 = x = a

a2

x = y2/a

x2 = a2(1 y2 /b2 )

(for 2e, set a = 1)

a2 y. Using the method of washers:

2
2
2
2
[(a + a y) (a a y) ]dy =

a2

4a

a2 ydy

= (8/3)a(a2 y)3/2 = 8a4 /3


0

120

4. Applications of integration

E. Solutions to 18.01 Exercises

g) Using washers:
a
a
(a2 (y 2 /a)2 )dy = (a2 y y 5 /5a2 )0 = 4a3 /5.
0

h)

x2 dy = 2

b
a2 (1 y 2 /b2 )dy = 2(a2 y a2 y 3 /3b2 )0 = 4a2 b/3

(The answer in 2(h) is double the answer in 1(h), with a and b reversed. Can you
see why?)
4B-3 Put the pyramid upside-down. By similar triangles, the base of the smaller
bottom pyramid has sides of length (z/h)L and (z/h)M .
The base of the big pyramid has area b = LM ; the base of the smaller pyramid
forms a cross-sectional slice, and has area

(z/h)L (z/h)M = (z/h)2 LM = (z/h)2 b


Therefore, the volume is

h
(z/h)2 bdz = bz 3 /3h2 0 = bh/3

4B-4 The slice perpendicular to the xz-plane are right triangles with base of length
x and height z = 2x. Therefore the area of a slice is x2 . The volume is
y

2x

x
x

1 x
side view of
wedge along
y-axis

x2 dy =

top view of
wedge along
z-axis

side view of
slice along
y-axis

(1 y 2 )dy = 4/3
1
1

4B-5 One side can be described by y = 3x for 0 x a/2.


Therefore, the volume is
a/2

2
2
y dx2
0

a/2

( 3x)2 dx = a3 /4

121

E. Solutions to 18.01 Exercises

4. Applications of integration
y = 3x

a/2

4B-6 If the hypotenuse of an isoceles right triangle has length


h, then its area
ish2 /4. The endpoints of the slice in the xy-plane are y = a2 x2 , so h =
2 a2 x2 . In all the volume is
a
a
2
(h /4)dx =
(a2 x2 )dx = 4a3 /3
a

4B-7 Solving for x in y = (x 1)2 and y = (x + 1)2 gives the values


a2-x 2
a
2 a2-x 2

slice

top view

x=1

and

x = 1

The hard part is deciding which sign of the square root representing the endpoints
of the square.

x = - 1+ y

x =1 - y

-1

1
2

(x- y)

Method 1: The point (0, 1) has to be on the two curves. Plug in y = 1 and x = 0

to see that the square root must have the opposite sign from 1: x = 1 y and

x = 1 + y.

Method 2: Look at the picture. x = 1 + y is the wrong choice because it is the

right half of the parabola with vertex (1, 0). We want the left half: x = 1 y.

Similarly, we want x = 1 + y, the right half of the parabola with vertex (1, 0).

Hence, the side of the square is the interval 1 + y x 1 y, whose length

is 2(1 y), and the


1
1
2

Volume =
(2(1 y) dy = 4
(1 2 y + y)dy = 2/3 .
0

122

4. Applications of integration

E. Solutions to 18.01 Exercises

4C. Volumes by shells


4C-1 a)

b+a

Shells:

b+a

4x a2 (x b)2 dx

(2x)(2y)dx =
ba

ba

(x b)2 = a2 y 2 = x = b a2 y 2
a
a

2
2
(x2 x1 )dy =
((b + a2 y 2 )2 (b a2 y 2 )2 )dy
Washers:
a
a
a
4b a2 y 2 dy
=
b)

y = a2 - (x - b) 2

x = b - a 2- y 2

b-a

x = b + a2 - y 2

b+a

b-a

-a

b+a

-a
2
y = - a 2- (x - b}

Washers

Shells

a2 y 2 dy = a2 /2, because its the area of a semicircle of radius a.

c)
a

Thus (b) = Volume of torus = 2 2 a2 b


d) z = x b, dz = dx

b+a

4x

a2 (x b)2 dx =

4(z + b) a2 z 2 dz =

ba

4b

a2 z 2 dz

because the part of the integrand with the factor z is odd, and so it integrates to 0.
1

4C-2

2x3 dx = /2

2xydx =
0

y = x2
4C2 (shells)

y= x
4C3a (shells)

x = y2
4C3b (discs)

123

E. Solutions to 18.01 Exercises


1

4C-3

4. Applications of integration

Disks:

x2 dy =

y 4 dy = /5

2y(2x)dy = 4

1 ydy

a2

a2

2y(2x)dy = 4

b)

x)dx = /5

2x(1
0

4C-4 a)

2x(1 y)dx =

Shells:

a2 ydy

2y(1 y)dy

c)
0
a

2y(a y)dy

d)
0

e) x2 2x + y = 0 = x = 1 1 y.

The interval 1 1 y x 1 + 1 y has length 2 1 y


1
1

2y(2 1 y)dy = 4
y 1 ydy
= V =
0

f) x2 2ax + y = 0 = x = a a2 y.

The interval a a2 y x a + a2 y
a2

2
2y(2 a ydy = 4
= V =
0

4b

has length 2
a2

4f

4d

x=y

4h
(a,a)

x = a + a 2- y
x = a - a2 - y

2y(a y 2 /a)dy
8

g)

a2 ydy

4g

2
(a, a )

a2

x = a 2 - y (right)
x = - a 2- y (left)

a2 y

x = y2/a

a
2
x = a 1 - y /b

124

4. Applications of integration
b

h)

2y(a2 (1 y 2 /b2 )dy

2yxdy =
0

E. Solutions to 18.01 Exercises

(Why is the lower limit of integration 0 rather than b?)


1

c)

d)

0
2

2x(2x x2 )dx

f)

5b

2x2 dx
0

2xydx =

2xydx =

e)

2x2 dx

0
a

2x(a2 x2 )dx

2xydx =

b)

2x(1 x2 )dx

4C-5 a)

2a

5d

2a

2x(ax

2xydx =
0

5f

5g

5h
a

-a

g)

y = ax

2x axdx

2xydx =

h)

y = 2ax - x 2
(for 5e, set a =1 )

y=x
for 5c, set a = 1 )

y=a-x
(for 5a, set a = 1 )

x2 )dx

2x(2b2 (1 x2 /a2 ))dx

2x(2y)dx =
0

(Why did y get doubled this time?)


4C-6
b

2x(2y)dx =

2x(2

b2 x2 )dx

= (4/3)(b2 x2 )3/2 = (4/3)(b2 a2 )3/2


a

4D. Average value


4D-1 Cross-sectional area at x is = y 2 = (x2 )2 = x4 . Therefore,
9

y = b 1 - x 2/a 2
y = -b 1 - x 2/a 2

125

E. Solutions to 18.01 Exercises

4. Applications of integration

y= b2- x 2
Shells
a

y=- b - x

base of removed
cylinder

average cross-sectional area =

1
4D-2 Average =
a

2a

1
2

x4 dx =

2
x5
16
=
.
5
10 0

2a

1
1
1
2a
ln 2
dx
= ln x = (ln 2a ln a) =
ln
=
.
a
x
a
a
a
a
a

4D-3 Let s(t) be the distance function; then the velocity is v(t) = s (t)

Average value of velocity =

1
ba

s (t)dt =

s(b) s(a)
by FT1
ba

= average velocity over time interval [a,b]


4D-4 By symmetry, we can restrict P to the upper semicircle.
By the law of cosines, we have
average of |P Q|2 =

|P Q|2 = 12 + 12 2 cos . Thus

(2 2 cos )d =
0

[2 2 sin ]0 = 2

(This is the value of |P Q|2 when = /2, so the answer is reasonable.))

P
1

1 x
f (t)dt To express f (x) in terms of g(x),
4D-5 By hypothesis, g(x) =
x 0
multiply thourgh by x and apply the Sec. Fund. Thm:
10

126

4. Applications of integration

E. Solutions to 18.01 Exercises

f (t)dt = xg(x) f (x) = g(x) + xg (x) , by FT2..

1
4D-6 Average value of A(t) =
T

A0 ert dt =

1 A0 rt T
A0 rT
(e 1)
e |0 =
T r
rT

(rT )2
, so we get
2
2
(rT )
rT
A0
) = A0 (1 +
).
A(t)
(rT +
2
2
rT

If rT is small, we can approximate: erT 1 + rT +

(If T 0, at the end of T years the interest added will be A0 rT ; thus the average
is approximately what the account grows to in T /2 years, which seems reasonable.)

4D-7

1
b

x2 dx = b2 /3

4D-8 The average on each side is the same as the average


over all four sides. Thus the average distance is

1 a/2 2
x + (a/2)2 dx
a a/2

x2+ (a/2)

a/2
x

Cant be evaluated by a formula until Unit 5. The average of the square of the
distance is

1 a/2 2
2 a/2 2
(x + (a/2)2 )dx = a2 /3
(x + (a/2)2 )dx =
a a/2
a 0
/a
/a

1
1
= 2/
4D-9
sin ax dx cos(ax)

/a 0
0

4D. Work
4D-1 According to Hookes law, we have F = kx, where F is the force, x is
the displacement (i.e., the added length), and k is the Hookes law constant for the
11

127

E. Solutions to 18.01 Exercises

4. Applications of integration

spring.
To nd k, substitute into Hookes law: 2, 000 = k (1/2) k = 4000.
To nd the work W , we have
6
6
6
W =
F dx =
4000x dx = 2000x2 0 = 72, 000 inch-pounds = 6, 000 foot-pounds.
0

4D-2 Let W (h) = weight of pail and paint at height h.


1
h, since the pulling and leakage
W (0) = 12, W (30) = 10 W (h) = 12 15
both occur at a constant rate.
30
30
30
h
h2
work =
W (h) dh =
(12 ) dh = 12h
= 330 ft-lbs.
15
30 0
0
0

4D-3 Think of the hose as divided into many equal little innitesimal pieces, of
length dh, each of which must be hauled up to the top of the building.
The piece at distance h from the top end has weight 2 dh; to haul it up to the
top requires 2h dh ft-lbs. Adding these up,
50
50
total work =
2h dh = h2 0 = 2500ft-lbs.
0

g m1 m2
.
x2

1 1
g m1 m2 n 1
=
.

= g m1 m2
nd d
d
n

4D-4 If they are x units apart, the gravitational force between them is

nd

g m1 m2
g m1 m2
dx =
work =
2
x
x
d
g m1 m2
The limit as n is
.
d

nd
d

4E. Parametric equations


4E-1 y x = t2 , y 2x = t. Therefore,
y x = (y 2x)2 = y 2 4xy + 4x2 y + x = 0

(parabola)

4E-2 x2 = t2 + 2 + 1/t2 and y 2 = t2 2 + 1/t2 . Subtract, getting the hyperbola


x2 y 2 = 4
4E-3 (x 1)2 + (y 4)2 = sin2 + cos2 t = 1 (circle)
4E-4 1 + tan2 t = sec2 t = 1 + x2 = y 2 (hyperbola)

4E-5 x = sin 2t = 2 sin t cos t = 2

1 y 2 y. This gives x2 = 4y 2 4y 4 .
12

128

4. Applications of integration

E. Solutions to 18.01 Exercises

4E-6 y = 2x, so t = 2x and


y = t2 /4

x = t/2,

4E-7 Implicit dierentiation gives 2x + 2yy = 0, so that y = x/y. So the


parameter is t = x/y. Substitute x = ty in x2 + y 2 = a2 to get
t2 y 2 + y 2 = a2 = y 2 = a2 /(1 + t2 )
Thus

a
at
, x=
y=
2
1 + t2
1+t
For < t < , this parametrization traverses the upper semicircle y > 0 (going
clockwise). One can also get the lower semicircle (also clockwise) by taking the
negative square root when solving for y,
a
at
, x=
y=
2
1 + t2
1+t
4E-8 The tip Q of the hour hand is given in terms of the angle by Q = (cos , sin )
(units are meters).
Next we express in terms of the time parameter t (hours). We have

/2, t = 0

/3, t = 1

=
2

decreases linearly with t

(t 0)
. Thus we get =
10

6 t.

Finally, for the snails position P , we have

P = (t cos , t sin ) , where t increases from 0 to 1. So,

x = t cos(

t) = t sin t,
6
6
2

y = t sin( 2 6 t) = t cos 6 t

4F. Arclength

4F-1 a) ds =

1+

(y )2 dx

= 26dx. Arclength =
13

26dx = 26.

129

E. Solutions to 18.01 Exercises

4. Applications of integration

b) ds = 1 + (y )2 dx = 1 + (9/4)xdx.
1
1

Arclength =
1 + (9/4)xdx = (8/27)(1 + 9x/4)3/2 = (8/27)((13/4)3/2 1)
0

c) y = x1/3 (1 x2/3 )1/2 = x2/3 1. Therefore, ds = x1/3 dx, and


1
1

Arclength =
x1/3 dx = (3/2)x2/3 = 3/2
0

d) y = x(2 + x2 )1/2 . Therefore, ds = 1 + 2x2 + x4 dx = (1 + x2 )dx and


2
2
Arclength =
(1 + x2 )dx = x + x3 /31 = 10/3
1

4F-2 y = (e e
ydx. Thus,

Arclength = (1/2)
0

4F-3 y = 2x,

1+

1 + (y )2 dx =

)/2, so the hint says 1 + (y )2 = y 2 and ds =

(y )2

b
(ex + ex )dx = (1/2)(ex ex )0 = (eb eb )/2

= 1 + 4x2 . Hence, arclength =

1 + 4x2 dx.

4F-4 ds =

(dx/dt)2 + (dy/dt)2 dt = 4t2 + 9t4 dt. Therefore,


2
2

2
4
Arclength =
4t + 9t dt =
(4 + 9t2 )1/2 tdt

2
2 3/2

= (1/27)(4 + 9t )

= (403/2 8)/27
0

4F-5 dx/dt = 1 1/t , dy/dt = 1 + 1/t . Thus

ds = (dx/dt)2 + (dy/dt)2 dt = 2 + 2/t4 dt


2
Arclength =
2 + 2/t4 dt

and

4F-6 a) dx/dt = 1 cos t, dy/dt = sin t.

ds/dt = (dx/dt)2 + (dy/dt)2 = 2 2 cos t (speed of the point)


Forward motion (dx/dt) is largest for t an odd multiple of (cos t = 1). Forward
motion is smallest for t an even multiple of (cos t = 1).
(continued )
Remark: The largest forward motion is when the point is at the top of the wheel
and the smallest is when the point is at the bottom (since y = 1 cos t.)

b)
0

2 2 cos tdt =

2 sin(t/2)dt = 4 cos(t/2)|0 = 8
14

130

4. Applications of integration
2

E. Solutions to 18.01 Exercises

a2 sin2 t + b2 cos2 tdt

4F-7
0

4F-8 dx/dt = et (cos t sin t), dy/dt = et (cos t + sin t).

ds = e2t (cos t sin t)2 + e2t (cos t + sin t)2 dt = et 2 cos2 t + 2 sin2 tdt = 2et dt
Therefore, the arclength is
10

2e dt = 2(e10 1)

4G. Surface Area


4G-1 The curve y =

R2 x2 for a x b is revolved around the x-axis.

Since we have y = x/ R2 x2 , we get


y= R2- x 2

ds =

1 + (y )2 dx =

1 + x2 /(R2 x2 )dx = R2 /(R2 x2 )dx = (R/y)dx

Therefore, the area element is


dA = 2yds = 2Rdx
and the area is

2Rdx = 2R(b a)

4G-2 Limits are 0 x 1/2. ds = 5dx, so


dA = 2yds = 2(1 2x) 5dx = A = 2 5
a

1/2

(1 2x)dx =

5/2

4G-3 Limits are 0 y 1. x = (1 y)/2, dx/dy = 1/2. Thus


ds =

1 + (dx/dy)2 dy =

5/4dy;

1
dA = 2yds = (1 y)( 5/2)dx = A = ( 5/2) 0 (1 y)dy = 5/4
15

131

E. Solutions to 18.01 Exercises

4. Applications of integration
4G3

y = 1 2x
x = (1 y)/2

4G2

1/2

4G-4 A =

2x2

2yds =

1 + 4x2 dx

4G-5 x =

y, dx/dy = 1/2 y, and ds = 1 + 1/4ydy

2

2 y 1 + 1/4ydy
A = 2xds =
0

2 y + 1/4dy

=
0

= (4/3)(y + 1/4)3/2 = (4/3)((9/4)3/2 (1/4)3/2 )


0

= 13/3
4G-6 y = (a2/3 x2/3 )3/2 = y = x1/3 (a2/3 x2/3 )1/2 . Hence

ds = 1 + x2/3 (a2/3 x2/3 )dx = a1/3 x1/3 dx


Therefore, (using symmetry on the interval a x a)

-a

y = (a2/3- x 2/3)

3/2

2(a2/3 x2/3 )3/2 a1/3 x1/3 dx

a
= (4)(2/5)(3/2)a1/3 (a2/3 x2/3 )5/2

A=

2yds = 2

= (12/5)a2

4G-7 a) Top half: y = a2 (x b)2 , y = (b x)/y. Hence,

ds = 1 + (b x)2 /y 2 dx = (y 2 + (b x)2 )/y 2 dx = (a/y)dx


Since we are only covering the top half we double the integral for area:

b+a
xdx

A = 2xds = 4a
2
a (x b)2
ba

b)

We need to rotate two curves x2 = b +


16

a2 y 2

132

4. Applications of integration

E. Solutions to 18.01 Exercises


y = a 2- (x-b)
a

a
b

y = - a 2- (x-b) 2
upper and lower surfaces are
symmetrical and equal

and x1 = b

a2 y 2 around the y-axis. The value

dx2 /dy = (dx1 /dy) = y/ a2 y 2

So in both cases,
ds =

1 + y 2 /(a2 y 2 )dy = (a/ a2 y 2 )dy

The integral is

x = b + a 2- y 2

x = b - a -2 y 2
inner and outer surfaces are
not symmetrical and not equal

A=

2x2 ds +

2x1 ds =

But x1 + x2 = 2b, so

A = 4ab
a

ady
2(x1 + x2 )
a2 y 2
a

dy

2
a y2

c) Substitute y = a sin , dy = a cos d to get


/2
/2
a cos d
A = 4ab
d = 4 2 ab
= 4ab
a
cos

/2
/2

4H. Polar coordinate graphs


4H-1 We give the polar coordinates in the form (r, ):
17

133

E. Solutions to 18.01 Exercises

4. Applications of integration

a) (3,
/2) b) (2, ) c) (2, /3) d) (2 2, 3/4)
e) ( 2, /4 or 7/4)
f) (2,
/2 or 3/2)
g) (2, /6 or 11/6)
h) (2 2, 3/4 or 5/4)
4H-2 a) (i) (x a)2 + y 2 = a2 x2 2ax + y 2 = 0 r2 2ar cos = 0
r = 2a cos .
(ii) OP Q = 90o , since it is an angle inscribed in a semicircle.

In the right triangle OPQ, |OP | = |OQ| cos , i.e., r = 2a cos .

b) (i) Analogous to 4H-2a(i); ans: r = 2a sin .


(ii) analogous to 4H-2a(ii); note that OQP = , since both angles are comple
ments of P OQ.
c) (i) OQP is a right triangle, |OP | = r, and P OQ = .

The polar equation is r cos( ) = a, or in expanded form,

r(cos cos + sin sin ) = a


, or nally
x
y
+
= 1,
A B
a
, sin =
since from the right triangles OAQ and OBQ, we have cos =
A
a
cos BOQ = .
B
d) Since |OQ| = sin , we have:
if P is above the x-axis, sin > 0, OP | = |OQ| |QR|, or r = a a sin ;
if P is below the x-axis, sin < 0, OP | = |OQ| + |QR|, or r = a + a| sin | =
a a sin . Thus the equation is r = a(1 sin ).
e) Briey, when P = (0, 0), |P Q||P R| = a a = a2 , the constant.
Using the law of cosines,
|P R|2 = r2 + a2 2ar cos ;
|P Q|2 = r2 + a2 2ar cos( ) = r2 + a2 + 2ar cos
Therefore
|P Q|2 |P R|2 = (r2 + a2 )2 (2ar cos )2 = (a2 )2
which simplies to
r2 = 2a2 cos 2.
4H-3 a) r = sec = r cos = 1 = x = 1
b) r = 2a cos = r2 = r 2a cos = 2ax = x2 + y 2 = 2ax
c) r = (a + b cos ) (This gure is a cardiod for a = b, a limacon with a loop
for 0 < a < b, and a limacon without a loop for a > b > 0.)
18

134

4. Applications of integration

E. Solutions to 18.01 Exercises

r2 = ar + br cos = ar + bx = x2 + y 2 = a x2 + y 2 + bx

b=|c|
parabola

y
r

2a
x

b>|c| ellipse

hyperbola b<|c|

limacon a<b

(d)

cardioid (a=b)

8b

8a

limacon a>b

8c

r = a/(b + c cos )

8d

r(b + c cos ) = a

= rb = a cx

rb + cx = a

2 2

r b = a2 2acx + c2 x2

= a2 2acx + (c2 b2 )x2 b2 y 2 = 0


r = a sin(2)

(e)

r = a cos 2

r = 2a sin cos = 2axy/r2

r3 = 2axy

r2 = a2sin 2

r2 = a cos 2

r = a sin 2

r = a cos(2) = a(2 cos2 1) = a(

f)

(x2 + y 2 )3/2 = 2axy

2x2
1) = (x2 + y 2 )3/2 =
+ y2

x2

a(x2 y 2 )
g) r2 = a2 sin(2) = 2a2 sin cos = 2a2
2axy
h) r2 = a2 cos(2) = a2 (

i)

xy
= r4 = 2a2 xy = (x2 +y 2 )2 =
r2

2x2
1) = (x2 + y 2 )2 = a2 (x2 y 2 )
+ y2

x2

r = ea = ln r = a = ln

x2 + y 2 = a tan1

y
x

4I. Area and arclength in polar coordinates


19

135

E. Solutions to 18.01 Exercises


4I-1

4. Applications of integration

(dr/d)2 + r2 d
a) sec2 d
b) 2ad
c)

d)

a b2 + c2 + 2bc cos
d
(b + c cos )2

a2 + b2 + 2ab cos d

e) a 4 cos2 (2) + sin2 (2)d

f) a 4 sin2 (2) + cos2 (2)d


g) Use implicit dierentiation:
2rr = 2a2 cos(2) = r = a2 cos(2)/r = (r )2 = a2 cos2 (2)/ sin(2)
Hence, using a common denominator and cos2 + sin2 = 1,

a
ds = a2 cos2 (2)/ sin(2) + a2 sin(2)d =
d
sin(2)

h) This is similar to (g):


ds =

i)

a
cos(2)

1 + a2 ea d

4I-2 dA = (r2 /2)d. The main diculty is to decide on the endpoints of integra
tion. Endpoints are successive times when r = 0.
cos(3) = 0 = 3 = /2 + k = = /6 + k/3,

Thus,

/6

(a2 cos2 (3)/2)d = a2

A=
/6

k an integer.

/6

cos2 (3)d.

(Stop here in Unit 4. Evaluated in Unit 5.)

4I-3 A =

(r /2)d =
0

(e6 /2)d = (1/12)e6 0 = (e6 1)/12


20

136

4. Applications of integration

E. Solutions to 18.01 Exercises

= /6

=/6
three-leaf rose
three empty sectors

e3

4I-4 Endpoints are successive time when r = 0.

Thus, A =

sin(2) = 0 = 2 = k, k an integer.
/2
/2
(r2 /2)d =
(a2 /2) sin(2)d = (a2 /4) cos(2)0 = a2 /2.
0

4I-5 r = 2a cos , ds = 2ad, /2 < < /2. (The range was chosen carefully so
that r > 0.) Total length of the circle is 2a. Since the upper and lower semicircles
are symmetric, it suces to calculate the average over the upper semicircle:

2a
r = 2a cos

/2
/2

1
4a
4a
sin
=
2a cos (2a)d =

a 0
0
4I-6 a) Since the upper and lower halves of the cardiod are symmetric, it suces
to calculate the average distance to the x-axis just for a point on the upper half.
We have r = a(1 cos ), and the distance to the x-axis is r sin , so

1
r sin d =

a(1 cos ) sin d =


0

21

a
2a

(1 cos )2 =
2

137

E. Solutions to 18.01 Exercises

4. Applications of integration
P
r
0

(b)

(dr/d)2 + r2 d = a (1 cos )2 + sin2 d

using the half angle formula.


= a 2 2 cos d = 2a sin(/2)d,
2
2
arclength =
2a sin(2)d = 4a cos(/2)|0 = 8a

ds =

For the average, dont use the half-angle version of the formula for ds, and use the
interval < < , where sin is odd:

1
1
Average =
|r sin |a 2 2 cos d =
| sin | 2a2 (1 cos )3/2 d
8a
8a

2a
2a
4

(1 cos )3/2 sin d =


(1 cos )5/2 = a
=

5
10
4
0
0

4I-7 dx = a sin d. So the semicircle y > 0 has area


0

a
2
ydx =
a sin (a sin )d = a
a

But

sin2 d

1
sin d =
2
2

(1 cos(2)d = /2
0

So the area is a2 /2 as it should be for a semicircle.


Arclength: ds2 = dx2 + dy 2
= (ds)2 = (a sin d)2 + (a cos d)2 = a2 (sin2 d + cos2 d)(d)2
= ds = ad (obvious from picture).

ds

-a

ds =

ad = 2a
0

22

138

4. Applications of integration

E. Solutions to 18.01 Exercises

4J. Other applications

4J-1 Divide the water in the hole into n equal circular discs of thickness y.
2
1
y
Volume of each disc:
2

kyi y.
Energy to raise the disc of water at depth yi to surface:
4
Adding up the energies for the dierent discs, and passing to the limit,
E = lim

100

kyi y =
0

k y 2
ky dy =
4
4 2

100
=
0

k104
.
8

4J-2 Divide the hour into n equal small time intervals t.


At time ti , i = 1, . . . , n, there are x0 ekti grams of material, producing approx
imately rx0 ekti t radiation units over the time interval [ti , ti + t].
Adding and passing to the limit,
R = lim

r x0 e

kti

t =

60

r x0 e
0

kt

ekt
dt = r x0
k

60
=
0

r x0
1 e60k .
k

4J-3 Divide up the pool into n thin concentric cylindrical shells, of radius ri ,
i = 1, . . . , n, and thickness r.
The volume of the i-th shell is approximately 2 ri D r.
k
The amount of chemical in the i-th shell is approximately
2 ri D r.
1 + ri2

Adding, and passing to the limit,

R
k
r
2
r
D
r
=
2kD
dr
i
2
n
1
+
r
1
+
r2
0
i
1
R
2
= kD ln(1 + r )
= kD ln(1 + R2 ) gms.

A = lim

4J-4 Divide the time interval into n equal small intervals of length t by the
points ti , i = 1, . . . , n.
The approximate number of heating units required to maintain the temperature
at 75o over the time interval [ti , ti + t]: is

ti
75 10 6 cos
k t.
12
23

139

E. Solutions to 18.01 Exercises

4. Applications of integration

Adding over the time intervals and passing to the limit:

ti

k t

total heat = lim


75 10 6 cos
n
12
1

24
t
dt
=
k 75 10 6 cos
12
0

24
24
t
120
t
dt = k 15t +
sin
= 360k.
=
k 15 + 10 cos
12 0
12

0
4J-5 Divide the month into n equal intervals of length t by the points ti , i =
1, . . . , n.
Over the time interval [ti .ti + t], the number of units produced is about (10 +
ti ) t.
The cost of holding these in inventory until the end of the month is c(30
ti )(10 + ti ) t.
Adding and passing to the limit,
n

total cost = lim


c(30 ti )(10 + ti ) t
n

=
0

30

30
t3
c(30 t)(10 + t) dt = c 300t + 10t2
= 9000c.
3 0

4J-6 Divide the water in the tank into thin horizontal slices of width
dy.
If the slice is at height y above the center of the tank, its radius is r2 y 2 .
This formula for the radius of the slice is correct even if y < 0 i.e., the slice is
below the center of the tank as long as r < y < r, so that there really is a slice
at that height.
Volume of water in the slice = (r2 y 2 ) dy
Weight of water in the slice = w(r2 y 2 ) dy
Work to lift this slice from the ground to the height h+y = w(r2 y 2 ) dy (h+y).
r
Total work =
w(r2 y 2 )(h + y) dy
r
r
= w
(r2 h + r2 y hy 2 y 3 )
r
r

r2 y 2
hy 3
y4
= w r2 hy +

.
2
3
4 r
In this last line, the even powers of y have the same value at r and r, so contribute
0 when it is evaluated; we get therefore

y3
r3
4
= 2wh r3
= whr3 .
= wh r2 y
3 r
3
3

24

140

MIT OpenCourseWare
http://ocw.mit.edu

18.01SC Single Variable Calculus


Fall 2010

For information about citing these materials or our Terms of Use, visit: http://ocw.mit.edu/terms.

141

18.01 EXERCISES

Unit 5. Integration techniques


5A. Inverse trigonometric functions; Hyperbolic functions

5A-1 Evaluate

a) tan1

b) sin1 ( 3/2)

c) If = tan1 5, then evaluate sin , cos , cot , csc , and sec .


d) sin1 cos(/6)

e) tan1 tan(/3)

f) tan1 tan(2/3)

g) lim tan1 x.

5A-2 Calculate

a)
1

dx
2
x +1

2b

b)

dx
2
x + b2

c)
1

dx

.
1 x2

5A-3 Calculate the derivative with respect to x of the following

a) sin1

c) ln(x +

x1
x+1

x2 + 1)

e) sin1 (x/a)

b) tanh x
d) y such that cos y = x, 0 x 1 and 0 y /2.
f) sin1 (a/x)

g) tan1 (x/ 1 x2 ) h) sin1 1 x

5A-4 a) If the tangent line to y = cosh x at x = a goes through the origin, what
equation must a satisfy?
b) Solve for a using Newtons method.

5A-5 a) Sketch the graph of y = sinh x, by nding its critical points, points of
inection, symmetries, and limits as x and .
COPYRIGHT DAVID JERISON AND MIT 1996, 2003
1

142

E. 18.01 Exercises

5. Integration techniques

b) Give a suitable denition for sinh1 x, and sketch its graph, indicating
the domain of denition. (The inverse hyperbolic sine.)

c) Find

d
sinh1 x.
dx

dx

2
a + x2

d) Use your work to evaluate

5A-6 a) Find the average value of y with respect to arclength on the semicircle
x2 + y 2 = 1, y > 0, using polar coordinates.
b) A weighted average of a function is

b
f (x)w(x)dx
a

w(x)dx
a

Do part (a) over again expressing arclength as ds = w(x)dx. The change of variables
needed to evaluate the numerator and denominator will bring back part (a).

c) Find the average height of 1 x2 on 1 < x < 1 with respect to dx.


Notice that this diers from part (b) in both numerator and denominator.

5B. Integration by direct substitution

Evaluate the following integrals

5B-1.

5B-4.

5B-7.

x x2 1dx

cos xdx
2 + 3 sin x

5B-5.

6xdx

x2 + 4

5B-8.

5B-13.

5B-11.

sin x cos xdx

5B-6.

tan 4xdx

5B-9.

sec2 9xdx

ln xdx
x

sin 7xdx

sec 9xdx

5B-3.

5B-10.

e8x dx

5B-2.

5B-12.

ex (1 + ex )1/3 dx
2

xex dx

x2 dx
. Hint: Try u = x3 .
1 + x6

Evaluate the following integrals by substitution and changing the limits of integration.
2

143

5. Integration techniques

E. 18.01 Exercises

/3

sin x cos xdx

5B-14.

5B-15.

(ln x)3/2 dx
x

5B-16.
1

tan1 xdx
1 + x2

5C. Trigonometric integrals

Evaluate the following

sin2 xdx

5C-1.

5C-4.

5C-7.

cos (3x)dx

5C-5.

sin2 (4x) cos2 (4x)dx

5C-8.

(tan x + cot x)2 dx

5C-11.

5C-10.

sin3 (x/2)dx

5C-2.

sin x cos xdx

5C-3.

5C-6.

tan2 (ax) cos(ax)dx 5C-9.

sin4 xdx
sec4 xdx
sin3 x sec2 xdx

sin x cos(2x)dx (Use double angle formula.)

5C-12.

sin x cos(2x)dx (See 27.)


0

5C-13. Find the length of the curve y = ln sin x for /4 x /2.


5C-14. Find the volume of one hump of y = sin ax revolved around the x-axis.

5D. Integration by inverse substitution

Evaluate the following integrals

5D-1.

dx
(a2 x2 )3/2


5D-4.
a2 + x2 dx

x3 dx

a2 x2
2
a x2 dx
5D-5.
x2

5D-2.

5D-3.

5D-6.

(x + 1)dx
4 + x2
x2

a2 + x2 dx

(For 5D-4,6 use x = a sinh y, and cosh2 y = (cosh(2y) + 1)/2, sinh 2y = 2 sinh y cosh y.)
2
x a2 dx
5D-7.
x2

5D-8.
3

x x2 9dx

144

E. 18.01 Exercises

5. Integration techniques

5D-9. Find the arclength of y = ln x for 1 x b.

Completing the square

Calculate the following integrals

dx
(x2 + 4x + 13)3/2

5D-10.

dx

2x x2

5D-13.

5D-11.

5D-14.

8 + 6x x2 dx

x 8 + 6x x2 dx

5D-12.

xdx

2
x + 4x + 13

2
4x 4x + 17dx
5D-15.
2x 1

5E. Integration by partial fractions

5E-1.

5E-4.

dx
dx
(x 2)(x + 3)

5E-2.

3x2 + 4x 11
dx
(x2 1)(x 2)

5E-5.

xdx
dx
(x 2)(x + 3)

5E-3.

3x + 2
dx
x(x + 1)2

5E-6.

xdx
dx
(x2 4)(x + 3)

2x 9
dx
(x2 + 9)(x + 2)

5E-7 The equality (1) of Notes F is valid for x = 1, 2. Therefore, the equality (4)

is also valid only when x =


1, 2, since it arises from (1) by multiplication.

Why then is it legitimate to substitute x = 1 into (4)?

5E-8 Express the following as a sum of a polynomial and a proper rational function

a)

d)

x2
1

b)

x+2
3x 1

e)

x2

x3
1

x2

c)

x2
3x 1

x8
(just give the form of the solution)
(x + 2)2 (x 2)2

5E-9 Integrate the functions in Problem 5E-8.

5E-10 Evaluate the following integrals

a)

dx
x3 x

b)

(x + 1)dx
(x 2)(x 3)
4

c)

(x2 + x + 1)dx
x2 + 8x

145

5. Integration techniques

d)

g)

E. 18.01 Exercises

(x2 + x + 1)dx
x2 + 8x

e)

x3 dx
(x + 1)2 (x 1)

h)

dx
x 3 + x2

f)

(x2 + 1)dx
x3 + 2x2 + x

(x2 + 1)dx
x2 + 2x + 2

5E-11 Solve the dierential equation dy/dx = y(1 y).


5E-12 This problem shows how to integrate any rational function of sin and cos
using the substitution z = tan(/2). The integrand is transformed into a rational
function of z, which can be integrated using the method of partial fractions.
a) Show that

cos =

1 z 2

,
1 + z2

sin =

2z
,
1 + z2

d =

2dz
.
1 + z2

Calculate the following integrals using the substitution z = tan(/2) of part (a).

b)
0

d
1 + sin

c)
0

d
(1 + sin )2

d)

sin d (Not the easiest


0

way!)
5E-13 a) Use the polar coordinate formula for area to compute the area of the
region 0 < r < 1/(1 + cos ), 0 /2. Hint: Problem 12 shows how the substi
tution z = tan(/2) allows you to integrate any rational function of a trigonometric
function.
b) Compute this same area using rectangular coordinates and compare your
answers.

5F. Integration by parts. Reduction formulas

Evaluate the following integrals

5F-1 a)

xa ln xdx (a =
1)

b) Evaluate the case a = 1 by substitu

tion.

5F-2 a)

xe dx

b)

2 x

x e dx

c)
5

x3 ex dx

146

E. 18.01 Exercises

5. Integration techniques

d) Derive the reduction formula expressing

5F-3 Evaluate

5F-4 Evaluate

xn eax dx in terms of

xn1 eax dx.

sin1 (4x)dx

ex cos xdx. (Integrate by parts twice.)

5F-5 Evaluate

cos(ln x)dx. (Integrate by parts twice.)

5F-6 Show the substitution t = ex transforms the integral


Use a reduction procedure to evaluate this integral.

xn ex dx, into

(ln t)n dt.

147

MIT OpenCourseWare
http://ocw.mit.edu

18.01SC Single Variable Calculus


Fall 2010

For information about citing these materials or our Terms of Use, visit: http://ocw.mit.edu/terms.

148

SOLUTIONS TO 18.01 EXERCISES

Unit 5. Integration techniques

5A. Inverse trigonometric functions; Hyperbolic functions

5A-1 a) tan1

3=
3

b) sin1 (

)=
2
3

c)
tan = 5 implies sin = 5/ 26, cos = 1/ 26, cot = 1/5, csc = 26/5,
sec = 26 (from triangle)

)=
d) sin1 cos( ) = sin1 (
6
2
3
f) tan1 tan(

e) tan1 tan( ) =
3
3

) = tan1 tan(
)=
3
3
3

g) lim tan1 x =
x

5A-2

dx

1 2
x
= tan1 2
=
tan
2+1
1
4
x
1

2b
2b

dx
d(by)
(put x = by)
b)
=
2
2
(by)2 + b2
x +b
b
b
2
dy
1

= (tan1 2 )
=
2
b
4
1 b(y + 1)
1

dx

1

= sin1 x1 =
c)
=
2
2
2
1

x
1
a)

COPYRIGHT DAVID JERISON AND MIT 1996, 2003


1

.
2

149

E. Solutions to 18.01 Exercises


5A-3 a) y =

5. Integration techniques

x1
1
(x + 1)
, so 1 y 2 = 4x/(x + 1)2 , and
= . Hence
2
x+1
2 x
1y
dy
2
=
dx
(x + 1)2
d
dy/dx
sin1 y =
dx
1 y2
2
(x + 1)

=
(x + 1)2
2 x
1

=
(x + 1) x

b) sech2 x = 1/ cosh2 x = 4/(ex + ex )2


c) y = x +

x2 + 1, dy/dx = 1 + x/ x2 + 1.

d
dy/dx
1 + x/ x2 + 1
1

ln y =
=
=
2
2
dx
y
x+ x +1
x + 1

d) cos y = x = ( sin y)(dy/dx) = 1


dy
1
1
=
=
dx
sin y
1 x2

e) Chain rule:

d
1
1
1

=
sin1 (x/a) =
dx
a 2 x2
1 (x/a)2 a

f) Chain rule:

a
d
1

a
sin1 (a/x) =
2 =
2
x
dx
x x 2 a2
1 (a/x)

g) y = x/ 1 x2 , dy/dx = (1 x2 )3/2 , 1 + y 2 = 1/(1 x2 ). Thus


dy/dx
1
d
= (1 x2 )3/2 (1 x2 ) =
tan1 y =
dx
1 + y2
1 x2
Why is this the same as the derivative of sin1 x?
h) y =

x 1, dy/dx = 1/2 x 1, 1 y 2 = x. Thus,

dy/dx
1

d
sin1 y =
=
dx
2 x(1 x)

1 y2
2

150

5. Integration techniques

E. Solutions to 18.01 Exercises

5A-4 a) y = sinh x. A tangent line through the origin has the equation y = mx.
If it meets the graph at x = a, then ma = cosh(a) and m = sinh(a). Therefore,
a sinh(a) = cosh(a) .
b) Take the dierence:
F (a) = a sinh(a) cosh(a)
Newtons method for nding F (a) = 0, is the iteration
an+1 = an F (an )/F (an ) = an tanh(an ) + 1/an
With a1 = 1, a2 = 1.2384, a3 = 1.2009, a4 = 1.19968. A serviceable approximation
is
a 1.2
(The slope is m = sinh(a) 1.5.) The functions F and y are even. By symmetry,
there is another solution a with slope sinh a.
5A-5 a)
ex ex
2
x
e
+
ex
y = cosh x =
2
y = sinh x
y = sinh x =

y is never zero, so no critical points. Inection point x = 0; slope of y is 1 there.


y is an odd function, like ex /2 for x >> 0.

y = sinh 1x

y = sinh x

b) y = sinh1 x x = sinh y. Domain is the whole x-axis.


c) Dierentiate x = sinh y implicitly with respect to x:
1 = cosh y

dy
dx

dy
1
1
=
=
dx
cosh y
sinh2 y + 1
d sinh1 x
1
=
2
dx
x +1
3

151

E. Solutions to 18.01 Exercises

5. Integration techniques

d)

dx

=
2
x + a2

dx
a

x2

+ a2 /a2
d(x/a)

(x/a)2 + 1

= sinh
5A-6 a)

b) y =

(x/a) + c

sin d = 2/
0

1 x2 = y = x/ 1 x2 = 1 + (y )2 = 1/(1 x2 ). Thus

ds = w(x)dx = dx/ 1 x2 .

Therefore the average is

dx
dx
2

1x
2
1x
1 x2
1
1

The numerator is

dx = 2. To see that these integrals are the same as the ones


1

in part (a), take x = cos (as in polar coordinates). Then dx = sin d and the
limits of integral are from = to = 0. Reversing the limits changes the minus
back to plus:
1

dx
2
1x
=
sin d
1 x2
0
1
1

dx

=
d =
1 x2
1
0
(The substitution x = sin t works similarly, but the limits of integration are /2
and /2.)
c) (x = sin t, dx = cos tdt)

/2

1 1
1 /2
2
2
cos tdt =
cos2 tdt
1 x dx =
2 1
2 /2
0
/2
1 + cos 2t
=
dt
2
0
= /4

5B. Integration by direct substitution

Do these by guessing and correcting the factor out front. The substitution used
implicitly is given alongside the answer.
4

152

5. Integration techniques

E. Solutions to 18.01 Exercises

3
1

x x2 1dx = (x2 1) 2
+ c (u = x2 1, du = 2xdx)

5B-1

e8x dx =

5B-2

1 8x
e + c (u = 8x, du = 8dx)
8

ln xdx
1
= (ln x)2 + c (u = ln x, du = dx/x)
x
2

cos xdx
ln(2 + 3 sin x)
=
+ c (u = 2 + 3 sin x, du = 3 cos xdx)
2 + 3 sin x
3

sin2 x cos xdx =

5B-3

5B-4

5B-5

5B-6

sin 7xdx =

sin x3

+ c (u = sin x, du = cos xdx)


3

cos 7x
+ c (u = 7x, du = 7dx)
7

6xdx

= 6 x2 + 4 + c (u = x2 + 4, du = 2xdx)
x2 + 4

5B-7

5B-8 Use u = cos(4x), du = 4 sin(4x)dx,

sin(4x)dx
du
=
cos(4x)
4u
ln u
ln(cos 4x)
+c
=
+c=
4
4

tan 4xdx =

ex (1 + ex )1/3 dx =

5B-9

sec 9xdx =

5B-10

5B-11

5B-12

ln(sec(9x) + tan(9x)) + c (u = 9x, du = 9dx)


9

sec2 9xdx =

tan 9x
+ c (u = 9x, du = 9dx)
9

ex
dx =
+ c (u = x2 , du = 2xdx)
2

x2

xe

3
(1 + ex )2/3 + c (u = 1 + ex , du = ex dx)
2

153

E. Solutions to 18.01 Exercises

5. Integration techniques

5B-13 u = x3 , du = 3x2 dx implies

du
tan1 u
x2 dx
=
=
+c
3(1 + u2 )
3
1 + x6
tan1 (x3 )
=
+c
3
/3
sin /3
3
5B-14
sin x cos xdx =
u3 du (u = sin x, du = cos xdx)
0

sin 0

3/2

=
0

1
1

9
64

5B-15

3/2

u3 du = u4 /4
=

(ln x)3/2 dx
=
x

ln e

u3/2 du (u = ln x, du = dx/x)

ln 1

1
2

y 3/2 dy = (2/5)y 5/2 =


5
0

5B-16

tan1 xdx
=
1 + x2

tan1 1

udu (u = tan1 x, du = dx/(1 + x2 )


tan1 (1)

/4
u2
=
udu =
=0
2 /4
/4
/4

(tan x is odd and hence tan1 x is also odd, so the integral had better be 0)

5C. Trigonometric integrals

5C-1

sin2 xdx =

5C-2

1 cos 2x
x sin 2x
dx =
+c
2
2
4

sin (x/2)dx =

(1 cos (x/2)) sin(x/2)dx =

(put u = cos(x/2), du = (1/2) sin(x/2)dx)


3

= 2u +

2cos(x/2)
2u3
+ c = 2 cos(x/2) +
+c
3
3
6

2(1 u2 )du

154

5. Integration techniques

5C-3

sin4 xdx =

cos2 (2x)
dx =
4

E. Solutions to 18.01 Exercises

1 cos 2x 2
) dx =
2

1 2 cos 2x + cos2 2x
dx
4

1 + cos 4x
x sin 4x
dx = +
+c
8
32
8

Adding together all terms:

5C-4

sin4 xdx =

cos3 (3x)dx =

3x 1
1
sin(2x) +
sin(4x) + c
4
32
8

(1 sin2 (3x)) cos(3x)dx =

1 u2
du (u = sin(3x),
3

du = 3 cos(3x)dx)
3

sin(3x) sin(3x)
u u3

+c=

+c
3
9
3
9

5C-5

sin3 x cos2 xdx =

(1 cos2 x) cos2 x sin xdx =

(1 u2 )u2 dy (u =

cos x, du = sin xdx)

u3
u5
cos x3
cos x5
+
+c=
+
+c
3
5
3
5

sec xdx =

5C-6

(1 + tan x) sec xdx =

(1 + u2 )du (u = tan x, du =

sec2 xdx)

=u+

5C-7

tan3 x
u3
+ c = tan x +
+c
3
3

sin (4x) cos (4x)dx =

sin2 8xdx
=
4

(1 cos 16x)dx
1 sin 16x
=
+c
8
8
128

A slower way is to use


2

sin (4x) cos (4x) =

1 cos(8x)
2

1 + cos(8x)
2

multiply out and use a similar trick to handle cos2 (8x).


7

155

E. Solutions to 18.01 Exercises

5. Integration techniques

5C-8
sin2 (ax)
dx
cos(ax)

1 cos2 (ax)
=
dx
cos(ax)

= (sec(ax) cos(ax))dx

tan2 (ax) cos(ax)dx =

1
1
ln(sec(ax) + tan(ax)) sin(ax) + c
a
a

5C-9

1 cos2 x
sin xdx
cos2 x

1 u2
=
du
(u = cos x, du = sin xdx)
u2
1
= u + + c = cos x + sec x + c
u

sin3 x sec2 xdx =

5C-10

(tan x + cot x) dx =

tan x + 2 + cot xdx =

sec2 x + csc2 xdx

= tan x cot x + c

5C-11

sin x cos(2x)dx

sin x(2 cos x 1)dx =

(1 2u2 )du (u = cos x, du sin xdx)

2
2
= u u3 + c = cos x cos3 x + c
3
3

5C-12
0

2
2
3
sin x cos(2x)dx = cos x cos x =
(See 27.)
3
3
0

1 + (y )2 dx = 1 + cot2 xdx = csc xdx.


/2
/2

arclength =
csc xdx = ln(csc x + cot x)
= ln(1 + 2)

/4

5C-13 ds =

/4

5C-14
0

/a

sin2 (ax)dx =

/a

(1/2)(1 cos(2ax))dx = 2 /2a


0

156

5. Integration techniques

E. Solutions to 18.01 Exercises

5D. Integration by inverse substitution


5D-1 Put x = a sin , dx = a cos d:

x
dx
1
1
= 2
sec2 d = 2 tan + c =
+c
2
2
3/2
2
a
a
(a x )
a a2 x2
5D-2 Put x = a sin , dx = a cos d:

x3 dx
3
3
3

sin d = a
(1 cos2 ) sin d
=a
a2 x2
= a3 ( cos + (1/3) cos3 ) + c

= a2 a2 x2 + (a2 x2 )3/2 /3 + c
5D-3 By direct substitution (u = 4 + x2 ),

xdx
= (1/2) ln(4 + x2 ) + c
4 + x2
Put x = 2 tan , dx = 2 sec2 d,

dx
1
=
d = /2 + c
2
4 + x2
In all,

(x + 1)dx
= (1/2) ln(4 + x2 ) + (1/2) tan1 (x/2) + c
4 + x2
5D-4 Put x = a sinh y, dx = a cosh ydy. Since 1 + sinh2 y = cosh2 y,

a2
2
2
2
2
a + x dx = a
cosh ydy =
(cosh(2y) 1)dy

2
= (a2 /4) sinh(2y) a2 y/2 + c = (a2 /2) sinh y cosh y a2 y/2 + c

= x a2 + x2 /2 a2 sinh1 (x/a) + c
5D-5 Put x = a sin , dx = a cos d:
2

a x2 dx
=
cot2 d
x2

= (csc2 1)d = ln(csc + cot ) + c

= ln(a/x + a2 x2 /x) sin1 (x/a) + c


5D-6 Put x = a sinh y, dx = a cosh ydy.

x2 a2 + x2 dx = a4 sinh2 y cosh2 ydy

2
4
4
= (a /2) sinh (2y)dy = a /4 (cosh(4y) 1)dy
= (a4 /16) sinh(4y) a4 y/4 + c
= (a4 /8) sinh(2y) cosh(2y) a4 y/4 + c
= (a4 /4) sinh y cosh y(cosh2 y + sinh2 y) a4 y/4 + c

= (1/4)x a2 + x2 (2x2 + a2 ) (a4 /4) sinh1 (x/a) + c


9

157

E. Solutions to 18.01 Exercises

5. Integration techniques

5D-7 Put x = a sec , dx = a sec tan d:


2

x a2 dx
tan2 d
=
sec
x2

(sec2 1)d
=
= (sec cos )d
sec
= ln(sec + tan ) sin + c

= ln(x/a + x2 a2 /a) x2 a2 /x + c

= ln(x + x2 a2 ) x2 a2 /x + c1 (c1 = c ln a)
5D-8 Short way: u = x2 9, du = 2xdx,

x x2 9dx = (1/3)(x2 9)3/2 + c

direct substitution

Long way (method of this section): Put x = 3 sec , dx = 3 sec tan d.


x x2 9dx = 27 sec2 tan2 d

= 27 tan2 d(tan ) = 9 tan3 + c


= (1/3)(x2 9)3/2 + c

(tan = x2 9/3). The trig substitution method does not lead to a dead end,
but its not always fastest.
5D-9 y = 1/x, ds =

1 + 1/x2 dx, so
b

1 + 1/x2 dx

arclength =
1

Put x = tan , dx = sec2 d,


2

x + 1dx
=
x

sec
sec2 d
tan
sec (1 + tan2 )
d
tan
(csc + sec tan )d

= ln(csc + cot ) + sec + c

= ln( x2 + 1/x + 1/x) + x2 + 1 + c

= ln( x2 + 1 + 1) + ln x + x2 + 1 + c

arclength = ln( b2 + 1 + 1) + ln b + b2 + 1 + ln( 2 + 1) 2

10

158

5. Integration techniques

E. Solutions to 18.01 Exercises

Completing the square

5D-10

dx
=
2
(x + 4x + 13)3/2

dx
(x + 2 = 3 tan , dx =
((x + 2)2 + 32 )3/2

3 sec2 d)
=

1
9

cos d =

(x + 2)
1
sin + c =
+c
9
9 x2 + 4x + 13

5D-11


x 8 + 6x x2 dx = x 1 (x 3)2 dx

= (sin + 3) cos2 d
= (1/3) cos3 + (3/2)

(x 3 = sin , dx = cos d)

(cos 2 + 1)d

= (1/3) cos3 + (3/4) sin 2 + (3/2) + c


= (1/3) cos3 + (3/2) sin cos + (3/2) + c
= (1/3)(8 + 6x x2 )3/2

+ (3/2)(x 3) 8 + 6x x2 + (3/2) sin1 (x 3) + c


5D-12


1 (x 3)2 dx (x 3 = sin , dx = cos d)
8 + 6x x2 dx =

= cos2 d

1
=
(cos 2 + 1)d
2
1

= sin 2 + + c
4
2
1

= sin cos + + c
2
2

(x 3) 8 + 6x x2
sin1 (x 3)
=
+
+c
2
2

dx
dx

5D-13
=
. Put x 1 = sin , dx = cos d.
2
2x x
1 (x 1)2

= d = + c = sin1 (x 1) + c

xdx
xdx

5D-14
. Put x + 2 = 3 tan , dx = 3 sec2 .
=
2
x + 4x + 13
(x + 2)2 + 32

= (3 tan 2) sec d = 3 sec 2 ln(sec + tan ) + c

= x2 + 4x + 13 2 ln( x2 + 4x + 13/3 + (x + 2)/3) + c

= x2 + 4x + 13 2 ln( x2 + 4x + 13 + (x + 2)) + c1 (c1 = c ln 3)


11

159

E. Solutions to 18.01 Exercises

5. Integration techniques


2
(2x 1)2 + 42 dx
4x 4x + 17dx
5D-15
=
2x 1
2x 1
(put 2x 1 = 4 tan , dx = 2 sec2 d as in Problem 9)

sec
sec2 d
=2
tan

sec (1 + tan2 )
=2
d
tan

= 2 (csc + sec tan )d


= 2 ln(csc + cot ) + 2 sec + c

= 2 ln( 4x2 4x + 17/(2x 1) + 4/(2x 1)) + 4x2 4x + 17/2 + c

= 2 ln( 4x2 4x + 17 + 4) + 2 ln(2x 1) + 4x2 4x + 17/2 + c

5E. Integration by partial fractions

5E-1

5E-2

5E-3

5E-4

1
1/5
1/5
=
+
(cover up)
x2 x+3
(x 2)(x + 3)

dx
= (1/5) ln(x 2) (1/5) ln(x + 3) + c
(x 2)(x + 3)
x
2/5
3/5
+
(cover up)
=
x2 x+3
(x 2)(x + 3)

xdx
= (2/5) ln(x 2) + (3/5) ln(x + 3) + c
(x 2)(x + 3)
x
1/10
1/2
3/5
=
+
+
(cover up)
x2 x+2 x+3
(x 2)(x + 2)(x + 3)

xdx
= (1/10) ln(x 2) + (1/2) ln(x + 2) (3/5) ln(x + 3)
(x2 4)(x + 3)
2
2
3
3x2 + 4x 11
dx =
+
+
(cover-up)
2
x1 x+1 x2
(x 1)(x 2)

2dx
2dx
3dx
+
+
= 2 ln(x 1) 2 ln(x + 1) + 3 ln(x 2) + c
x1 x+1 x2

12

160

5. Integration techniques
5E-5

E. Solutions to 18.01 Exercises

3x + 2
2
B
1
= +
+
(coverup); to get B, put say x = 1:
2
x x + 1 (x + 1)2

x(x + 1)
5
B
1

=2+ +
= B = 2
4
2
4

3x + 2
1
dx = 2 ln x 2 ln(x + 1)
+c
2
x(x + 1)
x+1
2x 9
Ax + B
C
5E-6
= 2
+
x +9
x+2
(x2 + 9)(x + 2)
By cover-up, C = 1. To get B and A,
x = 0 =

9
B
1
=
= B = 0
92
9
2

7
A
1
=

= A = 1
10 3
10 3

1
2x 9
dx = ln(x2 + 9) ln(x + 2) + c
2
2
(x + 9)(x + 2)
x = 1 =

5E-7 Instead of thinking of (4) as arising from (1) by multiplication by x1, think
of it as arising from
x 7 = A(x + 2) + B(x 1)
by division by x + 2; since this new equation is valid for all x, the line (4) will be
valid for x =
2, in particular it will be valid for x = 1 .
5E-8 Long division:

a)

x2
1
=1+ 2
1
x 1

x2

b)

x3
x
=x+ 2
2
x 1
x 1

c)

x2
1/9
= x/3 + 1/9 +

3x 1
3x 1

d)

x+2
1
7/3
= +
3x 1
3 3x 1

e)

x8
B3 x3 + B2 x2 + B1 x + B0
4
3
2
=
A
x
+A
x
+A
x
+A
x+A
+
4
3
2
1
0
(x + 2)2 (x 2)2
(x + 2)2 (x 2)2

5E-9 a) Cover-up gives


1
1
1/2
1/2
=
=
+
(x 1)(x + 1)
x1 x+1
x2 1
13

161

E. Solutions to 18.01 Exercises

5. Integration techniques

From 8a,
1/2
1/2
x2
=1+
+
2
x1 x+1
x 1

and

x2 dx
= x + (1/2) ln(x 1) (1/2) ln(x + 1) + c

x2 1

b) Cover-up gives

x2

x
x
1/2
1/2
=
=
+
1
(x 1)(x + 1)
x1 x+1

From 8b,
1/2
1/2
x3
+
=x+
2
x1 x+1
x 1

and

x3 dx
= x2 /2 + (1/2) ln(x 1) + (1/2) ln(x + 1) + c
x2 1

c) From 8c,

x2
dx = x2 /6 + x/9 + (1/27) ln(3x 1) + c
3x 1

d) From 8d,

x+2
dx = x/3 + (7/9) ln(3x 1)
3x 1
e) Cover-up says that the proper rational function will be written as
a2
b1
b2
a1
+
+
+
x + 2 (x + 2)2
x 2 (x 2)2
where the coecients a2 and b2 can be evaluted from the Bs using cover-up and the
coecients a1 and b1 can then be evaluated using x = 0 and x = 1, say. Therefore,
the integral has the form
A4 x5 /5 + A3 x4 /4 + A2 x3 /3 + A1 x2 /2 + A0 x + c
+ a1 ln(x 2)

a2
b2
+ b1 ln(x + 2)
x2
x+2

5E-10 a) By cover-up,
x3

1
1
1/2
1/2
1
=
=
+
+
x
x(x 1)(x + 1)
x
x1 x+1
dx
1
1
= ln x + ln(x 1) + ln(x + 1) + c
x3 x
2
2

b) By cover-up,

(x + 1)
3
4
=
+
. Therefore,
(x 2)(x 3)
x2 x3

(x + 1)
dx = 3 ln(x 2) + 4 ln(x 3) + c
(x 2)(x 3)
14

162

5. Integration techniques
c)

E. Solutions to 18.01 Exercises

(x2 + x + 1)
7x + 1
=1+ 2
. By cover-up,
x + 8x

x2 + 8x
7x + 1
1/8 57/8

7x + 1
and
=
=
+
2
x(x + 8)
x
x+8
x + 8x

(x2 + x + 1)
= x + (1/8) ln x (57/8) ln(x + 8) + c
x2 + 8x

d) Seeing double? It must be late.

e)

x3

1
1
A
B
C
= + 2+
= 2
2
+x
x (x + 1)
x
x
x+1

Use the cover-up method to get B = 1 and C = 1. For A,

x = 1 =

1
1

=A+1+
= A = 1
2
2

In all,

f)

dx
=
x3 + x2

x3


1
1
1
1
dx = ln x + ln(x + 1) + c
+ 2+
x x
x+1
x

x2 + 1
x2 + 1
A
B
C
=
= +
+
2
+ 2x + x
x(x + 1)2
x
x + 1 (x + 1)2

By cover-up, A = 1 and C = 2. For B,

2
B
2

=1+
= B = 0 and
4
2
4


1
2
2
x2 + 1
dx =
+c

dx = ln x +
x3 + 2x2 + x
x (x + 1)2
x+1
x = 1 =

g) Multiply out denominator: (x + 1)2 (x 1) = x3 + x2 x 1. Divide into


numerator:
x2 + x + 1
x3
=1+ 3
3
2
x +x x1
x + x2 x 1
Write the proper rational function as
A
B
C

x2 + x + 1
=
+
+
(x + 1)2 (x 1)
x + 1 (x + 1)2
x 1

By cover-up, B = 1/2 and C = 1/4. For A,


x = 0 = 1 = A +

1 1
5

= A =
2 4
4
15

and

163

E. Solutions to 18.01 Exercises

5. Integration techniques

x3
dx =
(x + 1)2 (x 1)


5/4
1/2
1/4
1+
+
+
dx
x + 1 (x + 1)2
x 1

= x (5/4) ln(x + 1)
+ (1/4) ln(x 1) + c
2(x + 1)

h)

(x2 + 1)dx
=
x2 + 2x + 2

(1

1 + 2x
)dx = x
2
x + 2x + 2

(2y 1)dy
(put y =
y2 + 1

x + 1)
= x ln(y 2 + 1) + tan1 y + c
= x ln(x2 + 2x + 2) + tan1 (x + 1) + c

5E-11 Separate:
dy
= dx
y(1 y)
Expand using partial fractions and integrate

1
1
(
)dy = dx
y y1
Hence,
ln y ln(y 1) = x + c
Exponentiate:

y
= ex+c = Aex (A = ec )
y1
Aex
y=
Aex 1
(If you integrated 1/(1 y) to get ln(1 y) then you arrive at
y=

Aex
Aex + 1

This is the same family of answers with A and A traded.)


5E-12 a) 1 + z 2 = 1 + tan2 (/2) = sec2 (/2). Therefore,
cos2 (/2) =

1
1 + z2

and

sin2 (/2) = 1

1
z2
=
2
1+z
1 + z2

Next,
1 z2
1
z2

=
and
2
2
1+z
1+z
1 + z2

2z
1
z2
=
sin = 2 sin(/2) cos(/2) = 2
2
1 + z2
1 + z2
1+z

cos = cos2 (/2) sin2 (/2) =

Finally,
dz = (1/2) sec2 (/2)d = (1/2)(1 + z 2 )d = d =
16

2dz
1 + z2

164

5. Integration techniques

E. Solutions to 18.01 Exercises

b)

tan /2

2dz/(1 + z 2 )
1 + 2z/(1 + z 2 )
0
tan

2dz
2dz
=
=
2
(z + 1)2
z + 1 + 2z
0
0

2
=
=2
1 + z

d
=
1 + sin

c)


2dz/(1 + z 2 )
2(1 + z 2 )dz
=
(1 + 2z/(1 + z 2 ))2
(1 + z)4
tan 0
0

2
2(1 + (y 1) )dy
=
(put y = z + 1)
y4
1

(2y 2 4y + 4)dy
=
=
(2y 2 4y 3 + 4y 4 )dy
y4
1
1

= 2y 1 + 2y 2 (4/3)y 3 = 4/3

d
=
(1 + sin )2

tan /2

2z
2dz
=
2 1 + z2
z
1
+
0

2
=2
=
1 + z 2 0

sin d =

()
0

4zdz
(1 + z 2 )2

5E-13 a) z = tan(/2) = 1 + cos = 2/(1 + z 2 ) and 0 /2 corresponds


to 0 z 1.
1
/2
d
2dz/(1 + z 2 )
A=
=
2
8/(1 + z 2 )2
2(1 + cos )
0
0
1
1
=
(1/4)(1 + z 2 )dz = (1/4)(z + z 3 /3)0 = 1/3
0

b) The curve r = 1/(1 + cos ) is a parabola:


r + r cos = 1 = r + x = 1 = r2 = (1 x)2 = y 2 = 1 2x

This is the region under y = 1 2x in the rst quadrant:


1/2
1/2

1 2xdx = (1/3)(1 2x)3/2


A=
= 1/3
0

5F. Integration by parts. Reduction formulas

5F-1 a)

x ln xdx =

xa+1
xa+1

ln xd(
) = ln x
a+1
a+1
17

xa+1 1
dx
a+1 x

165

E. Solutions to 18.01 Exercises


=

xa+1 ln x

a+1

b)

5F-2 a)

b)

c)

xa
xa+1 ln x
xa+1

dx =

+ c (a =
1)

a+1
(a + 1)2
a+1

x1 ln xdx = (ln x)2 /2 + c (u = ln x, du = dx/x)

xex dx =

x2 ex dx =

= x 2 ex 2

5. Integration techniques

xd(ex ) = x ex

x2 d(ex ) = x2 ex

ex dx = x ex ex + c

ex 2xdx

xex dx = x2 ex 2x ex + 2ex + c

3 x

x d(e ) = x e

x e dx =

ex 3x2 dx

= x3 ex 3 x2 ex dx = x3 ex 3x2 ex + 6x ex 6ex + c

d)

n ax

x e dx =

eax n n
x
=
a
a

eax
eax n
)=
x
x d(
a
a
n

eax

nxn1 dx

xn1 eax dx

5F-3

4dx
sin1 (4x)dx = x sin1 (4x) xd(sin1 (4x)) = x sin1 (4x) x
1 (4x)2

du

= x sin1 (4x) +
(put u = 1 16x2 , du = 32xdx)
8 u
1
u+c
= x sin1 (4x) +
4

1
= x sin1 (4x) +
1 16x2 + c
4
5F-4

e d(sin x) = e sin x ex sin xdx

= ex sin x ex d( cos x)

x
x
= e sin x + e cos x ex cos xdx

e cos xdx =

18

166

5. Integration techniques

Add

E. Solutions to 18.01 Exercises

ex cos xdx to both sides to get

2 ex cos xdx = ex sin x + ex cos x + c

Divide by 2 and replace the arbitrary constant c by c/2:

ex cos xdx = (ex sin x + ex cos x)/2 + c


5F-5

cos(ln x)dx = x cos(ln x)

xd(cos(ln x))

= x cos(ln x) +

sin(ln x)dx

= x cos(ln x) + x sin(ln x)

xd(sin(ln x))

= x cos(ln x) + x sin(ln x)

cos(ln x)dx

Add

cos(ln x)dx to both sides to get

2 cos(ln x)dx = x cos(ln x) + x sin(ln x) + c

Divide by 2 and replace the arbitrary constant c by c/2:

cos(ln x)dx = (x cos(ln x) + x sin(ln x))/2 + c


5F-6 Put t = ex = dt = ex dx and x = ln t. Therefore

xn ex dx = (ln t)n dt
Integrate by parts:

(ln t)n dt = t (ln t)n td(ln t)n = t(ln t)n n (ln t)n1 dt
because d(ln t)n = n(ln t)n1 t1 dt.

19

167

MIT OpenCourseWare
http://ocw.mit.edu

18.01SC Single Variable Calculus


Fall 2010

For information about citing these materials or our Terms of Use, visit: http://ocw.mit.edu/terms.

You might also like